SlideShare a Scribd company logo
Question 1: It’s an extremely __________ cure for a headache.
A. effect B. effective C. effectual D. effector
A. effect /i’fekt/ (n): tác động, ảnh hưởng
B. effective /i’fektiv/ (a): hiệu quả
C. effectual /i’fekt∫ʊəl/ (a): có hiệu lực, thành công
D. effector/i’fekt/ (n): người thực hành
Căn cứ vào trạng từ “extremely’’ đứng trước và danh từ theo sau thì
vị trí còn trống cần một tính từ.
Question 2: He needs to do the homework before going out with his
friends, ____?
A. doesn’t he B. does he C. needn’t he D. need he
Dịch nghĩa: “Anh ấy cần làm bài tập về nhà trước khi đi chơi với bạn,
phải không?”
Mệnh đề giới thiệu khẳng định, phần hỏi đuôi phủ định. Ở hiện tại
đơn với động từ thường mượn trợ động từ do hoặc does tùy theo
chủ ngữ. Câu bắt đầu với He needs thì thành lập hỏi đuôi sẽ dùng
doesn’t he.
Question 3: We thought the repairs on the car would cost about
$500, but our estimate was way off the
it was $4000.
A. mark B. point C. spot D. trace
Kiến thức về thành ngữ
Off the mark: không chính xác, sai, ngoài dự đoán, vượt quá dự
tính
Tạm dịch: Chúng tôi nghĩ rằng việc sửa chữa xe sẽ tốn khoảng 500
đô la, nhưng dự tính của chúng tôi đã
sai- nó là $ 4000.
Question 4. Vietnam has played_________high spirits and had an
impressive 2-0 victory over Yemen.
A.At B.in C. on D. with
Hướng dẫn:
Trước vị trí cần điền là động từ has played (chơi), sau vị trí cần điền là
cụm danh từ high spirits (tinh thần quyết liệt) nên ta cần một giới từ
để diễn tả việc thực hiện hành động gì với thái độ như thế nào.
Question 5. All the applicants for the post are thoroughly ________
for their suitability.
A. searched B. vetted C. investigated D.
scrutinized
Kiến thức về từ vựng
A. search /sɜːrtʃ/ (v): tìm kiếm, lục soát
B. vet /vet/ (v): xem xét kĩ lưỡng (lý lịch, chuyên môn.. của ai)
C. investigate /ɪnˈvestɪɡeɪt/ (v): điều tra
D. scrutinize /ˈskruːtənaɪz/ (v): nhìn chăm chú, nghiên cứu cẩn thận
Tạm dịch: Tất cả các ứng viên cho vị trí đó được xem xét sự phù hợp
một cách kĩ lưỡng
Question 6. Solar energy is not widely used________it is friendly to
the environment.
A.Since B.although C. in spite of D. because of
Hướng dẫn:
Dựa vào nghĩa của câu, ta có vế thứ nhất “Solar energy is not widely
used” (Năng lượng mặt trời không được sử dụng rộng rãi) và vế thứ
hai “it is friendly to the environment” (nó thân thiện với môi trường)
mang nghĩa tương phản nhau.
Dịch nghĩa của các phương án, ta có phương án A và D cùng có nghĩa
là “bởi vì” chỉ nguyên nhân nên không phù hợp để điền vào chỗ
trống. Phương án B và C cùng có nghĩa là “mặc dù” chỉ sự tương
phản. Tuy nhiên, xét về cấu trúc thì “although” đi với mệnh đề còn
“in spite of’ đi với danh từ hoặc cụm danh từ.
Question 7 : Peter’s wife gave him a (n) _____ bike as a birthday
present last week.
A. blue Japanese cheap B. cheap Japanese blue
C. Japanese cheap blue D. cheap blue Japanese
*Theo quy tắc trật tự tính từ trong câu: OSASCOMP
: cheap - Opinion; blue - C; Japanese - Origin
Tạm dịch: Vợ của Peter tặng anh ấy chiếc xe đạp Nhật màu xanh rẻ
tiền như một món quà sinh nhật tuần trước.
Question 8: You shouldn’t lose heart; successoften comes to those
who are not _________ by failures.
A. left out B. put off C. switched off D.
turned on
Kiến thức về cụm động từ
Xét các đáp án:
A. leave out sb/sth (phr.v): để sót, thiếu ai/cái gì
B. put sb off (phr.v): làm ai nhụt chí, thất vọng; ngăn cản ai đó làm
điều gì
C. switch off (phr.v): ngừng dành sự chú ý của bản thân với ai/cái gì
D. turn on sb (phr.v): tấn công, chỉ trích ai đó bất ngờ
Tạm dịch: Bạn không nên bỏ cuộc, thành công thường chỉ đến với
những người không nhụt chí bởi những thất bại.
Cấu trúc khác cần lưu ý:
Lose heart (coll): mất niềm tin vào sự thành công của bản thân, bỏ
cuộc
Question 9: The students_______ the topic when the bell rang.
A. discuss B. were discussing C. have discussed D.
are discussing
Giải thích:
+ Sự việc đang diễn ra chia ở thì quá khứ tiếp diễn: S + was/were +
V.ing => was watching
Tạm dịch: Học sinh đang thảo luận chủ đề này thì chuông reo.
Question 10: _______, he will go out with his friends.
A. When Tony finished his project
B. When Tony finishes his project
C. When Tony had finished his project
D. When Tony was finishing his project
Kiến thức: Mệnh đề chỉ thời gian / Sự hòa hợp thì trong mệnh đề
Giải thích:
Mệnh đề chính chia thì tương lai => động từ trong mệnh đề chỉ thời
gian chia thì hiện tại (hiện tại đơn, hiện tại hoàn thành).
Loại luôn được các phương án A, C, D do sai thì.
Tạm dịch: khi Tony hoàn thành dự án, anh ấy sẽ đi chơi cùng bạn
bè.
Question 11: My mother doesn't eye to eye with my father
sometimes.
A. see B. glance C. look D. agree
Kiến thức về cụm từ cố định
See eye to eye with some one: đồng quan điểm với ai.
Tạm dịch: Mẹ tôi thỉnh thoảng không đồng tình với cha tôi.
Question 12: It is not always easy to_______ a decision at the last
minute.
A. do B. make C. buy D. marry
Kiến thức về cụm từ cố định
Make decision: quyết định.
Tạm dịch: Không phải lúc nào việc đưa ra quyết định đúng đắn vào
phút cuối cùng cũng dễ dàng.
Question 13. I demand to know how this vase __________, and no
one is leaving till I find out.
A. got broken B. was breaking C. has broken D. is broke
Kiến thức: Câu bị động.
Giải thích: Ngoài cấu trúc bị động quen thuộc là be + P2 thì
ta còn có cách diễn đạt bị động khác nữa với cấu trúc: Get +
P2.
Loại B và C là 2 phương án viết ở chủ động, loại D do cách
thể hiện “is broke” sai ngữ pháp. Chọn A.
Tạm dịch: Tôi yêu cầu được biết làm sao mà cái lọ hoa này
lại bị vỡ, và sẽ không ai rời đi cho đến khi nào tôi tìm ra.
Question 14. ________ hard all day, I was exhausted.
A. To work B. Working C. Having worked D. worked
Kiến thức: Mệnh đề phân tử / Rút gọn mệnh đề đồng ngữ
Giải thích:
Khi 2 mệnh đề có cùng chủ ngữ (I ) thì có thể rút gọn 1 trong 2
mệnh đề về dạng:
- V-ing / Having P2: nếu mệnh đề được rút gọn mang nghĩa chủ
động
- P2 (quá khứ phân từ): nếu mệnh đề được rút gọn mang nghĩa bị
động
Chủ ngữ “I ” có thể làm chủ (tự thực hiện hành động “work => nghĩa
chủ động.
Tạm dịch:Làm việc vất vả cả ngày, tôi đã kiệt sức
Question 15: The older you are, _______________ you may
become.
A. the more worrrying B. the more worried
C. the more worry D. worried
Dịch nghĩa: Càng lớn tuổi, bạn càng lo lắng.
Giải thích:
*Cấu trúc so sánh kép:
“The + more adj(dài)/adj_er(ngắn) + S + V, the + more
adj(dài)/adj_er(ngắn) + S + V” (càng……..càng….)
Question 16. Cathy and Graham are at a restaurant.
Cathy: “Oh, $400. I will pay this bill.”
Graham: “_______”
A. Do you have any cash? B. I pay by credit, please
C. No, it’s on me D. We’d better leave off
Giải thích:
Kiến thức về giao tiếp
Cathy và Graham đang ở một nhà hàng.
Cathy: “Ồ, 400 đô la. Tôi sẽ thanh toán hóa đơn này ”.
Graham: “_____________”
A. Bạn có tiền mặt không?
B. Tôi thanh toán bằng tín dụng
C. Không, cứ để tôi
D. Tốt hơn chúng ta nên rời đi
→ Chọn đáp án C
Question 17. Mark and Tim are talking about British literature.
Mark: “I immersed myself in the book on British literature we had
borrowed from the library.”
Tim: “_______”
A. Shall we get started now? B. Is it called “The Pickwick
Papers”?
C. Really? I assumed you’re quite good at it D. I read a book for ages
Giải thích:
Kiến thức về giao tiếp : Mark và Tim đang nói về văn học Anh.
Mark: “Tôi đắm chìm trong cuốn sách về văn học Anh mà chúng ta đã
mượn từ thư viện.”
Tim: “_____________”
A. Chúng ta sẽ bắt đầu ngay bây giờ chứ?
B. Nó có tên là “The Pickwick Papers” đúng không?
C. Thật không? Tôi cho rằng bạn khá giỏi về nó
D. Tôi đọc một cuốn sách lâu lắm rồi
Question 18. A. scholar B. honor C. motto D. fortune
Question 19. A. borrowed B. conserved
C. approached D. complained
Question 18
Giải thích:
Kiến thức về phát âm
A. scholar /ˈskɒlə(r)/ B. honor /ˈɒnə(r)/ C. motto /ˈmɒtəʊ/ D.
fortune /ˈfɔːtʃuːn/
Đáp án D, âm o đọc là /ɔː/, còn lại đọc là /ɒ/
Question 19. Đáp án C đúng vì phần gạch chân của đáp án C đọc là
âm /t/ còn phần gạch chân của các đáp án còn lại được đọc là âm
/d/
A. borrowed /ˈbɒrəʊd/ (v): mượn, vay
B. conserved /kənˈsɜːvd/ (v): bảo tồn, giữ gìn
C. approached /əˈprəʊtʃt/ (v): tiếp cận, đến gần
D. complained /kəmˈpleɪnd/ (v): phàn nàn, than phiền
Question 20.
A. continue B. recognise C. uncover D. remember
Question 21. A. leaflet B. model C. export D. cartoon
Question 20:
Giải thích:
Kiến thức về trọng âm
A. continue /kənˈtɪn.juː/ B. recognise /ˈrek.əɡ.naɪz/
C. uncover /ʌnˈkʌv.ɚ/ D. remember /rɪˈmem.bɚ/
→ Đáp án B trọng âm rơi vào âm tiết thứ nhất, các đáp án còn lại
trọng âm rơi vào âm tiết thứ hai
Q21Giải thích:
Kiến thức về trọng âm
A. leaflet /ˈliː.flət/ B. model /ˈmɑː.dəl/
C. export /ˈek.spɔːrt/ D. cartoon /kɑːrˈtuːn/
→ Đáp án D trọng âm rơi vào âm tiết thứ hai, các đáp án còn lại trọng
âm rơi vào âm tiết thứ nhất
Question 22. She is a down-to-earth woman with no pretensions.
A. ambitious B. creative C. idealistic D. practical
Question 23. For environment safety, we need to find ways to reduce
emission of fumes and smoke of factory.
A. leak B. release C. poison D. pollutant
22.
Kiến thức về từ đồng nghĩa : Down-to-earth: thực tế
A. ambitious (a): tham vọng B. creative (a): sáng tạo
C. idealistic (a): duy tâm D. practical (a): thực tế
down-to-earth = practical
Dịch: Cô ấy là một người phụ nữ bình thường và không có ảo tưởng.
23.
Kiến thức về từ đồng nghĩa : Emission (n): khí thải, sự bốc ra, toả ra
A. leak (n): lỗ rò, khe hở B. release (n): sự tha, xả
C. poison (n): chất độc D. pollutant (n): chất ô nhiễm
→ emission = release
Dịch: Để đảm bảo an toàn môi trường, chúng ta cần tìm cách giảm
Question 24. Poverty in many African countries increases the likelihood
that people poach animals to earn their living.
A. chance B. prospect C. possibility D. improbability
Kiến thức về từ trái nghĩa
Likelihood (n): khả năng, có khả năng
A. chance (n): cơ hội B. prospect (n): triển vọng
C. possibility (n): khả năng D. improbability (n): sự
không có khả năng xảy ra
→ likelihood >< improbability
Dịch: Nghèo đói ở nhiều nước châu Phi làm tăng khả năng người dân
săn trộm động vật để kiếm sống.
Question 25. I don't believe in anything he says, he is unreliable.
A. inaccurate B. unstable C. trustworthyD. irresponsible
Kiến thức về từ trái nghĩa
Unreliable (a): không thể tin tưởng được
A. inaccurate (a): không chính xác B. unstable (a): không ổn định
C. trustworthy(a): đáng tin D. irresponsible (a): không có
trách nhiệm
→ unreliable >< trustworthy
Dịch: Tôi không tin vào bất cứ điều gì anh ấy nói, anh ấy không đáng tin
cậy.
Question 26: He helped us a lot with the project. We couldn’t continue
without him.
A.Provided his contribution wouldn’t come, we couldn’t continue with the
project.
B.But for his contribution, we could have continued with the project.
C.Unless we had his contribution, we could continue with the project.
D.If he hadn’t contributed positively, we couldn’t have continued with the
project.
Anh ấy đã giúp đỡ chúng ta rất nhiều trong dự án này. Chúng ta đã không thể
tiếp tục nếu không có anh
ấy.”
Đây là sự việc trong quá khứ nên ta phải dùng câu điều kiện loại 3 để diễn tả
sự việc trái trái thực tế trong quá khứ.
Provided: miễn là ; But for + N: nếu không có ; -Unless : If ..not: nếu... không
Miễn là sự đóng góp của anh ấy sẽ không đến thì chúng tôi không thể tiếp
tục dự án. (sai cấu trúc)
Nếu không có sự đóng góp của anh ấy thì chúng tôi đã có thể tiếp tục dự
án. (sai nghĩa)
Nếu chúng tôi không có sự đóng góp của anh ấy thì chúng tôi có thể tiếp
tục dự án. (sai nghĩa và cấutrúc)
Những cách khác để diễn đạt câu điều kiện:
a. Unless = If....not (Trừ phi, nếu...không)
E.g: If you don’t study hard, you can’t pass the exam. = Unless you
study hard, you can’t pass the exam.
b. Suppose / Supposing (giả sử như), in case (trong trường hợp),
even if (ngay cả khi, cho dù), as long as, so long as, provided (that),
on condition (that) (miễn là, với điều kiện là) có thể thay cho if trong
câu điều kiện
E.g: Supposing (that) you are wrong, what will you do then?
c. Without/ But for: nếu không có
E.g: Without water, life wouldn’t exist. = If there were no water, life
wouldn’t exist.
Nếu anh ấy đã không đóng góp tích cực thì chúng tôi không thể tiếp
tục dự án
Question27: The storm was so great. Many families had to be evacuated to safer
parts of the city.
A.Although the storm was not great, many families had to be evacuated to safer
parts of the city.
B.So great was the storm that many families had to be evacuated to safer parts of
the city.
C.Many families had to be evacuated to safer parts of the city in spite of the great
storm.
D.It was so a great storm that many families had to be evacuated to safer parts of
the city.
“Cơn bão quá mạnh. Nhiều gia đình phải được sơ tán đến những nơi an toàn
trong thành phố.”
A.Mặc dù cơn bão không mạnh nhưng nhiều gia đình phải được sơ tán đến những
nơi an toàn trong thành phố. (sai nghĩa)
B.Cơn bão quá mạnh nên nhiều gia đình phải được sơ tán đến những nơi an toàn
trong thành phố.
C.Nhiều gia đình phải được sơ tán đến những nơi an toàn trong thành phố mặc dù
cơn bão mạnh, (sai
nghĩa)
D.Sai cấu trúc (so a great storm => such a great storm/ so great a storm)
Question 28: We have conducted (A) exhausting research (B)into the
effects of smartphones on students’ (C) behaviour and their (D)
academic performance.
Giải thích
exhaustive (adj): including everything possible; very thorough or
complete: toàn diện hết mọi khía cạnh
+ exhaustive research: nghiên cứu toàn diện
exhausting (adj): làm kiệt sức, mệt nhoài
academic performance (n, p): thành tích học tập
Do đó: exhausting => exhaustive
“Chúng tôi đã tiến hành nghiên cứu toàn diện ảnh hưởng của điện
thoại thông minh vào cách cư xử và thành tích học tập của học sinh.”
Question 29: (A) On the table (B) is (C) hundreds of books written
(D) in English.
Question 29 Chọn đáp án B
Cấu trúc: - Adv of place + V + S (đảo ngữ trạng từ nơi chốn lên trước
để nhấn mạnh)
Hundreds of + N-plural + V -plural
E.g: Hundreds of people are standing in front of the cinema.
Do đó: is => are
Question 30: Tim works (A) as a doctor and he (B) earns (C) twice (D)as much
as her brother.
Cấu trúc: S + V + multiple number (half/ twice/ three times/....) + as +
much/many/adj/adv + (N) + as
+ N/pronoun (so sánh bội so)
g: The yellow skirt costs twice as much as the red one. (Cái váy mầu vàng có
giá gấp đôi cái váy màu
đỏ.)
“Tim là một bác sĩ và cậu ấy kiếm được gấp đôi anh trai cậu ấy.”
Do đó sai :”her”-his
Question 31. “You should have finished your work by noon, Fiona” said
Mark.
A. Mark warned Fiona against having finished her work by noon.
B. Mark encouraged Fiona to finish her work by noon.
C. Mark reproached Fiona for not having finished her work by noon.
D. Mark scolded Fiona for having finished her work by noon.
Giải thích:
Câu gốc: Mark nói: “Bạn đã nên hoàn thành công việc của mình vào buổi
trưa, Fiona."
→ cấu trúc "should have done: nên làm gì nhưng đã không làm trong quá
khứ"
A. Mark cảnh báo Fiona không nên hoàn thành công việc của cô ấy vào buổi
trưa. → sai nghĩa
B. Mark khuyến khích Fiona hoàn thành công việc của mình vào buổi trưa. →
sai nghĩa
C. Mark trách móc Fiona vì đã không hoàn thành công việc của cô ấy vào
buổi trưa.
D. Mark mắng Fiona vì cô ấy đã hoàn thành công việc vào buổi trưa. → sai
nghĩa
Question 32. It is necessary for the local authority to impose social
distancing on this infected area.
A. The local authority may impose social distancing on this infected area.
B. Social distancing needs imposing on this infected area.
C. Social distancing should be imposed on that infected area.
D. The local authority can impose social distancing on this infected area.
Giải thích:
Tạm dịch: Chính quyền địa phương cần áp dụng giãn cách xã hội với khu
vực bị nhiễm bệnh này.
A. Chính quyền địa phương có thể áp đặt sự giãn cách xã hội đối với khu
vực bị nhiễm bệnh này.
→ Sai ở từ “may” (có thể) # necessary (cần thiết) được đề cập trong bài
B. Giãn cách xã hội cần được áp dụng ra đối với khu vực bị nhiễm này.
C. Cần áp đặt giãn cách xa xã hội đối với khu vực bị nhiễm bệnh đó. → sai ở
từ “that”
(That → this)
D. Chính quyền địa phương có thể áp dụng giãn cách xã hội đối với khu vực
bị nhiễm bệnh này. → khác nghĩa với câu gốc.
Need doing sth: cần được làm gì
Question 33. I haven’t bought any books for 2 years.
A. This is the first time I have ever bought books for 2 years.
B. I last bought books for 2 years.
C. It’s 2 years since I started to buy books.
D. The last time I bought books was 2 years ago.
Giải thích:
Tạm dịch: Tôi đã không mua bất kỳ cuốn sách nào trong 2 năm.
~ D. Lần cuối cùng tôi mua sách là 2 năm trước.
Một số cấu trúc đồng nghĩa:
S + last + Ved + O + khoảng thời gian + ago
= It’s + khoảng thời gian + since + S + last + Ved
= S + haven’t/hasn’t + Ved/PII + O + for + khoảng thời gian
= The last time + S + V(ed) + was + khoảng thời gian +ago
Question 34.
Take exercise: we used to think that the longer we spent on, say, a
pleasurable walk in the countryside, the more good it did us. Not anymore.
The new (34) _______ is for HIIT – short for High-Intensity Interval Training –
(35) _______ just twelve minutes of very intense activity is supposed to be
every bit as beneficial as conventional exercise.
A. potential B. leisure C. habit D. fad
Giải thích:
Kiến thức về từ vựng
A. potential (n): tiềm năng
B. leisure (n): nhàn hạ
C. habit (n): thói quen
D. fad (n): mốt, sự thịnh hành
Căn cứ vào nghĩa ta chọn D
Tạm dịch: Lấy ví dụ như tập thể dục: chúng ta từng nghĩ rằng chúng ta càng
dành nhiều thời gian cho việc đi bộ chậm rãi ở 1 vùng nông thôn, thì điều đó
càng có lợi cho chúng ta. Tuy nhiên bây giờ không còn như vậy nữa. Thứ trào
lưu thịnh hành là HIIT - viết tắt của Luyện tập ngắt quãng cường độ cao
Question 35.
The new (34) _______ is for HIIT – short for High-Intensity Interval
Training – (35) _______ just twelve minutes of very intense activity is
supposed to be every bit as beneficial as conventional exercise.
A. which B. whereby C. whereas D. whom
Giải thích:
Kiến thức về liên từ
A. which: cái mà → không thay thế cho danh từ chỉ vật nào nên sai
B. whereby: theo đó, nhờ đó
C. whereas: trong khi → không phù hợp về nghĩa
D. whom: người mà → không thay thế cho danh từ chỉ người nào nên
sai
Tạm dịch: Thứ trào lưu thịnh hành là là HIIT - viết tắt của Luyện tập
ngắt quãng cường độ cao - theo đó chỉ cần 12 phút hoạt động cường
độ cao được cho là có lợi như bài tập thông thường.
Question 36.
They also maintain that it speeds up metabolism and so makes you
burn more calories throughout the day. However, some recent
research would appear to (36) ______ these claims.
A. engender B. subscribe C. dispute D. cease
Giải thích:
Kiến thức về từ vựng
A. engender (v): gây ra
B. subscribe (v): đồng tình, đăng ký
C. dispute (v): tranh cãi, tranh luận
D. cease (v): chấm dứt
Tạm dịch: Họ cũng xác nhận rằng nó tăng tốc độ trao đổi chất và
do đó khiến bạn đốt cháy nhiều calo hơn trong suốt cả ngày. Tuy
nhiên, một số nghiên cứu gần đây dường như không không tình với
những tuyên bố này.
Question 37.
(37) _______ have our personalities changed, too? Smartphones
allow us to access information in no time at all. Research
demonstrates that 80% of people will not wait more than 30 seconds
for a video to load.
A. So B. However C. But D. And
Giải thích:
Kiến thức về liên từ
A. So: vì vậy → không phù hợp về nghĩa
B. However: tuy nhiên → sau “however” có dấu phẩy
C. But: nhưng
D. And: và → không phù hợp về nghĩa
Tạm dịch: Nhưng tính cách của chúng ta có thay đổi không? Điện
thoại thông minh cho phép chúng ta truy cập thông tin nhanh chóng.
Nghiên cứu chứng minh rằng 80% mọi người sẽ không đợi quá 30
giây để tải video.
Question 38.
Overtime, we come to expect (38) _ to be available infinitely more
quickly than in the past. We have far less patience. We’ve forgotten
how to slow down. Welcome to modern life.
A. each B. everything C. few D. almost
Giải thích:
Kiến thức về từ vựng
A. each: mỗi
B. everything: mọi thứ
C. few: một vài
D. almost: gần như
Tạm dịch: Qua thời gian, chúng ta mong đợi mọi thứ sẽ khả dụng
nhanh hơn vô hạn so với trước đây. Chúng ta có ít kiên nhẫn hơn.
Chúng ta đã quên cách sống chậm lại rồi. Chào mừng bạn đến với cuộc
sống hiện đại.
Question 39. Which would be the best title for the passage?
A. Singapore fixes you up B. The dating life of single Singaporeans
C. Singapore’s dating agency D. How to get a partner in Singapore
Giải thích:
Đâu sẽ là tiêu đề hay nhất cho đoạn văn?
A. Singapore làm mai cho bạn
B. Cuộc sống hẹn hò của những người Singapore độc thân
C. Đại lý hẹn hò của Singapore
D. Làm thế nào để có được một người yêu ở Singapore
Đoạn văn chủ yếu nói về tình trạng yêu đương kết hôn của những
người ở Singapore và việc chính phủ Singapore đang thực hiện một
chương trình để giúp mọi người có thể hẹn hò và cưới đúng người
Question 40. The phrase “settle down” in paragraph 1 mostly
means _______.
A. leave a job B. find a job
C. get married D. keep calm
Giải thích:
Cụm từ "settle down" trong đoạn 1 chủ yếu có nghĩa là _________.
A. nghỉ việc
B. tìm việc làm
C. kết hôn
D. giữ bình tĩnh
→ settle down: Bắt đầu cuộc sống ổn định, kết hôn = get married
Question41. According to the passage, the main reason why delaying parenthood
becomes a norm in developed countries is that _______.
A. single individuals focus more on their jobs than on seeking a partner
B. they have little time to meet possible partners due to their jobs
C. people who have reached the age of 30 are used to being alone
D. individuals are not prepared to have a major responsibility
Giải thích:
Theo đoạn văn, lý do chính khiến việc trì hoãn việc làm cha mẹ trở thành một thứ quá
đỗi bình thường ở các nước phát triển là do _________
A. các cá nhân độc thân tập trung nhiều hơn vào công việc của họ hơn là tìm kiếm bạn
đời
B. họ có ít thời gian để gặp gỡ người bạn đời có thể do công việc của họ
C. những người đã bước qua tuổi 30 đã quen với việc cô đơn
D. các cá nhân không được chuẩn bị để chịu trách nhiệm chính
Căn cứ vào thông tin:
Putting off marriage and parenthood is becoming more common in all industrialized
nations. This is partly because high pressure jobs leave little time for socializing and
meeting potential partners.
(Trì hoãn hôn nhân và làm cha mẹ đang trở nên phổ biến hơn ở tất cả các quốc gia
công nghiệp hóa. Điều này một phần là do công việc áp lực cao khiến bạn có ít thời
gian để giao lưu và gặp gỡ những người có tiềm năng trở thành bạn đời của bạn.)
Question42. The word “it” in paragraph 4 refers to _______.
A. government B. agency C. Social Development Unit D. graduate
Giải thích:
Từ "it" trong đoạn 4 đề cập đến _________
A. chính quyền
B. cơ quan
C. Đơn vị phát triển xã hội
D. tốt nghiệp
Căn cứ vào ngữ nghĩa của câu:
Research scientists Kee-Chuan Goh (29) has never had a girlfriend. “There’s a first
time for everything.”, he says. Like Madeline he has signed up with the SDU (Social
Development Unit), the government’s very own dating agency. It is open only to
graduates, who sign up for five years, although mostmembers are married within
three.
(Nhà khoa học nghiên cứu Kee-Chuan Goh (29 tuổi) chưa từng có bạn gái. “Đó là
lần đầu tiên của mọi thứ.”, Anh ấy nói. Giống như Madeline, anh ấy đã đăng ký với
SDU (Đơn vị phát triển xã hội), cơ quan hẹn hò của chính phủ. Nó chỉ mở cửa cho
sinh viên tốt nghiệp đăng ký trong năm năm, mặc dù hầu hết các thành viên đã kết
hôn trong vòng ba năm.)
Question 43. Which of the following is true, according to the passage?
Giải thích:
Điều nào sau đây là đúng, theo đoạn văn?
A. Áp lực kết hôn lớn nhất đến từ cha mẹ của Madeline
B. Những người phụ nữ tốt nghiệp đại học trở thành những người vợ được săn đón
C. Đàn ông và phụ nữ khó hòa hợp một cách tự nhiên
D. Quảng cáo khuyến khích những người trẻ tuổi có những kỳ vọng thực tế hơn
Căn cứ vào thông tin: Singapore’s government believes that, to ensure continuousprosperity,
future generations must become more intelligent. And it is concerned that female graduates –
ideal for breeding this super-race – are staying single and childless. Singapore’smen, on the
other hand, are marrying less educated women. So, the government has launcheda campaign
to encourage the ‘right’ couples to get together.
A sai vì trong bài chỉ nói “And it is not only her family but also her government who are keen
for her to marry and reproduce.” – Không chỉ cha mẹ mà cả chin phủ cũng muốn cô ấy kết hôn
và sinh con
C sai vì trong bài chỉ đề cập “The sexes are not encouraged to mix during childhoodor to date
until after university.” – Nam và nữ được khuyến khích là không nên ở cùng nhau trong suốt
thời thơ ấu hay hẹn họ cho tới khi tốt nghiệp.”
D sai vì đoạn cuối có nói “Government-sponsored ads on prime-time TV aim to get the
message across. In one, a young man sits on a park bench, sighing, “Where is my dream girl?”
At the other end, a girl stares into space thinking, “Where is the man of my dreams?” They fail
to notice each other and wander off, lonely.A voice-over warns “Why not reality? You could
wait a lifetime for a dream.” - ……Tạisao không phải là thật chứ? Bạn chỉ có thể chờ đợi 1 giấc
mơ cả đời mà thôi” → Quảng cáo tạo cho con người ta cảm giác tìm bạn đời ảo, chứ không hề
chân thực.
Question 44. Which best serves as the title for the passage?
A. The growing disparity between rich and poor
B. Seeing valuable time from a crooked angle
C. The mad rush to save time
D. The slower pace of life: Is it suitable for all?
Giải thích:
Đâu là tiêu đề hay nhất cho đoạn văn?
A. Sự chênh lệch giàu nghèo ngày càng tăng
B. Chiêm nghiệm về thời gian quý báu theo cách nhìn hoàn toàn mới
(from crooked angle ~ from a completely different perspective)
C. Sự vội vã điên cuồng để tiết kiệm thời gian
D. Nhịp sống chậm hơn: Liệu có phù hợp cho tất cả
Bài viết nói về thời gian và việc con người ngày càng vội vã để tiết
kiệm thời gian nên đáp án C là hợp lý
Question 45. The word “malaise” in paragraph 1 is closest in
meaning to _______.
A. uneasinessB. tactic C. morale D. trend
Giải thích:
Từ “malaise” trong đoạn 1 gần nghĩa nhất với ________
A. bất an
B. chiến thuật
C. tinh thần
D. xu hướng
→ malaise: bất ổn = uneasiness: bất an
Question46. The consequences of not having enough time for family are mentioned
in the passage EXCEPT _______.
A. Working couples find themselves in a situation of lacking time for each other
B. Children find it impossible to pay a visit to their grandparents
C. Time to spend with children becomes scarce
D. Parents have quarrels over the amount of time allotted for doing household
chores
Giải thích:
Hậu quả của việc không có đủ thời gian cho gia đình được đề cập trong đoạn văn
NGOẠI TRỪ __________
A. Các cặp vợ chồng đang đi làm thấy mình trong tình trạng thiếu thời gian dành cho
nhau
B. Con cái không về thăm ông bà được.
C. Thời gian dành cho con cái trở nên khan hiếm
D. Cha mẹ cãi nhau về lượng thời gian dành cho việc nhà
Căn cứ vào thông tin đoạn 2: The result is parents with a lack of quality time to spend
with their children, and surveys showing that working couples see less of each other
than ever before and that rows over time spent on domesticlabour or childcare top
the list of marital discord.
Question 47. The word “this” in paragraph 3 refers to _______.
A. answering the phone B. calling someone on the phone
C. asking someone to hold the line D. arguing with customers
Giải thích:
Từ “this” trong đoạn 3 đề cập đến ________
A. trả lời điện thoại
B. gọi điện thoại cho ai đó
C. yêu cầu ai đó giữ điện thoại
D. tranh cãi với khách hàng
Căn cứ vào ngữ nghĩa của câu: It is now perfectly acceptable to be
asked to hold the instant the phone is answered. This saves the
company time and money, but costs you time. We are engaged in a
constant, subtle war over time.
(Bây giờ hoàn toàn có thể chấp nhận khi được yêu cầu giữ điện thoại
ngay sau khi điện thoại được trả lời. Điều này giúp tiết kiệm thời gian
và tiền bạc cho công ty, nhưng lại khiến bạn mất thời gian. Chúng ta
đang tham gia vào một cuộc chiến liên tục, tinh vi theo thời gian.)
~ Lưu ý: the instant = as soon as
Question48. The word “treadmill”in paragraph 5 mostly describes something
which is _______.
A. energizing B. conducive C. alien D. monotonous
Giải thích:
Từ “treadmill” trong đoạn 5 chủ yếu mô tả một cái gì đó _________
A. tiếp thêm sinh lực
B. có lợi
C. người ngoài hành tinh
D. đơn điệu, tẻ nhạt
→ treadmill: guồng quay liên tục = cái gì đó rất tẻ nhạt đơn điệu
Căn cứ vào ngữ cảnh trong câu trước nó:
At the same time, they are employing others - cleaners, childminders, fast-food
restaurant workers - in order to allow themselves to work all the time. Meanwhile,
more and more of us are putting ourselves on the treadmill of constant activity,
taking on an increasingly heavy workload, and never stopping for a moment to ask
ourselves why.
(Đồng thời, họ đang thuê những người khác - người dọn dẹp, người trông trẻ, nhân
viên nhà hàng thức ăn nhanh - để cho phép bản thân làm việc mọi lúc. Trong khi đó,
ngày càng có nhiều người trong chúng ta đặt mình vào guồng quay của hoạt động
liên tục, gánh vác khối lượng công việc ngày càng nặng nề, và không bao giờ dừng
lại một chút để tự hỏi bản thân tại sao.)
Question 49. Which of the following is true, according to the
passage?
A. People who are idle often experience a sense of willingness
B. The rich had more time in the past compared to the poor
C. People signing up for time management courses wish to have a
free afternoon
D. Being late is a symbol of status in the past
Giải thích:
Điều nào sau đây là đúng, theo đoạn văn?
A. Những người nhàn rỗi thường có cảm giác hạnh phúc và sẵn sàng
làm mọi thứ
B. Người giàu có nhiều thời gian hơn trong quá khứ so với người
nghèo
C. Những người đăng ký khóa học quản lý thời gian muốn có một
buổi chiều rảnh rỗi
D. Đến muộn là biểu tượng của địa vị trong quá khứ
Căn cứ vào thông tin đoạn 4:
Of course, there is a class dimension to the rush culture. One of the biggest
transitions of the past few decades has been to take the previous relationship
between time and status - the rich had lots of time, the poor very little - and
reverse it.
(Tất nhiên, có một khía cạnh giai cấp đối với văn hóa của sự vội vàng. Một trong
những bước chuyển đổi lớn nhất trong vài thập kỷ qua là lấy mối quan hệ trước
đây giữa thời gian và địa vị - người giàu có rất nhiều thời gian, người nghèo rất ít
- và đảo ngược nó.)
A sai vì “The idea of doing nothing has become terrifying, a sure sign of
worthlessness.” – Cái việc ăn không ngồi rồi trở nên vô cùng đáng sợ, 1 dấu hiệu
của sự vô dụng” → Lưu ý phép paragraphase: idle (rảnh rỗi ) ~ do nothing
C sai vì “We yearn for the lazy afternoons and days of yesteryear - but
enthusiastically sign up for email, messaging services, language classes. Even time
managementcourses.” – “Chúng tôi mong muốn có được những buổi chiều thảnh
thơi, nhưng vẫn cứ hừng hực đăng ký email, dịch vụ tin nhắn, các lớp học ngôn
ngữ, hay thậm chí là cả những khóa học quản lý thời gian nữa”
D sai vì “Moreover, to be seen to have time to spare is a sign of low status:
arranging lunch, it is never done to be available too soon. Similarly, being late is
moving from a sign of rudeness to a sign of status.” – “……. Tương tự như vậy, đến
muộn từ dần chuyển từ trạng thái bất lịch sự sang dấu hiệu của người có địa vị
Question 50. Which of the following can be inferred from the
passage?
A. People living in an instant society tend to multitask
B. Advertising led to a larger number of people who opt to give up
their free time
C. There is a strong positive correlation between time and status
D. Manual workers are required to work against the clock
Giải thích:
Điều nào sau đây có thể được suy ra từ đoạn văn?
A. Những người sống trong một xã hội cấp bách có xu hướng làm
nhiều nhiệm vụ
B. Quảng cáo đã dẫn đến một số lượng lớn hơn những người chọn từ
bỏ thời gian rảnh rỗi của họ
C. Có mối tương quan thuận chặt chẽ giữa thời gian và địa vị
D. Người lao động chân tay được yêu cầu làm chạy đua với thời gian
Căn cứ vào các thông tin:
Symptoms include jabbingthe 'door close' button on lift doors to save the two to
four seconds required for the door to do it on its own, and an inabilityto do one
thing at a time, so that every journey is a phone call opportunity.
(Các triệu chứng bao gồm ấn nút 'đóng cửa' trên cửa thang máy để tiết kiệm 2-4 giây
cần thiết để cửa tự làm việc đó và việc không thể làm 1 thứ 1 lúc được ………”
→ Nghĩa là con người ta không thể làm 1 thứ 1 lúc, mà phải làm nhiều thứ 1 lúc để
còn tiết kiệm thời gian → multitask.
B sai vì trong bài chỉ nói “Adverts for energy-boosting drinks read: 'Having trouble
keeping up with yourself?'” – Các quảng cáo đồ uống tăng lực với khẩu hiệu “Không
thể chạy kịp bản thân ư?” → Điều đó chưa thể kết luận quảng cáo khiến con người ta
từ bỏ thời gian rảnh được.
C sai vì trong bài chỉ nói “Similarly,being late is moving from a sign of rudeness to a
sign of status.” - Tương tự như vậy, đến muộn từ dần chuyển từ trạng thái bất lịch sự
sang dấu hiệu của người có địa vị rồi
D sai vì “The rich are working longer and longer hours in order to compete with each
other. At the same time, they are employingothers - cleaners, childminders,fast-
food restaurant workers - in order to allow themselves to work all the time.” –
“Người giàu giờ càng làm nhiều hơn để muốn cạnh tranh với nhau. Cùng lúc đó, họ
thuê những người lau dọn, vú nuôi, nhân viên tại các cửa hàng thức ăn nhanh, để
cho phép bản thân mình làm việc mọi lúc” → Chính người giàu mới làm việc chạy đua
với thời gian, chứ không đề cập là nhân viên lao động chân tay.
→ Chọn đáp án A
Question 1: The children slept soundly because it was so _________
and quiet in the garden.
A. peaceable B. peacefulness C. peaceably D. peaceful
A. peaceable /’pi:səbl/ (a): thích sống hòa bình
B. peacefulness /’pi:sflnis/ (n): sự yên bình
C. peaceably /’pi:səbli/ (adv): một cách yên bình
D. peaceful /’pi:sfl/ (a): yên bình, hòa bình
Căn cứ vào cấu trúc câu (S + to be + Adj…) và cấu trúc song song vì vậy vị
trí ô trống cần điền là một tính từ.
Dịch nghĩa: Những đứa trẻ đã ngủ rất ngon bởi vì ở ngoài vườn thật yên
bình và yên lặng.
Question 2: Donald John Trump is the 45th President of the United
States, _______?
A. is heB. Is not he C. isn’t it D. isn’t he
Dịch nghĩa: “Donald John Trump là Tổng thống thứ 45 của Hoa Kỳ phải
không?”
Câu bắt đầu với S + is thì thành lập hỏi đuôi sẽ dùng isn’t + S.
Question 3: Mr Nam knows Hanoi City like the back of his . He
used to be a taxi driver there for 10 years.
A. head B. mind C. hand D. life
head (n): đầu
mind (n): trí óc, tâm trí
hand (n): tay
life (n): cuộc sống
+ know sb/ sth like the back of one’s hand ~ to be very familiar
with somebody/something:hiểu biết rất rõ, biết rõ như lòng bàn
tay
Dịch: Ông Nam biết thành phổ Hà Nội rõ như lòng bàn tay. Ông ấy
từng lái xe taxi ở đó 10 năm.
Question 4: Bill Gates gave away $4.6 billion in Microsoft shares
______June to his personal charity, the Bill & Melinda Gates
Foundation.
A. on B. for C. in D. at
Dịch nghĩa: Bill Gates đã tặng 4,6 tỷ đô la cổ phiếu Microsoft vào
tháng 6 cho tổ chức từ thiện cá nhân của mình, Quỹ Bill & Melinda
Gates.
Căn cứ vào cấu trúc:
-in+ month.
Question 5: Whenever a camera was pointed at her, Marilyn would
instantly
________herself into a radiant star.
A. transmit B. transform C. convert D. transfer
A. transfer / trænz mit / (v): truyền tài (thông tin, tín hiệu)
B. biến đổi / træns fo rm / (v): chuyển đối đối, biển đổi (hinh dang cái
gi hay ai dó thành một cái / ngurời hoàn toàn khác)
C. convert / ken v3 t / (v): chuyển đối. change (from the position /
this status to vi tri / khác trang hoặc đơn vị đo lường, vị trí tiên
phong) chuyển nhượng lại (tài sản, thu lợi) cho ai
D. transfer / træns fa r / (v ): chuyển đổi (từ nơi này sang nơi khác)
Tạm dịch: Bát cử khi nào máy ảnh chia vào có, Marilyn sẽ lập tức
biển minh thành một ngôi sao rang.
Question 6. ________ she was the most prominent candidate, she
was not chosen.
A. Though B. Because of C. As D. Since
Kiến thức: Liên từ
Giải thích:
A. Though : Mặc dù
B. Because of: ( theo sau cụm danh từ)Vì
C. As: Vì
D. Since: Vì, Từ khi
Dịch : Mặc dù cô ấy là ứng cử viên nổi bật nhất, cô ấy đã không
được chọn.
Question 7. She has just bought a bag.
A. small plastic blue B. blue small plastic
C. plastic small blue D. small blue plastic.
Trật tự của tính từ trong câu này là: small (size) - blue (color) -
plastic (material)
Dịch: Anh ấy vừa mới mua một cái túi nhỏ màu xanh da trời bằng
nhựa
Question 8: Although she is my sister, I find it hard to her
selfishness.
A. put up with B. catch up with C. keep up with D. come
down with
put up with: chịu đựng
E.g: I cannot put up with him any more. catch up with (sb): đuổi kịp
(ai)
E.g: Go on ahead. I’ll catch up with you. keep up with: theo kịp, bắt
kịp
E.g: Technology changes so fast it’s hard to keep up with it. come
down with: mắc bệnh, ốm (nhẹ)
E.g: I came down with flu last week.
Question 9. When he came, I_______ in the sitting room.
A. readB. am reading C. has read D. was reading
Kiến thức: Sự phối hợp về thì của động từ
Giải thích: Thì quá khứ đơn và quá khứ tiếp diễn kết hợp trong câu:
Diễn tả hành động đang xảy ra trong quá khứ thì một hành động khác
xen vào, hành động đang xảy ra chi thì quá khứ tiếp diễn, hành động
xen vào chia thì quá khứ đơn
Cấu trúc: S + was/ were + V_ing + when + S + Ved/bqt
Tạm dịch: Khi anh ấy đến, tôi đang đọc sách ở phòng khách.
Question 10. She will call you to confirm _______.
A. until she received the parcel B. when she was
receiving the parcel
C. the moment she receives the parcel D. after she had
received the parcel
10.Kiến thức: Mệnh đề chỉ thời gian / Sự hòa hợp thì trong mệnh đề
Giải thích:
Mệnh đề chính chia thì tương lai => động từ trong mệnh đề chỉ thời
gian chia thì hiện tại (hiện tại đơn, hiện tại hoàn thành).
Loại luôn được các phương án B, A, D do sai thì.
A. cho đến khi cô ấy nhận được bưu kiện
B. khi cô ấy đang nhận bưu kiện
C. thời điểm cô ấy nhận được bưu kiện
D. sau khi cô ấy đã nhận được bưu kiện
Question 11: I get quite depressed when I think about the damage
we are to the environment.
A. having B. taking C. making D. causing
(to) cause the damage: gây thiệt hại.
Dịch: Tôi rất thất vọng mỗi khi nghĩ về những thiệt hại mà chúng ta
đã gây ra cho môi trường
Câu 12: : I'm not going to go ice skating! I'd only fall over
and...........a fool of myself.
A. create B. show C.do D. make
Kiến thức về cụm từ cố định
make a fool (out) of sb/yourself: khiến ai đó trông như kẻ ngốc
Tạm dịch: Tôi sẽ không đi trượt băng đâu! Tôi chỉ bị ngã và khiến
mình trông như kẻ ngốc.
Kiến thức về cụm từ cố định
cause the damage: gây thiệt hại.
Question 13. A new supermarket ________ in the city cente now.
A. is being built B. being built C. are being built D.
be built
Câu bị động của hiện tại tiếp diễn S+ is/am/are+ being+Vpp
Question 14: _____ a scholarship, she entered one of the most
privileged universities of the United Kingdom.
A. To award B. Being awarded
C. Having awarded D. Having been awarded
Kiến thức về rút gọn câu
Tạm dịch: Được nhận học bổng, cô ấy vào học tại một trong những
trường đại học đặc quyền nhất của Vương quốc Anh.
+ Hai vế có cùng chủ ngữ là “she”, ta có thể bỏ chủ ngữ vế đầu,
đồng thời chuyển động từ về dạng Ving khi nó mang nghĩa chủ động
và Vp2/Ved khi nó mang nghĩa bị động.
+ Vì hành động “được nhận học bổng” xảy ra trước hành động còn
lại trong câu nên ta dùng “having been Vp2/Ved”.
Question 15: The larger the flat is, the ____________ its price is.
A. expensive B. more expensive C. expensively D. most
expensive
Dịch nghĩa: Căn hộ càng lớn thì giá của nó càng đắt.
Xét các đáp án:
A. expensive → Đây là cấu trúc so sánh kép nên chỗ trống cần điền từ
có dạng so sánh hơn
B. more expensive → Cấu trúc so sánh kép khi nói về 2 người hoặc sự
vật: The more/-er + S + V, the more/-er + S + V
C. expensively → Đây là cấu trúc so sánh kép nên chỗ trống cần điền từ
có dạng so sánh hơn
D. most expensive → Đây là cấu trúc so sánh kép nên chỗ trống cần
điền từ có dạng so sánh hơn
Question 16: Marry is talking to Linda over the phone. Mary: “Thank
you for helping me prepare for the party.” Linda: “ ”.
A. My pleasure B. The meal was out of this world
C. Never mention me.D. Of course not
Mary đang nói chuyện điện thoại với Linda.
Mary: Cảm ơn cậu vì đã giúp tớ chuẩn bị cho bữa tiệc.
Đó là niềm hân hạnh của tớ khi giúp được cậu.
Bữa ăn thì vô cùng tuyệt vời.
Đừng bao giờ nhắc tới tớ.
Tất nhiên là không.
Ta dùng “You’re welcome”, “My pleasure” để đáp lại lời cảm ơn.
Question 17: Tom is in Ho Chi Minh City and asks a passer-by the
way to the railway station.
Tom: “Can you show me the way to the railway station, please?”
- Passer-by: “ ”
A. No way. B. Just round the comer over there.
C. Look it up in a dictionary! D. There’s no traffic near here.
Tom đang ở thành phố Hồ Chí Minh và hỏi người qua đường
đường tới nhà ga xe lửa.
Không đời nào.
Nó chỉ ở quanh góc kia thôi.
Hãy tra cứu nó trong cuốn từ điển!
Không có giao thông ở gần đây.
Question 18. A. legal B. diverse C. polite D. complete
Question 19. A. suspicious B. marvelous C. physical D. argument
Question 18
Kiến thức: Trọng âm từ có 2 âm tiết
Giải thích:
legal /ˈliːɡl/ diverse /daɪˈvɜːs/ polite /pəˈlaɪt/complete
/kəmˈpliːt/
Câu A trọng âm 1 còn lại trọng âm 2.
Question 19.
Kiến thức: Trọng âm của từ có 3 âm tiết trở lên
Giải thích:
suspicious /səˈspɪʃəs/ marvelous /ˈmɑːvələs/ physical /ˈfɪzɪkl/
argument /ˈɑːɡjumənt/
Câu A trọng âm rơi vào âm tiết thứ 2, còn lại là thứ nhất
Question 20. A. Lived B. cooked C. laughed D. watched
Kiến thức: Phát âm “-ed”
Giải thích:
Những từ kết thúc bằng các âm: /ʧ/, /s/, /k/, /f/, /p/, /θ/, /∫/ thì
„ed‟ sẽ được đọc là /t/.
Những từ kết thúc bằng âm /t/ hay /d/ thì „ed‟ sẽ được đọc là /ɪd/.
Các trường hợp còn lại, „ed‟ sẽ đọc là /d/.
Lived /livd/ cooked /kukt/ laughed /lɑ:ft/watched /wɔtʃ/
Đáp án A có phần gạch chân đọc là /d/, các đáp án còn lại đọc là /t/
Question 21. A. Educate B. engineer C. acceptable D. department
Kiến thức: Cách phát âm “e”
Giải thích:
educate /ˈedʒukeɪt/ engineer /ˌendʒɪˈnɪə(r)/ acceptable
/əkˈseptəbl/ department /dɪˈpɑːtmənt/ Âm “e” trong từ
“department” phát âm là /ɪ/, trong các từ còn lại phát âm là /e/
Question 22: That restaurant tempts many customers because of its
affordable price and good food.
A. attracts B. frees C. refuses D. offers
attract (v): thu hút, hấp dẫn, lôi cuốn
free (v): thả, giải thoát, phóng thích .
refuse (V): từ chối
offer (v): cung cấp
tempt (v): cám dỗ, lôi cuốn, quyến rũ
Question 23: I’ll back up your ideas if they don’t believe you.
A. support B. rebel C. admire D. protect
support (v): ủng hộ
admire (v): ngưỡng mộ
protect (v): bảo vệ
back up (ph.v): ủng hộ
Dịch: Tôi sẽ ủng hộ ý kiến của bạn nếu họ không tin bạn.
Question 24: She had never seen such discourtesy towards the
director as it happened in the meeting last week.
A. politeness B. rudeness C. measurement D.
encouragement
politeness (n): sự lịch sự
rudeness (n): sự thô lỗ, bất lịch sự, sự khiếm nhã
measurement (n): sự đo lường, phép đo
encouragement (n): sự khuyến khích, động viên
discourtesy (n) ~ impoliteness, rudeness: sự bất lịch sự, sự thô lỗ
politeness
Dịch: Cô ấy chưa bao giờ nhìn thấy sự bất lịch sự như vậy đối với
giám đốc khi điều này đã xảy ra tại cuộc họp vào tuần trước.
Question 25: They conducteda campaign to promote people’s
awareness of environmental problems.
A. encourage B. publicize C. hinder D. strengthen
encourage (v): khuyến khích, động viên
publicize (v): quảng cáo, đưa ra công khai
hinder (v): cản trở, gây trở ngại
strengthen(v); tăng cường
promote (v) ~ encourage: đẩy mạnh, khuyến khích
Dịch: Họ đã thực hiện một chiến lược để tăng cường ý thức của mọi
người về các vấn đề môi trường. Mark the letter A, B, C, or D on your
answer sheet to indicate the word(s) CLOSEST in meaning to the
underlined word(s) in each of the following questions.
Question26: He got down to writing the letter as soon as he returned from his walk.
A. No sooner had he returned from his walk when he got down to writing the letter.
B. Not until he returned from his walk did he get down to writing the letter.
C. Only after he had returned from his walk did he get down to writing the letter.
D. Hardly had he returned from his walk when he got down to writing the letter.
Dịch nghĩa: Anh ta ngồi xuống viết thư ngay khi đi bộ về.
Xét các đáp án:
A. No sooner had he returned from his walk when he got down to writing the letter. →
Cấu trúc đảo ngữ với các cụm từ so sánh về thời gian: No sooner + had + S + P2 + than
+ S + Ved (nếu dùng no sooner phải có than đằng sau).
B. Not until he returned from his walk did he get down to writing the letter. → Mãi
cho đến khi anh trở về sau khi đi bộ, anh mới xuống viết thư, sai nghĩa.
C. Only after he had returned from his walk did he get down to writing the letter. Chỉ
sau khi anh ấy trở về sau khi đi bộ, anh ấy mới viết thư, sai nghĩa
D. Hardly had he returned from his walk when he got down to writing the letter. →
Cấu trúc đảo ngữ với các cụm từ so sánh về thời gian: Hardly/Scarcely/Barely + had +
S + P2 + when + S + Ved: ngay khi/vừa mới….thì
Question 27 (VDC): I invitedRachel to my party, but she couldn’t come. She had arranged
to do something else.
A. Without having arranged to do something else, Rachel would have come to my party as
invited.
B. If it hadn’t been for her arrangement for something else, Rachel would come to my
party as invited.
C. Rachel would have come to my party, unless she hadn’t arranged to so something else.
D. If Rachel hadn’t arranged to do something else, she would come to my party as invited.
Kiến thức: Câu điều kiện loại 3
Giải chi tiết:
- Cách dùng: Diễn tả một giả thiết trái ngược với thực tế đã xảy ra ở quá khứ
Công thức chung: If + S + had (not) + Ved/ V3, S + would/ could (not) + have + Ved/ V3.
- Trong câu điều kiện loại 3, có thể sử dụng “but for” và “without” “had it not been for”
thay cho “if”
Công thức chung: But for/ Without/ Had it not been for + N/Ving, S + would/ could (not) +
have + Ved/ V3.
Tạm dịch: Tôi đã mời Rachel đến bữa tiệc của tôi, nhưng cô ấy không thể đến. Cô đã sắp
xếp để làm việc khác.
= Nếu sắp xếp để làm việc khác, Rachel sẽ đến bữa tiệc của tôi như được mời rồi.
B. Sai công thức. “would come” => “would have come”
C. Sai công thức. Mệnh đề sau “unless” (nếu … không) không để dạng phủ định
D. Sai công thức. “would come” => “would have come”
Question 28: I don’t think that he has responsed to my email as I have received nothing.
A. don’t think B. received C. as D. responsed
Question 29: In order to make progress in her speaking skill, you need to practice
everyday.
A. in order to B. practice C. make progress D. her speaking skill
Question 30: Fifty minutes are the maximum length of time allotted for the exam.
A. allotted B. are C. length D. for
28.
Kiến thức: Từ vựng
* Response (n): sự hồi âm, sự đáp lại * Respond (v): hồi âm, trả lời.
Sửa: responsed → responded
Tạm dịch: Tôi không nghĩ là anh ấy đã trả lời thư của tôi vì tôi chưa nhận được gì cả.
29.
Kiến thức: Cụm từ • make progress: tiến bộ
Sửa
D. her speaking skill- BỎ HER.
Tạm dịch: Để tiến bộ trong kỹ năng nói, bạn cần luyện tập hàng ngày.
30.
Kiến thức: Sự phù hợp giữa chủ ngữ và động từ
• “Fifty minutes” là một khoảng thời gian, động từ luôn chia số ít
Sửa: are => is
Tạm dịch: Năm mươi phút là khoảng thời gian tối đa dành cho bài thi.
Question 31: It's time for me to start to think about the job I will do
in the future.
A. I should start to think about the job I will do in the future.
B. I may start to think about the job I will do in the future.
C. I needn't start to think about the job I will do in the future.
D. I must have started to think about the job I will do in the future.
Dịch nghĩa: Đã đến lúc tôi bắt đầu nghĩ về công việc tôi sẽ làm trong
tương lai.
A. Tôi nên bắt đầu nghĩ về công việc tôi sẽ làm trong tương lai.
(Đúng)
B. Tôi có thể bắt đầu nghĩ về công việc tôi sẽ làm trong tương lai.
C. Tôi không cần phải bắt đầu nghĩ về công việc tôi sẽ làm trong
tương lai.
D. Tôi hẳn là đã phải bắt đầu nghĩ về công việc tôi sẽ làm trong tương
lai.
=> Vì cấu trúc: It’s time for sb to do st (thiên về nghĩa khuyên bảo ) →
Question 32: “Buy some meat in the shop”, said my mother to
me.
A. My mother said to me to buy some meat in the shop.
B. My mother told to me to buy some meat in the shop.
C. My mother told me to buy some meat in the shop.
D. My mother said me to buy some meat in the shop.
Dịch nghĩa: “Mua một ít thịt trong cửa hàng nhé!”, mẹ nói với tôi.
Xét các đáp án:
A. My mother said to me to buy some meat in the shop.→ Cấu trúc:
say sth to sb: nói với ai điều gì, không có cấu trúc “say to sb to do
sth”
B. My mother told to me to buy some meat in the shop. → Cấu trúc:
tell sb sth: nói với ai điều gì
C. My mother told me to buy some meat in the shop. → Đúng (cấu
trúc tell sb to do st: bảo ai làm gì)
D. My mother said me to buy some meat in the shop.→ Cấu trúc: say
sth to sb: nói với ai điều gì
Question 33: I haven't met my grandparents for five years.
A. I often met my grandparents five years ago.
B. I last met my grandparents five years ago.
C. I have met my grandparents for five years.
A. I didn't meet my grandparents five years ago.
Tạm dịch: “Tôi đã không gặp ông bà tôi trong năm năm.”
A. Tôi thường gặp ông bà tôi năm năm trước. (Sai vì nghĩa khác so với
câu gốc.)
B. Tôi gặp ông bà tôi lần cuối cùng năm năm trước. (Đúng)
C. Tôi đã gặp ông bà tôi trong năm năm. (Sai vì khác nghĩa so với câu
gốc.)
D. Tôi không gặp ông bà tôi cách đây 5 năm. (Sai vì khác nghĩa so với
câu gốc.)
Question 34
Father’s Day was created to complement Mother’s Day. Like
Mother’s Day (34) ________ honors mothers and motherhood,
: A. which B. who C. where D. when
Kiến thức: Mệnh đề quan hệ
* Đại từ quan hệ which dùng để thay thế cho danh từ đứng trước
nó, được dùng như chủ ngữ trong mệnh đề quan hệ.
Tạm dịch: Giống như Ngày của Mẹ, ngày mà tôn vinh những người
mẹ và tình mẫu tử.
Question 35
Father’s Day celebrates fatherhood and paternal bonds; it highlights
the (35) ________ of fathers in society.
A. conquest B. effect C. influence D. impact
Kiến thức: Từ vựng
A. conquest (n): sự chinh phục
B. effect (n): Sự thay đổi hoặc kết quả, hậu quả mà một ai đó/một
việc gì đó ảnh hưởng lên người khác/việc khác.( vừa tích cực, vừa
tiêu cực)
C. influence (n): Sự tác động mạnh mẽ làm thay đổi cách nghĩ, hành
động của một người hay sự phát triển của một việc nào đó (không
dùng đến vũ lực mà thường là cho họ một tấm gương để noi theo.)
D. impact (n): Có ảnh hưởng đáng chú ý lên một người/một việc nào
đó, đặc biệt theo chiều hướng xấu.
Tạm dịch: nó làm nổi bật ảnh hưởng của những người cha trong xã
hội.
Question 36
Her father, the Civil War veteran William Jackson Smart, was a single
parent who (36) ________ his six children there.
A. raised B. realized C. tookD. made
Kiến thức: Từ vựng
* Raise (v): chăm sóc một người hoặc một động vật hoặc thực vật, cho
đến khi chúng đã trưởng thành hoàn toàn.
Tạm dịch: Cha của cô, cựu chiến binh Nội chiến William Jackson Smart,
là một người cha đơn thân nuôi sáu đứa con của mình ở đó.
Question 37
Since then it has become a traditional day (37) ________ year.
: A. any B. another C. other D. every
Kiến thức: Từ vựng
• Every: mỗi, hằng.
Tạm dịch: Kể từ đó nó đã trở thành một ngày truyền thống hàng
năm.
Question 38
In recognition of what fathers do for their families, on this day
people may have a party celebrating male parenting or simply make
a phone call or send a greeting card. (38) ________, schools help
children prepare handmade gifts for their fathers many days before
the celebration.
A. Besides B. However C. In contrast D. Then
Kiến thức: Từ vựng
A. Besides: bên cạnh đó (thêm thông tin)
B. However: tuy nhiên (chỉ nhượng bộ)
C. In contrast: trái lại (chỉ tương phản)
D. Then: vậy thì (chỉ kết quả)
Tạm dịch: Bên cạnh đó, các trường học giúp các em chuẩn bị những
món quà tự tay làm để tặng cha nhiều ngày trước lễ kỷ niệm.
Question 39: What does the passage mainly discuss?
A.The role of instinct in animalbehavior
B.Observations that suggest consciousness in animal behavior
C.The use of food in studies of animal behavior
D.Differences between the behavior of animals in their natural environments and in
laboratoryexperiments.
Đoạn văn chủ yếu thảo luận về điều gì?
Vai trò của bản năng trong hành vi động vật
Những quan sát cho thấy ý thức trong hành vi động vật
Việc sử dụng thức ăn trong các nghiên cứu về hành vi động vật
Những sự khác nhau giữa hành vi của động vật trong môi trưởng tự nhiên và trong phòng thí
nghiệm.
Dẫn chứng: - Some animalbehavioristsargue that certain animals can remember past events
... These scientists, however, are cautiousabout the extent to which animalscan be credited
with consciousprocessing. (Một số nhà nghiên cứu hành vi của động vật tranh luận rằng một
số động vật có thể nhớ các sự việc trong quá khứ... Tuy nhiên, các nhà khoa này thận trọng về
mức độ mà động vật có thể được cho là xử lý có ý thức)
Explanationsof animalbehaviorthat leave out any sort of consciousness at all and ascribe
actionsentirely to instinct leave many questionsunanswered. (Các giải thích về hành vi động
vật bỏ qua bất cứ loạiý thức nào và gán cho các hành động hoàn toàn do bản năng để lại
nhiều câu hỏi chưa được trả lởi)
Other behaviorsthat may indicatesome cognition includetool use (Các hành vi khác có thể
chỉ ra một số nhận thức bao gồm việc sử dụng công cụ)
Question 40: Which of the following is NOT discussed as an ability
animals are thought to have?
A. Selecting among choices B. Anticipating events to come
C. Remembering past experiences D. Communicating emotions
Điều nào sau đây KHÔNG được thảo luận là khả năng mà động vật
có?
Lựa chọn trong số các lựa chọn
Dự đoán được sự sắp đến
Nhớ các việc trong quá khứ
Truyền đạt cảm xúc
Dẫn chứng: Some animal behaviorists argue that certain animals can
remember past events. anticipate future ones, make plans and
choices, and coordinate activities within a group. (Một số nhà nghiên
cứu hành vi của động vật tranh luận rằng một số động vật có thể nhớ
các sự việc quá khứ, dự đoán sự việc tương lai, lên kế hoạch và đưa
ra lựa chọn và phối hợp các hoạt động trong nhóm)
Question 41: What is the purpose of the honeybee dance?
A.To determine the quantity of food at a site
B.To communicate the location of food
C.To identify the type of nectar that is available
D.To identify the type of nectar that is available
Mục đích của vũ điệu ong mật?
Để xác định số lượng thức ăn ở một địa điểm
Để truyền đạt thông tin về vị trí thức ăn
Để tăng tốc độ di chuyển đến nguồn thức ăn
Để nhận ra loại mật hoa sẵn có.
Dẫn chứng: “Honeybees communicate the sources of nectar to one
another by doing a dance in a figure- eight pattern.” (Loài ong mật
truyền đạt thông tin về các nguồn mật hoa với nhau bằng việc thực
hiện vũ điệu hình số 8.)
Question42: The word “yet” in line 15 is closest in meaning to
A. however B. since C. generally D. so far
however: tuy nhiên
since: từ khi
generally: nói chung, thông thưởng
so far: cho đến bây giở
“But in one study, when experimenters kept changing the site of the food source,
each time moving the food 25 percent farther from the previous site, foraging
honeybees began to anticipate where the food source would appear next. When the
researchers arrived at the new location, they would find the bees circling the spot,
waiting for their food. No one has yet explained how bees, whose brains weigh four
ten- thousandths of an ounce, could have infeưed the location of the new site.“
(Nhưng trong một nghiên cứu, khi các nhà thí nghiệm tiếp tục thay đổi địa điểm đặt
thức ăn, mỗi lần di chuyển thức ăn cách xa hơn 25% so với địa điểm trước, những
con ong mật săn mồi bắt đầu dự đoán nguồn thức ăn sẽ xuất hiện ở đâu tiếp theo.
Khi các nhà nghiên cứu đến địa điểm mới, họ thấy những con ong vây quanh nơi đó,
chở đợi thức ăn. Cho đến bây giở chưa ai giải thích được làm thế nào mà loài ong có
bộ não với trọng lượng bốn phần mưởi nghìn của một ounce, có thể suy đoán được
vị trí của địa điểm mới.)
Do đó: yet ~ so far
Question 43: It can be inferred from the passage that brain size is
assumed to .
A. be an indicator of cognitive ability B. vary among individuals within
a species
C. be related to food consumption D. correspond to levels of
activity
Chúng ta có thể suy ra từ bài đọc rằng kích thước não bộ được cho là
.
một dấu hiệu của khả năng nhận thức
đa dạng giữa các thành viên trong cùng một loài
liên quan đến việc tiêu thụ thức ăn
tương đương với mức độ hoạt động
Dẫn chứng: “No one has yet explained how bees, whose brains weigh
four ten-thousandthsof an ounce could have inferred the location of
the new site.” (Chưa ai giải thích được làm thế nào mà loài ong có bộ
não với trọng lượng bốn phần mưởi nghìn của một ounce, có thể suy
đoán được vị trí của địa điểm mới.)
Question 44. What is the passage mainly about?
A.The difficulties of industrializationin North America
B.The influence of changes in manufacturing on the growth of urban centers
C.The rapid speed of industrialization inNorth America
D.Improved ways of organizing the manufacturing of goods
Bài đọc chủ yếu nói về điều gì?
Khó khăn của việc công nghiệp hóa ở Bắc Mỹ
Ảnh hưởng của thay đổi trong việc sản xuất vào sự phát triển của các trung tâm đô thị
Tốc độ công nghiệp hóa nhanh chóng ở Bắc Mỹ.
Những cách cải tiến tổ chức việc sản xuất hàng hóa
Dẫn chứng: - Industrialization cameto the United State after 1790 as North American
entrepreneurs increased productivityby reorganizing work and buildingfactories. These
innovationsin manufacturing boosted output and living standardsto an unprecedented
extent ... (Công nghiệp hóa đã đến nước Mỹ sau năm 1790 khi các doanhnhân Bắc Mỹ tăng
năng suất bằng cách tổ chức lại công việc và xây dựng nhà máy. Những cải tiến này trong
sản xuất đã giúp tăng sản lượng và mức sống đến mức độ chưa từng thấy...)
For tasks that were not suited to the outwork system, entrepreneurs created an even more
important new organization, the modem factory, which used power-driven machines and
assembly-line techniques to turn out large quantities of well-made goods. (Đối với các
nhiệm vụ không phù hợp với hệ thống gia công, các chủ thầu thậm chí đã tạo ra một tổ
chức mới quan trọng hơn, nhà máy hiện đại, mà sử dụng các máy chạy bằng năng lượng và
các kỹ thuật dây chuyền lắp ráp để tạo ra số lượng lớn hàng hóa được chế tạo tốt.)
Question45. The word “scope” in the second paragraph is closest in meaning to
.
A.value B. popularity C. extent D. diversity
Từ “scope” trong đoạn 2 gần nghĩa nhất với .
giá trị
sự phổ biến
quy mô, phạm vi
Sự đa dạng
“The impressive gain in output stemmed primarily from the way in which workers
made goods, since the 1790’s, North American entrepreneurs - even without
technological improvements - had broadened the scope of the outwork system
that made manufacturing more efficient by distributing materials to a succession of
workers who each performed a single step of the production process” (Sự gia tăng
ấn tượng về sản lượng xuất phát chủ yếu từ cách công nhân sản xuất hàng hóa, từ
những năm 1790 thì các doanh nhân Bắc Mỹ - thậm chí không có cải tiến công nghệ
- đã mở rộng phạm vi của hệ thống gia công giúp việc sản xuất hiệu quả hơn bằng
cách phân phát chất liệu cho một dây chuyền công nhân mà mỗi người thực hiện
một bước duy nhất trong quá trình sản xuất.)
Do đó: scope ~ extent
Question 46. The authormentions the shoe industry in the second paragraph to provide an
example of how
entrepreneurs increased output by using an extended outwork system
entrepreneurs used technologicalimprovementsto increase output
rural workers responded to “shoe bosses”
changes in the outwork system improved the quality of shoes
Question 46 Chọn đáp án A
Tác giả đề cập đến công nghiệp sản xuất giày trong đoạn 2 để cung cấp ví dụ về cách thức .....
các doanh nhân tăng sản lượng bằng việc sử dụng hệ thống gia công mở rộng
các doanh nhân sử dụng cài tiến công nghệ để tăng sản lượng
công nhân ở nông thôn phản ứng với “các ông chủ xưởng giày”
những thay đổi trong hệ thống gia công đã cải thiện chất lượng giày
Dẫn chứng: The impressive gain in output stemmed primarily from the way in which workers
made goods, since the 1790’s, North American entrepreneurs - even without technological
improvements - had broadened the scope of the outwork system that made manufacturing
more efficient by distributingmaterials to a succession of workers who each performed a
single step of the productionprocess. For example, during the 1820’s and 1830’s the shoe
industry greatlyexpanded the scale of the outwork system.... However, it also dramatically
increased the output of shoes while cutting their price. (Sự gia tăng ấn tượng về sản lượng
xuất phát chủ yếu từ cách công nhân sản xuất hàng hóa, từ những năm 1790 thì các doanh
nhân Bắc Mỹ - thậm chí không có cải tiến công nghệ - đă mở rộng phạm vi của hệ thống gia
công giúp việc sản xuất hiệu quả hơn bằng cách phân phát chất liệu cho một dây chuyền công
nhân mà mỗi người thực hiện một bước duy nhất trong qui trình sản xuất. Chẳng hạn như,
trong những năm 1820 và 1830, ngành công nghiệp sản xuất giày đã mở rộng quy mô của hệ
Question47. All of the following are true of the outwork system EXCEPT .
A. It involved stages of production.
B. It was more efficient than the systems used before 1790.
C. It made many employers less powerful than they had been before.
D. It did not necessarily involve any technological improvements.
Tất cả những điều sau đúng về hệ thống gia công NGOẠI TRỪ .
Nó bao gồm các giai đoạn sản xuất
Nó hiệu quả hơn các hệ thống đã được dùng trước 1790
Nó đã làm cho nhiều ông chủ ít quyền lực hơn trước đây
Nó không nhất thiết có cải tiến công nghệ
Dẫn chứng: - The impressive gain in output stemmed primarily from the way in which
workers made goods, since the 1790’s. North American entrepreneurs - even without
technological improvements - had broadened the scope of the outwork system that
made manufacturing more efficient by distributing materials to a succession of
workers who each performed a single step of the production process. => A, B, D đúng
This system of production made the employer a powerful “shoe boss” => C sai
Question 48. The word “prolific” in the last paragraph is closest in
meaning to .
A. famous B. productive C. self-employed D. progressive
famous (adj): nổi tiếng
productive (adj): sản xuất nhiều, sáng tác nhiều, phát minh nhiều
self-employed (adj): tự kinh doanh
progressive (adj): tiếp diễn, tiếng bộ
“As early as 1782 the prolific Delaware inventor Oliver Evans had built
a highly automated, laborsaving flour mill driven by water power.”
(Ngay từ năm 1782, nhà phát minh có nhiều sáng chế ở tiểu bang
Delaware tên là Oliver Evans đã phát minh máy xay bột tự động giúp
tiết kiệm sức lao đông được điều khiển bởi năng lượng nước.)
Do đó: prolific ~ productive
Question 49. According to the passage, how did later mills differ
from the mills built by Oliver Evans?
A.They were located away from large cities.
B.They used new technology to produce power.
C.They did not allow flour to cool before it was placed in Barrels.
D.They combined technology with the outwork system.
Theo bài đọc, máy xay sau này khác với máy xay được phát minh bởi
Oliver Evans như thế nào?
Chúng được đặt ở xa các thành phố lớn.
Chúng dùng công nghệ mới để tạo ra năng lượng.
Chúng không cho phép bột nguội trước khi được bỏ vào khoang.
Chúng kết hợp công nghệ với hệ thống gia công.
Dẫn chứng: “Subsequently, manufacturers made use of new
improved stationary steam engines to power their mills...” (Sau đó,
các nhà sản xuất đã sử dụng động cơ hơi nước cố định được cải tiến
để cung cấp năng lượng cho các máy xay...)
Question 50 The passage mentions which of the following as a result of
improvements in factory machinery?
A.It become easier for factory’ owners to find workers and customers.
B.Manufacturers had to employ more highly skilled workers.
C.The amount of power required for factories operate was reduced.
D.Factories could operate more than one engine at a time.
lượng cho các máy xay...)
Bài đọc đề cập đến điều nào sau đây là kết quả của sự cải thiện máy móc
nhà máy?
Các ông chủ nhà máy trở nên dễ dàng tìm công nhân và khách hàng.
Nhà sản xuất phải thuê nhiều công nhân có tay nghề cao.
Năng lượng cần cho nhà máy hoạt động thì giảm xuống.
Nhà máy không thể hoạt động nhiều hơn 1 động cơ vào một thởi điểm
Dẫn chứng: This new technology enabled them to build factories in the
nation’s largest cities, taking advantage of urban concentrations of
inexpensive labor, good transportation networks, and eager customers.
Question 1: Mr. Lam is a cycle driver in Ho Chi Minh City, who usually
has a ________ working day.
A. business B. busyC. busily D. busying
A. business /’biznis/ (n): công việc, việc buôn bán kinh doanh
B. busy /’bizi/ (a): bận, nhộn nhịp
C. busily/’buzili/ (a): một cách bận rộn, một cách nhộn nhịp
D. busying: không tồn tại từ này
=> Căn cứ vào mạo từ “a” cùng danh từ theo sau nên vị trí còn trống
cần một tính từ. Từ đó ta chọn B
Dịch nghĩa: Ông Lam là một người lái xích lô ở TP. Hồ Chí Minh, người
mà thường có một ngày làm
Question 2: You couldn’t give me a helping hand, _____?
A. could you B. do you C. couldn’t you D. don’t you
Dịch nghĩa: “Bạn không thể giúp tôi một tay phải không?”
Câu bắt đầu với you couldn’t thì thành lập hỏi đuôi sẽ dùng could you.
Question 3: The employees have been working______out to
get the job finished ahead of the deadline.
A. narrow B. big C. flat D. large
A. narrow /ˈnærəʊ/ (a): chật hẹp
B. big /bɪɡ/ (a): to lớn
C. flat /flæt/ (a): phẳng
D. large /lɑːdʒ/ (a): rộng lớn
Ta có: flat out: hoàn toàn, hết sức
Tạm dịch: Các nhân viên đã làm việc hết sức để hoàn thành công
việc trước thời hạn cuối.
Question 4: If you get a laptop as a reward, what will you use it
____?
A. by B. for C. to D. with
* Dịch nghĩa: Nếu bạn nhận được một chiếc máy tính xách tay như
một phần thưởng, bạn sẽ sử dụng nó để làm gì?
* Căn cứ vào: use for: dùng để làm gì
Question 5: The bank will insist you produce a driving________or
passport as a form of ID.
A. diploma B. certificate C. degree D. licence
A. diploma /dr pleume/ (n): chứng chỉ, văn băng (cái do trường đại
học, cao đăng cấp sau khi vuợt qua một kỳ thi đặc biệt hoặc một khóa
học nảo đó)
B. certificate /se trfiket/ (n): giấy chứng nhận, chứng chỉ (tai liệu
chính thức có chúa thông tin chinh xác, chuẩn xác như giấy khai sinh,
giay dăng kí kết hôn,. hoặc tài liệu được cung cấp bởi nhà trường sau
khi vượt qua kỳ thi)
C. degree /dr gri / (n). bằng cấp (bằng có khi mức độ, trình đo,năng
lưc của ai đó sau khi tốt nghiệp dại học, cao đảng)
D. licence/latsns/ (n): giấy phép (thể hiện quyền có thể am gl, so hữu
cái gì hay sử dụng cải gì một cach hợp pháp) Tạm dịch: Ngân hàng sẽ
yêu cầu bạn xuất trình giấy phép lài xe hoặc hó chiều dưới dạng ID.
Question 6: ____ the TRUMPET is the smallest brass instrument, it
can play the highest notes of all the brass instruments and often
plays in marches or fanfares.
A. Because B. Although C. However D. Since
Dịch nghĩa: Mặc dù kèn là nhạc cụ bằng đồng nhỏ nhất, nhưng nó có
thể chơi những nốt cao nhất trong tất cả các nhạc cụ bằng đồng và
thường chơi trong các cuộc diễn hành hoặc kèn lệnh.
Xét các đáp án:
A. Because → Mệnh đề chỉ nguyên nhân, dịch là - bởi vì, nếu cho vào
câu sẽ trở nên vô nghĩa
B. Although → Mệnh đề chỉ sự nhượng bộ, các liên từ được dịch là
mặc dù: Although = Even though = Though = In spite that + clause
C. However → however là tuy nhiên, nếu cho vào câu sẽ trở nên vô
nghĩa
D. Since → since = because (bởi vì), nếu cho vào câu sẽ trở nên vô
nghĩa
Question 7: His sister is not only a (n) ______________ singer but also
a distinguished painter.
A. famous opera Italian B. opera famous Italian
C. famous Italian opera D. Italian famous opera
*Theo quy tắc trật tự tính từ trong câu: OSASCOMP
: famous - Opinion; Italian - Origin
*Note: Opera (n): nghệ thuật, nhạc kịch ô-pê-ra
-Tính từ luôn đứng trước danh từ để bổ nghĩa cho danh từ
Tạm dịch: Chị anh ấy không chỉ là một ca sĩ opera người Ý nổi tiếng mà
còn là một hoạ sĩ lỗi lạc.
Question 8: Mark invented a new game, but it never really
_________________ with people.
A. called for B. caught on C. cut off D. came across
A. called for: yêu cầu
B. caught on: trở nên phổ biến
C. cut off: ngắt, cắt (điện)
D. came across: tình cờ gặp
Dịch nghĩa: Mark đã phát minh ra một trò chơi mới nhưng nó không
bao giờ thực sự trở nên phổ biến với mọi người.
Question 9: He fell down when he______towards the pagoda.
A. runB. runsC. was runningD. had run
“ When” dùng để diễn tả 1 hành động đang xảy ra 1 hành động
khác xen vào
Thì quá khứ tiếp diễn
Tạm dịch: Anh ấy bị ngã khi chạy về phía ngôi chùa.
Question 10. She will apply for a job _______.
A. when she is graduating from university B. until
she graduated from university
C. after she had graduated from university D. as
soon as she graduates from university
Xét các đáp án ta thấy:
- Động từ của vế câu cho trước ở trong đề bài ở thì tương lai diễn tả
một hành động chưa xảy ra hay sẽ xảy ra ở trong tương lai nên ở vế
sau, không thể chia động từ ở thì quá khứ, quá khứ tiếp diễn hay quá
khứ hoàn thành.
loại A, B, C 4 đáp án D đúng (phối thì hiện tại đơn với tương lai đơn)
Dịch: Cô ta sẽ đi xin việc ngay sau khi anh ta tốt nghiệp.
Question 11: It ___ without saying that Mrs Ngoc Anh is a very
enthusiastic teacher. I love her so much.
A. goesB. comes C. appears D. gets
A. goes / ɡoʊz/ (v): đi
B. comes /kʌmz/ (v): đến
C. appears /əˈpɪərz/ (v): xuất hiện
D. gets /gets/ (v): có được
Cấu trúc: It goes without saying: chắc chắn, hiển nhiên, khỏi phải
nói
Dịch nghĩa: Khỏi phải nói rằng cô Trang Anh là một giáo viên rất
nhiệt tình. Tôi yêu cô ấy rất nhiều.
Question 12: As soon as Ferlin came to party, he immediately set his
_______on Melin. Maybe he was captivated by her.
A. eyesB. heart C. decision D. feeling
A. eyes /aɪs/ (n): đôi mắt
B. heart /hɑːt/ (n): trái tim
C. decision /dɪˈsɪʒən/ (n): quyết định
D. feeling /ˈfiːlɪŋ/ (n): cảm xúc
=> Cấu trúc: Clap/lay/set eyes on sb/sth: để ý, để mắt tới cái gì/ai
đó ngay từ lần đầu gặp
Dịch nghĩa: Ngay khi Ferlin đến bữa tiệc, anh lập tức để ý tới Melin.
Có lẽ anh ấy đã bị cô quyến rũ.
Question 13. English ___________in many countries.
A. are spoke B. is spoke C. are spoken D. is spoken
Phương pháp giải:
Kiến thức: Câu bị động:
Giải chi tiết: Câu bị động thì Hiện tại đơn . Hành động được nhấn
mạnh là tiếng Anh được nói ở nhiều nước
S + is/are/am + V-ed/V3
Tạm dịch: Tiếng Anh được nói ở nhiều nước
Question 14: ___________ all the lights and other electric devices,
we left the classroom.
A. Having been turned off B. Turning off
C. To have turned off D. Having turned off
Xét các đáp án:
A. Having been turned off => không dùng dạng bị động ở đây vì đằng
sau đã có tân ngữ là “all the lights and other electric devices” => đáp
án A sai.
B. Turning off => Sai vì ở đây 2 vế đồng chủ ngữ, ta lược bỏ chủ ngữ
vế đầu, động từ đưa về dạng Ving khi nó mang nghĩa chủ động. Tuy
nhiên, hành động ở vế trước xảy ra trước nên ta dùng “Having Vp2”.
C. To have turned off => từ “to” đứng đầu tạo thành mệnh đề chỉ
mục đích. Nhưng xét vào ngữ nghĩa, câu diễn tả 2 hành động xảy ra
trước và sau, không phải nói về mục đích của hành động.
D. Having turned off => Đúng.
Tạm dịch: Sau khi tắt hết các đèn và các thiết bị điện tử khác, chúng
tôi rời khỏi lớp học.
Question 15: ___________ you study for these exams, the better you
will do.
A. The harder B. The more C. The hardest D. The
more hard
HD: Dịch nghĩa: Bạn càng học chăm chỉ cho các kỳ thi này, bạn sẽ càng
làm tốt hơn.
Xét các đáp án:
A. The harder / the better → Cấu trúc so sánh kép khi nói về 2 người
hoặc sự vật:
The more/-er + S + V, the more/-er + S + V
B. The more → Sai cấu trúc so sánh kép
C. The hardest → Sai cấu trúc so sánh kép
D. The more hard → Sai cấu trúc so sánh hơn (hard là từ 1 âm tiết,
good là trường hợp bất quy tắc)
Question 16. Ben: “You didn’t go to school yesterday, did you?”
Jasmine: “ . I saw you, but you were talking to someone”
A. No, I didn’t B. Yes, I didn’t C. Let me see D. I went
Kỹ năng: Dịch
Giải thích:
Ben: “Hôm qua, cậu không đi học phải không?”
Jasamine: “Tớ có đi. Tớ đã nhìn thấy cậu, nhưng cậu đang nói
chuyệnvới ai đó.”
Không, tớ đã không đi
Sai cấu trúc
Để xem nào
Question 17. Mother: “How come you didn’t tell me that you would
quit your job?” Lisa: “ .”
A.I’d love to tell you now
B.Because I knew that you would make a fuss about it
C.I have no idea
D.Because I’m so bored with it
Kỹ năng:
Giải thích:
Mẹ: “Sao con có thể không nói với mẹ rằng con bỏ việc?”
Lisa: Bởi vì con biết là mẹ sẽ làm ầm lên
Con muốn nói với mẹ bây giờ.
Con không biết.
D.Bởi vì con chán chuyện đấy rồi
Question 18:
A. attitude B. manager C. invention D. company
A. attitude/ˈætɪtjuːd/(n): thái độ (từ này có trọng âm rơi vào âm
tiết đầu tiên)
B. manager /ˈmænɪdʒər/ (n): người quản lý (từ này có trọng âm rơi
vào âm tiết đầu tiên. Vì theo quy tắc nếu tất cả các âm mà ngắn hết
thì trọng âm rơi vào âm tiết đầu.)
C. invention /ɪnˈvenʃn/ (n): sáng chế (từ này có trọng âm rơi vào âm
tiết thứ hai. Vì theo quy tắc đuôi -ion làm trọng âm rơi vào trước
âm đó.)
D. company /ˈkʌmpəni/: công ty (từ này có trọng âm rơi vào âm tiết
đầu tiên. Vì theo quy tắc nếu tất cả các âm mà ngắn hết thì trọng
âm rơi vào âm tiết đầu.)
=> Đáp án C có trọng âm rơi vào âm tiết thứ hai, các phương án còn
lại có trọng âm rơi vào âm tiết thứ nhất.
Question 19:
A. carry B. remove C. protect D. consist
A. carry /ˈkæri/ (v): mang (từ này có trọng âm rơi vào âm tiết đầu
tiên. Vì theo quy tắc nếu tất cả các âm mà ngắn hết thì trọng âm rơi
vào âm tiết đầu.)
B. remove /rɪˈmuːv/ (v): loại bỏ (từ này có trọng âm rơi vào âm tiết
thứ hai. Vì theo quy tắc, trọng âm ưu tiên rơi vào nguyên âm dài
/uː/.)
C. protect /prəˈtekt/ (v): bảo vệ (từ này có trọng âm rơi vào âm tiết
thứ hai. Vì theo quy tắc trọng âm không rơi vào âm /ə/.)
D. consist /kənˈsɪst/ (v): bao gồm (từ này có trọng âm rơi vào âm
tiết thứ hai. Vì theo quy tắc trọng âm không rơi vào âm /ə/.)
Đáp án A có trọng âm rơi vào âm tiết thứ nhất, các phương án còn
lại có trọng âm rơi vào âm tiết thứ hai.
Question 20:
A. finished B. developed C. defeated D. looked
A. finished /ˈfɪnɪʃt/
B. developed/dɪˈveləpt/
C. defeated /dɪˈfiːtid/
D. looked /lʊkt/
Giải thích: Có 3 cách phát âm ed chính:
1. Đuôi /ed/ được phát âm là /t/: Khi động từ có phát âm kết thúc
là /s/, /f/, /p/, /ʃ/, /tʃ/, /k/ và những động từ có từ phát âm cuối là
“s”.
2. Đuôi /ed/ được phát âm là /id/: Khi động từ có phát âm kết thúc
là /t/ hay /d/.
3. Đuôi /ed/ được phát âm là /d/ với những trường hợp còn lại.
Question 21: A. sale B. chalk C. dateD. plane
A. sale /seɪl/
B. chalk /tʃɔːk/
C. date /deɪt/
D. plane /pleɪn/
Question 22. In the 1980s TV viewers began to hook up
videocassette players to their TVs.
A. combine B. connect C. fasten D. blend
Kỹ năng: Đồng/trái nghĩa Giải thích:
hook up = connect: kết nối
combine: kết hợp
fasten: buộc chặt, thắt blend: pha trộn
Question 23. The police have not had time to complete their
investigations, but they have concluded
tentatively that the explosion was caused by a bomb.
A. temporally B. intentionally C. certainly D. hesitantly
Kỹ năng: Đồng/trái nghĩa
Giải thích:
tentatively = hesitantly: ngập ngừng, không chắc chắn
temporally: tạm thời intentionally:cố tình certainly: chắc chắn
Question 24. Humans can use language deceptively by telling lies
or half-truths.
A. in an honest way B. in a dishonest way C. for a serious
purpose D. at the wrong time
A Kỹ năng: Đồng/trái nghĩa Giải thích:
deceptively: một cách dối trá, dễ nhầm lần>< in an honest way:
một cách thật thà
Question 25. Tired of being a tiny cog in a vast machine, he
handed in his resignation.
A. an important person B. a large piece of equipment
C. a small group of people D. a significant instrument
Kỹ năng: Đồng/trái nghĩa Giải thích:
a tiny cog in a vast machine: một phần nhỏ, kém quan trọng>< an
important person: một người quan
trọng
Question 26: The only way to eliminate world terrorism is by
united opposition.
A. Only with united opposition could we eliminate terrorism.
B. Only by united opposition can we eliminate terrorism.
C. Only in this way can world terrorism be eliminated.
D. Only then can we eliminate terrorism.
Dịch nghĩa: Cách duy nhất để loại bỏ khủng bố thế giới là thành lập
một phe liên minh chống lại.
Xét các đáp án:
A. Only with united opposition could we eliminate terrorism. →
Trợ động từ là “can”, không phải là “could”.
B. Only by united opposition can we eliminate terrorism. → Cấu
trúc đảo ngữ: Only by V-ing/N + Auxiliary + S + V.
C. Only in this way can world terrorism be eliminated. → Thiếu “by
united opposition”.
D. Only then can we eliminate terrorism. → Thiếu “by united
Question 27: Collin is not very good at English. He failed the English exam
last week.
A. If Collin were good at English, he wouldn’t have failed the English exam
last week.
B. If Collin were better at English, he would pass the English exam last week.
C. Should Collin be good at English, he would not fail the English exam last
week.
D. Had Collin been better at English, he would’ve passed the English exam
last week.
Đáp án A
Ta thấy ở câu đề bài cho một động từ ở thì hiện tại, một động từ ở thì quá
khứ.
=> Nên suy ra ở đây ta sẽ dùng câu điều kiện trộn giữa loại 2 và loại 3.
- If S + V - qkđ, S + would + have + V-pp
Tạm dịch: Collin không giỏi tiếng Anh lắm. Anh ấy đã trượt kỳ thi tiếng Anh
tuần trước.
= A. Nếu Collin giỏi tiếng Anh, anh ấy đã không trượt kỳ thi tiếng Anh tuần
trước.
Question 28: Papyrus (A) was(B)usedful for making not only paper
(C) but also sails, baskets, (D) and clothing.
(useful=> used)
Cấu trúc: be used for + V-ing: được sử dụng làm gì
Question 29: (A) Because the boy spent (B) too much time (C)
surfing the Internet and playing computer games, (D) she didn’t
pass the final exam.
Cấu trúc: many + N- đếm được sốnhiều: nhiều; much + N - không
đếm được
+ “time - thời gian” là danh từ không đếm được nên ta dùng
“much”
The boy là chủ ngữ đại từ thay thế “he”
Question 30: According to (A) most medical experts, massage (B)
relieves pain and anxiety, eases depression and (C) speeding up
recovery from (D) illnesses.
Đây là câu trúc song song nên các động từ “relieve, ease và speed
up” trong câu này đều được chia theo danh từ “massage”
Dịch: Theo nhiều chuyên gia y học, việc mát-xa làm giảm cơn đau và
lo lắng, làm giảm chứng trầm cảm
và đẩy nhanh việc phục hồi bệnh tật.
(speeding up => speeds up)
Question 31: It was impossible that she forgot to wear the helmet.
A. She should have worn the helmet
B. She must have worn the helmet
C She might have forgot to wear the helmet
D. She needn’t have forgot wearing the helmet
Dịch nghĩa: Anh ta không thể nào đã quên đội mũ bảo hiểm được.
A. Cô ta lẽ ra nên đội mũ bảo hiểm. (Loại vì câu không mang nghĩa
khuyên bảo)
B. Cô ta chắc hẳn đã đội mũ bảo hiểm. (Đúng)
C. Cô ta có lẽ đã quên đội mũ bảo hiểm. (Sai vì câu không mang nghĩa
dự đoán)
D Cô ta lẽ ra không cần thiết phải quên đã đội mũ bảo hiểm. (Sai nghĩa
so với câu gốc)
Question 32: Cindy said: “I haven’t seen John since last month.”
A. Cindy said she hasn’t seen John since the previous month.
B. Cindy said she hadn’t seen John since the previous month.
C. Cindy said she wasn’t seen John since the previous month.
D. Cindy said she doesn’t see John since the previous month.
HD: Dịch nghĩa: Cindy nói: “Tôi không gặp John kể từ tháng trước.”
Xét các đáp án:
A. Cindy said she hasn’t seen John since the previous month. → Vì là
câu tường thuật nên hiện tại hoàn thành chuyển thành quá khứ
hoàn thành.
B. Cindy said she hadn’t seen John since the previous month. → Vì là
câu tường thuật nên hiện tại hoàn thành chuyển thành quá khứ
hoàn thành.
C. Cindy said she wasn’t seen John since the previous month. →
Không dùng dạng diễn đạt này.
D. Cindy said she doesn’t see John since the previous month. →
Không dùng dạng diễn đạt này với cấu trúc câu tường thuật.
Question 33: He last visited London three years ago.
A. He has been in London for three years.
B. He hasn't visited London for three years.
C.He didn't visit London three years ago.
D.He was in London for three years.
HD: “Anh ấy đến Luân Đôn lần cuối vào ba năm trước.”
A. Anh ấy đã ở Luận Đôn ba năm rồi. (Sai vì nghĩa không giống với câu
gốc.)
B. Anh ấy đã không đến Luân Đôn 3 năm rồi. (Đúng)
C. Anh ấy đã không đến Luân Đôn ba năm trước. (Sai vì nghĩa không
giống với câu gốc.)
D. Anh ấy đã ở Luân Đôn được ba năm. (Sai vì nghĩa không giống với
câu gốc.)
Question 34.
Rapidly rising caseloads alarm researchers, who fear the virus may
make its way across the globe. (34) _________, scientists cannot yet
predict how many deaths may result.
A. So B. However C. Because D. Although
Kiến thức: Liên từ
Giải thích:
A. So S + V: vì vậy, vì thế B. However, S + V: tuy nhiên
C. Because S + V: bởi vì D. Although S + V: mặc dù
Tạm dịch: Các nhà nghiên cứu đang lo sợ rằng virus có thể
xâm nhập khắp toàn cầu. Tuy nhiên, các nhà khoa học vẫn
chưa thể dự đoán có bao nhiêu trường hợp tử vong.
Chọn B.
Question 35.
The Wuhan coronavirus spreading from China is now likely to become
a pandemic (35) _________ circles the globe, according to many of
the world’s leading infectious disease experts.
A. whoB. why C. which D. where
Kiến thức: Đại từ quan hệ
Giải thích:
Dấu hiệu: “a pandemic” (đại dịch) là danh từ chỉ vật.
Trong mệnh đề quan hệ:
- who: thay thế cho danh từ chỉ người, đóng vai trò làm chủ ngữ
- why: tại sao
- which: thay thế cho danh từ chỉ vật
- where: ở đâu
Tạm dịch: Theo nhiều chuyên gia về bệnh truyền nhiễm hàng
đầu thế giới, virus coronavirus ở Vũ Hán lây lan từ Trung Quốc
có khả năng trở thành đại dịch trên toàn cầu.
Chọn C.
Question 36.
A pandemic - an ongoing epidemic on two or more continents- may
well have global consequences, despite the extraordinary travel
restrictions and quarantines now imposed by China and (36)
_________ countries, including the United States.
A. other B. more C. another D. others
Kiến thức: Từ vựng
Giải thích:
Sau chỗ trống là danh từ “countries” (các nước) là danh từ số
nhiều.
A. other + N (số nhiều): những cái khác B. more + N số nhiều:
nhiều hơn
C. another + N số ít: cái khác D. others = other + N (số
nhiều): những cái khác
Tạm dịch: Một đại dịch - một dịch bệnh đang diễn ra trên hai
hoặc nhiều lục địa - có thể gây ra những hậu quả toàn cầu, bất
chấp những hạn chế đi lại và cách ly bị bắt buộc bởi Trung Quốc
và các nước khác, bao gồm cả Hoa Kỳ.
Question 37.
Scientists do not yet know how (37) _________ the new coronavirus
is, however, so there is uncertainty about how much damage a
pandemic might cause.
A. unhealthy B. lethal C. dead D. wonderful
Kiến thức: Từ vựng
Giải thích:
A. unhealthy (adj): không lành mạnh B. lethal (adj): làm
chết người, gây chết người
C. dead (adj): chết D. wonderful (adj): tuyệt vời
Scientists do not yet know how (37) lethal the new coronavirus
is, however, so there is uncertainty about how much damage a
pandemic might cause.
Tạm dịch: Tuy nhiên, các nhà khoa học vẫn chưa biết mức độ
gây chết người của loại coronavirus mới này, do đó, vẫn chưa
chắc chắn về mức độ thiệt hại mà một đại dịch có thể gây ra.
DE PHAT TRIEN THEO CAU TRUC DE MINH HOA 2022 MON TIENG ANH DE 11 15.pdf
DE PHAT TRIEN THEO CAU TRUC DE MINH HOA 2022 MON TIENG ANH DE 11 15.pdf
DE PHAT TRIEN THEO CAU TRUC DE MINH HOA 2022 MON TIENG ANH DE 11 15.pdf
DE PHAT TRIEN THEO CAU TRUC DE MINH HOA 2022 MON TIENG ANH DE 11 15.pdf
DE PHAT TRIEN THEO CAU TRUC DE MINH HOA 2022 MON TIENG ANH DE 11 15.pdf
DE PHAT TRIEN THEO CAU TRUC DE MINH HOA 2022 MON TIENG ANH DE 11 15.pdf
DE PHAT TRIEN THEO CAU TRUC DE MINH HOA 2022 MON TIENG ANH DE 11 15.pdf
DE PHAT TRIEN THEO CAU TRUC DE MINH HOA 2022 MON TIENG ANH DE 11 15.pdf
DE PHAT TRIEN THEO CAU TRUC DE MINH HOA 2022 MON TIENG ANH DE 11 15.pdf
DE PHAT TRIEN THEO CAU TRUC DE MINH HOA 2022 MON TIENG ANH DE 11 15.pdf
DE PHAT TRIEN THEO CAU TRUC DE MINH HOA 2022 MON TIENG ANH DE 11 15.pdf
DE PHAT TRIEN THEO CAU TRUC DE MINH HOA 2022 MON TIENG ANH DE 11 15.pdf
DE PHAT TRIEN THEO CAU TRUC DE MINH HOA 2022 MON TIENG ANH DE 11 15.pdf
DE PHAT TRIEN THEO CAU TRUC DE MINH HOA 2022 MON TIENG ANH DE 11 15.pdf
DE PHAT TRIEN THEO CAU TRUC DE MINH HOA 2022 MON TIENG ANH DE 11 15.pdf
DE PHAT TRIEN THEO CAU TRUC DE MINH HOA 2022 MON TIENG ANH DE 11 15.pdf
DE PHAT TRIEN THEO CAU TRUC DE MINH HOA 2022 MON TIENG ANH DE 11 15.pdf
DE PHAT TRIEN THEO CAU TRUC DE MINH HOA 2022 MON TIENG ANH DE 11 15.pdf
DE PHAT TRIEN THEO CAU TRUC DE MINH HOA 2022 MON TIENG ANH DE 11 15.pdf
DE PHAT TRIEN THEO CAU TRUC DE MINH HOA 2022 MON TIENG ANH DE 11 15.pdf
DE PHAT TRIEN THEO CAU TRUC DE MINH HOA 2022 MON TIENG ANH DE 11 15.pdf
DE PHAT TRIEN THEO CAU TRUC DE MINH HOA 2022 MON TIENG ANH DE 11 15.pdf
DE PHAT TRIEN THEO CAU TRUC DE MINH HOA 2022 MON TIENG ANH DE 11 15.pdf
DE PHAT TRIEN THEO CAU TRUC DE MINH HOA 2022 MON TIENG ANH DE 11 15.pdf
DE PHAT TRIEN THEO CAU TRUC DE MINH HOA 2022 MON TIENG ANH DE 11 15.pdf
DE PHAT TRIEN THEO CAU TRUC DE MINH HOA 2022 MON TIENG ANH DE 11 15.pdf
DE PHAT TRIEN THEO CAU TRUC DE MINH HOA 2022 MON TIENG ANH DE 11 15.pdf
DE PHAT TRIEN THEO CAU TRUC DE MINH HOA 2022 MON TIENG ANH DE 11 15.pdf
DE PHAT TRIEN THEO CAU TRUC DE MINH HOA 2022 MON TIENG ANH DE 11 15.pdf
DE PHAT TRIEN THEO CAU TRUC DE MINH HOA 2022 MON TIENG ANH DE 11 15.pdf
DE PHAT TRIEN THEO CAU TRUC DE MINH HOA 2022 MON TIENG ANH DE 11 15.pdf
DE PHAT TRIEN THEO CAU TRUC DE MINH HOA 2022 MON TIENG ANH DE 11 15.pdf
DE PHAT TRIEN THEO CAU TRUC DE MINH HOA 2022 MON TIENG ANH DE 11 15.pdf
DE PHAT TRIEN THEO CAU TRUC DE MINH HOA 2022 MON TIENG ANH DE 11 15.pdf
DE PHAT TRIEN THEO CAU TRUC DE MINH HOA 2022 MON TIENG ANH DE 11 15.pdf
DE PHAT TRIEN THEO CAU TRUC DE MINH HOA 2022 MON TIENG ANH DE 11 15.pdf
DE PHAT TRIEN THEO CAU TRUC DE MINH HOA 2022 MON TIENG ANH DE 11 15.pdf
DE PHAT TRIEN THEO CAU TRUC DE MINH HOA 2022 MON TIENG ANH DE 11 15.pdf
DE PHAT TRIEN THEO CAU TRUC DE MINH HOA 2022 MON TIENG ANH DE 11 15.pdf
DE PHAT TRIEN THEO CAU TRUC DE MINH HOA 2022 MON TIENG ANH DE 11 15.pdf
DE PHAT TRIEN THEO CAU TRUC DE MINH HOA 2022 MON TIENG ANH DE 11 15.pdf
DE PHAT TRIEN THEO CAU TRUC DE MINH HOA 2022 MON TIENG ANH DE 11 15.pdf
DE PHAT TRIEN THEO CAU TRUC DE MINH HOA 2022 MON TIENG ANH DE 11 15.pdf
DE PHAT TRIEN THEO CAU TRUC DE MINH HOA 2022 MON TIENG ANH DE 11 15.pdf
DE PHAT TRIEN THEO CAU TRUC DE MINH HOA 2022 MON TIENG ANH DE 11 15.pdf
DE PHAT TRIEN THEO CAU TRUC DE MINH HOA 2022 MON TIENG ANH DE 11 15.pdf
DE PHAT TRIEN THEO CAU TRUC DE MINH HOA 2022 MON TIENG ANH DE 11 15.pdf
DE PHAT TRIEN THEO CAU TRUC DE MINH HOA 2022 MON TIENG ANH DE 11 15.pdf
DE PHAT TRIEN THEO CAU TRUC DE MINH HOA 2022 MON TIENG ANH DE 11 15.pdf
DE PHAT TRIEN THEO CAU TRUC DE MINH HOA 2022 MON TIENG ANH DE 11 15.pdf
DE PHAT TRIEN THEO CAU TRUC DE MINH HOA 2022 MON TIENG ANH DE 11 15.pdf
DE PHAT TRIEN THEO CAU TRUC DE MINH HOA 2022 MON TIENG ANH DE 11 15.pdf
DE PHAT TRIEN THEO CAU TRUC DE MINH HOA 2022 MON TIENG ANH DE 11 15.pdf
DE PHAT TRIEN THEO CAU TRUC DE MINH HOA 2022 MON TIENG ANH DE 11 15.pdf
DE PHAT TRIEN THEO CAU TRUC DE MINH HOA 2022 MON TIENG ANH DE 11 15.pdf
DE PHAT TRIEN THEO CAU TRUC DE MINH HOA 2022 MON TIENG ANH DE 11 15.pdf
DE PHAT TRIEN THEO CAU TRUC DE MINH HOA 2022 MON TIENG ANH DE 11 15.pdf
DE PHAT TRIEN THEO CAU TRUC DE MINH HOA 2022 MON TIENG ANH DE 11 15.pdf
DE PHAT TRIEN THEO CAU TRUC DE MINH HOA 2022 MON TIENG ANH DE 11 15.pdf
DE PHAT TRIEN THEO CAU TRUC DE MINH HOA 2022 MON TIENG ANH DE 11 15.pdf
DE PHAT TRIEN THEO CAU TRUC DE MINH HOA 2022 MON TIENG ANH DE 11 15.pdf
DE PHAT TRIEN THEO CAU TRUC DE MINH HOA 2022 MON TIENG ANH DE 11 15.pdf
DE PHAT TRIEN THEO CAU TRUC DE MINH HOA 2022 MON TIENG ANH DE 11 15.pdf
DE PHAT TRIEN THEO CAU TRUC DE MINH HOA 2022 MON TIENG ANH DE 11 15.pdf
DE PHAT TRIEN THEO CAU TRUC DE MINH HOA 2022 MON TIENG ANH DE 11 15.pdf
DE PHAT TRIEN THEO CAU TRUC DE MINH HOA 2022 MON TIENG ANH DE 11 15.pdf
DE PHAT TRIEN THEO CAU TRUC DE MINH HOA 2022 MON TIENG ANH DE 11 15.pdf
DE PHAT TRIEN THEO CAU TRUC DE MINH HOA 2022 MON TIENG ANH DE 11 15.pdf
DE PHAT TRIEN THEO CAU TRUC DE MINH HOA 2022 MON TIENG ANH DE 11 15.pdf
DE PHAT TRIEN THEO CAU TRUC DE MINH HOA 2022 MON TIENG ANH DE 11 15.pdf
DE PHAT TRIEN THEO CAU TRUC DE MINH HOA 2022 MON TIENG ANH DE 11 15.pdf
DE PHAT TRIEN THEO CAU TRUC DE MINH HOA 2022 MON TIENG ANH DE 11 15.pdf
DE PHAT TRIEN THEO CAU TRUC DE MINH HOA 2022 MON TIENG ANH DE 11 15.pdf
DE PHAT TRIEN THEO CAU TRUC DE MINH HOA 2022 MON TIENG ANH DE 11 15.pdf
DE PHAT TRIEN THEO CAU TRUC DE MINH HOA 2022 MON TIENG ANH DE 11 15.pdf
DE PHAT TRIEN THEO CAU TRUC DE MINH HOA 2022 MON TIENG ANH DE 11 15.pdf
DE PHAT TRIEN THEO CAU TRUC DE MINH HOA 2022 MON TIENG ANH DE 11 15.pdf
DE PHAT TRIEN THEO CAU TRUC DE MINH HOA 2022 MON TIENG ANH DE 11 15.pdf
DE PHAT TRIEN THEO CAU TRUC DE MINH HOA 2022 MON TIENG ANH DE 11 15.pdf
DE PHAT TRIEN THEO CAU TRUC DE MINH HOA 2022 MON TIENG ANH DE 11 15.pdf
DE PHAT TRIEN THEO CAU TRUC DE MINH HOA 2022 MON TIENG ANH DE 11 15.pdf
DE PHAT TRIEN THEO CAU TRUC DE MINH HOA 2022 MON TIENG ANH DE 11 15.pdf
DE PHAT TRIEN THEO CAU TRUC DE MINH HOA 2022 MON TIENG ANH DE 11 15.pdf
DE PHAT TRIEN THEO CAU TRUC DE MINH HOA 2022 MON TIENG ANH DE 11 15.pdf
DE PHAT TRIEN THEO CAU TRUC DE MINH HOA 2022 MON TIENG ANH DE 11 15.pdf
DE PHAT TRIEN THEO CAU TRUC DE MINH HOA 2022 MON TIENG ANH DE 11 15.pdf
DE PHAT TRIEN THEO CAU TRUC DE MINH HOA 2022 MON TIENG ANH DE 11 15.pdf
DE PHAT TRIEN THEO CAU TRUC DE MINH HOA 2022 MON TIENG ANH DE 11 15.pdf
DE PHAT TRIEN THEO CAU TRUC DE MINH HOA 2022 MON TIENG ANH DE 11 15.pdf
DE PHAT TRIEN THEO CAU TRUC DE MINH HOA 2022 MON TIENG ANH DE 11 15.pdf
DE PHAT TRIEN THEO CAU TRUC DE MINH HOA 2022 MON TIENG ANH DE 11 15.pdf
DE PHAT TRIEN THEO CAU TRUC DE MINH HOA 2022 MON TIENG ANH DE 11 15.pdf
DE PHAT TRIEN THEO CAU TRUC DE MINH HOA 2022 MON TIENG ANH DE 11 15.pdf
DE PHAT TRIEN THEO CAU TRUC DE MINH HOA 2022 MON TIENG ANH DE 11 15.pdf
DE PHAT TRIEN THEO CAU TRUC DE MINH HOA 2022 MON TIENG ANH DE 11 15.pdf
DE PHAT TRIEN THEO CAU TRUC DE MINH HOA 2022 MON TIENG ANH DE 11 15.pdf
DE PHAT TRIEN THEO CAU TRUC DE MINH HOA 2022 MON TIENG ANH DE 11 15.pdf
DE PHAT TRIEN THEO CAU TRUC DE MINH HOA 2022 MON TIENG ANH DE 11 15.pdf
DE PHAT TRIEN THEO CAU TRUC DE MINH HOA 2022 MON TIENG ANH DE 11 15.pdf
DE PHAT TRIEN THEO CAU TRUC DE MINH HOA 2022 MON TIENG ANH DE 11 15.pdf
DE PHAT TRIEN THEO CAU TRUC DE MINH HOA 2022 MON TIENG ANH DE 11 15.pdf
DE PHAT TRIEN THEO CAU TRUC DE MINH HOA 2022 MON TIENG ANH DE 11 15.pdf
DE PHAT TRIEN THEO CAU TRUC DE MINH HOA 2022 MON TIENG ANH DE 11 15.pdf
DE PHAT TRIEN THEO CAU TRUC DE MINH HOA 2022 MON TIENG ANH DE 11 15.pdf
DE PHAT TRIEN THEO CAU TRUC DE MINH HOA 2022 MON TIENG ANH DE 11 15.pdf
DE PHAT TRIEN THEO CAU TRUC DE MINH HOA 2022 MON TIENG ANH DE 11 15.pdf
DE PHAT TRIEN THEO CAU TRUC DE MINH HOA 2022 MON TIENG ANH DE 11 15.pdf
DE PHAT TRIEN THEO CAU TRUC DE MINH HOA 2022 MON TIENG ANH DE 11 15.pdf
DE PHAT TRIEN THEO CAU TRUC DE MINH HOA 2022 MON TIENG ANH DE 11 15.pdf
DE PHAT TRIEN THEO CAU TRUC DE MINH HOA 2022 MON TIENG ANH DE 11 15.pdf
DE PHAT TRIEN THEO CAU TRUC DE MINH HOA 2022 MON TIENG ANH DE 11 15.pdf

More Related Content

What's hot

Đề Tiếng Anh 12 cơ bản unit 4 có đáp án - VipLam.Net
Đề Tiếng Anh 12 cơ bản unit 4 có đáp án - VipLam.NetĐề Tiếng Anh 12 cơ bản unit 4 có đáp án - VipLam.Net
Đề Tiếng Anh 12 cơ bản unit 4 có đáp án - VipLam.Net
Thùy Linh
 
Dịch nghĩa và giải thích chi tiết starter toeic unit 3
Dịch nghĩa và giải thích chi tiết starter toeic unit 3Dịch nghĩa và giải thích chi tiết starter toeic unit 3
Dịch nghĩa và giải thích chi tiết starter toeic unit 3
my nguyễn
 
Starter Toeic
Starter ToeicStarter Toeic
Starter Toeic
Võ Phúc
 
Dịch nghĩa và giải thích chi tiết starter toeic unit 2
Dịch nghĩa và giải thích chi tiết starter toeic unit 2Dịch nghĩa và giải thích chi tiết starter toeic unit 2
Dịch nghĩa và giải thích chi tiết starter toeic unit 2
my nguyễn
 
Cách dùng modal perfect và bài tập thực hành
Cách dùng modal perfect và bài tập thực hànhCách dùng modal perfect và bài tập thực hành
Cách dùng modal perfect và bài tập thực hànhthuonglindo
 
đáP án và giải thích đề 8
đáP án và giải thích đề 8đáP án và giải thích đề 8
đáP án và giải thích đề 8Huynh ICT
 
100 cấu trúc và cụm từ thông dụng trong tiếng anh phổ thông
100 cấu trúc và cụm từ thông dụng trong tiếng anh phổ thông100 cấu trúc và cụm từ thông dụng trong tiếng anh phổ thông
100 cấu trúc và cụm từ thông dụng trong tiếng anh phổ thông
Hong Phuong Nguyen
 
Bai tap-tieng-anh-12unit-10
Bai tap-tieng-anh-12unit-10Bai tap-tieng-anh-12unit-10
Bai tap-tieng-anh-12unit-10
Huyen Tran
 
Tổng hợp 1000 từ tiếng anh Word form thông dụng
Tổng hợp 1000 từ tiếng anh Word form thông dụng Tổng hợp 1000 từ tiếng anh Word form thông dụng
Tổng hợp 1000 từ tiếng anh Word form thông dụng
Jackson Linh
 
50 ĐỀ PHÁT TRIỂN THEO CẤU TRÚC ĐỀ MINH HỌA BGD NGÀY 22-3-2024 KỲ THI TỐT NGHI...
50 ĐỀ PHÁT TRIỂN THEO CẤU TRÚC ĐỀ MINH HỌA BGD NGÀY 22-3-2024 KỲ THI TỐT NGHI...50 ĐỀ PHÁT TRIỂN THEO CẤU TRÚC ĐỀ MINH HỌA BGD NGÀY 22-3-2024 KỲ THI TỐT NGHI...
50 ĐỀ PHÁT TRIỂN THEO CẤU TRÚC ĐỀ MINH HỌA BGD NGÀY 22-3-2024 KỲ THI TỐT NGHI...
Nguyen Thanh Tu Collection
 
Đề Tiếng Anh 12 cơ bản unit 8 có đáp án - VipLam.Net
Đề Tiếng Anh 12 cơ bản unit 8 có đáp án - VipLam.NetĐề Tiếng Anh 12 cơ bản unit 8 có đáp án - VipLam.Net
Đề Tiếng Anh 12 cơ bản unit 8 có đáp án - VipLam.Net
Thùy Linh
 
50 ĐỀ PHÁT TRIỂN THEO CẤU TRÚC ĐỀ MINH HỌA BGD NGÀY 22-3-2024 KỲ THI TỐT NGHI...
50 ĐỀ PHÁT TRIỂN THEO CẤU TRÚC ĐỀ MINH HỌA BGD NGÀY 22-3-2024 KỲ THI TỐT NGHI...50 ĐỀ PHÁT TRIỂN THEO CẤU TRÚC ĐỀ MINH HỌA BGD NGÀY 22-3-2024 KỲ THI TỐT NGHI...
50 ĐỀ PHÁT TRIỂN THEO CẤU TRÚC ĐỀ MINH HỌA BGD NGÀY 22-3-2024 KỲ THI TỐT NGHI...
Nguyen Thanh Tu Collection
 
ĐỀ KIỂM TRA GIỮA KÌ 1 TIẾNG ANH 11 GLOBAL SUCCESS NĂM HỌC 2023 - 2024 (CÓ LỜI...
ĐỀ KIỂM TRA GIỮA KÌ 1 TIẾNG ANH 11 GLOBAL SUCCESS NĂM HỌC 2023 - 2024 (CÓ LỜI...ĐỀ KIỂM TRA GIỮA KÌ 1 TIẾNG ANH 11 GLOBAL SUCCESS NĂM HỌC 2023 - 2024 (CÓ LỜI...
ĐỀ KIỂM TRA GIỮA KÌ 1 TIẾNG ANH 11 GLOBAL SUCCESS NĂM HỌC 2023 - 2024 (CÓ LỜI...
Nguyen Thanh Tu Collection
 
Đề Tiếng Anh 12 cơ bản unit 12 có đáp án - VipLam.Net
Đề Tiếng Anh 12 cơ bản unit 12 có đáp án - VipLam.NetĐề Tiếng Anh 12 cơ bản unit 12 có đáp án - VipLam.Net
Đề Tiếng Anh 12 cơ bản unit 12 có đáp án - VipLam.Net
Thùy Linh
 
Tổng hợp đề Tiếng Anh 12 cơ bản unit 1 đến unit 16 có đáp án - VipLam.Net
Tổng hợp đề Tiếng Anh 12 cơ bản unit 1 đến unit 16 có đáp án - VipLam.NetTổng hợp đề Tiếng Anh 12 cơ bản unit 1 đến unit 16 có đáp án - VipLam.Net
Tổng hợp đề Tiếng Anh 12 cơ bản unit 1 đến unit 16 có đáp án - VipLam.Net
Thùy Linh
 
Đề Tiếng Anh 12 cơ bản unit 6 có đáp án - VipLam.Net
Đề Tiếng Anh 12 cơ bản unit 6 có đáp án - VipLam.NetĐề Tiếng Anh 12 cơ bản unit 6 có đáp án - VipLam.Net
Đề Tiếng Anh 12 cơ bản unit 6 có đáp án - VipLam.Net
Thùy Linh
 
Lý thuyết câu bị động
Lý thuyết câu bị độngLý thuyết câu bị động
Lý thuyết câu bị động
Tươi Sama
 
Bai tap trac nghiem ve cach phat am ed va s/es
Bai tap trac nghiem ve cach phat am ed va s/esBai tap trac nghiem ve cach phat am ed va s/es
Bai tap trac nghiem ve cach phat am ed va s/es
Vui Lên Bạn Nhé
 
Đề Tiếng Anh 12 cơ bản unit 5 có đáp án - VipLam.Net
Đề Tiếng Anh 12 cơ bản unit 5 có đáp án - VipLam.NetĐề Tiếng Anh 12 cơ bản unit 5 có đáp án - VipLam.Net
Đề Tiếng Anh 12 cơ bản unit 5 có đáp án - VipLam.Net
Thùy Linh
 
Các trường hợp viết lại câu full
Các trường hợp viết lại câu fullCác trường hợp viết lại câu full
Các trường hợp viết lại câu full
Trong Nguyen
 

What's hot (20)

Đề Tiếng Anh 12 cơ bản unit 4 có đáp án - VipLam.Net
Đề Tiếng Anh 12 cơ bản unit 4 có đáp án - VipLam.NetĐề Tiếng Anh 12 cơ bản unit 4 có đáp án - VipLam.Net
Đề Tiếng Anh 12 cơ bản unit 4 có đáp án - VipLam.Net
 
Dịch nghĩa và giải thích chi tiết starter toeic unit 3
Dịch nghĩa và giải thích chi tiết starter toeic unit 3Dịch nghĩa và giải thích chi tiết starter toeic unit 3
Dịch nghĩa và giải thích chi tiết starter toeic unit 3
 
Starter Toeic
Starter ToeicStarter Toeic
Starter Toeic
 
Dịch nghĩa và giải thích chi tiết starter toeic unit 2
Dịch nghĩa và giải thích chi tiết starter toeic unit 2Dịch nghĩa và giải thích chi tiết starter toeic unit 2
Dịch nghĩa và giải thích chi tiết starter toeic unit 2
 
Cách dùng modal perfect và bài tập thực hành
Cách dùng modal perfect và bài tập thực hànhCách dùng modal perfect và bài tập thực hành
Cách dùng modal perfect và bài tập thực hành
 
đáP án và giải thích đề 8
đáP án và giải thích đề 8đáP án và giải thích đề 8
đáP án và giải thích đề 8
 
100 cấu trúc và cụm từ thông dụng trong tiếng anh phổ thông
100 cấu trúc và cụm từ thông dụng trong tiếng anh phổ thông100 cấu trúc và cụm từ thông dụng trong tiếng anh phổ thông
100 cấu trúc và cụm từ thông dụng trong tiếng anh phổ thông
 
Bai tap-tieng-anh-12unit-10
Bai tap-tieng-anh-12unit-10Bai tap-tieng-anh-12unit-10
Bai tap-tieng-anh-12unit-10
 
Tổng hợp 1000 từ tiếng anh Word form thông dụng
Tổng hợp 1000 từ tiếng anh Word form thông dụng Tổng hợp 1000 từ tiếng anh Word form thông dụng
Tổng hợp 1000 từ tiếng anh Word form thông dụng
 
50 ĐỀ PHÁT TRIỂN THEO CẤU TRÚC ĐỀ MINH HỌA BGD NGÀY 22-3-2024 KỲ THI TỐT NGHI...
50 ĐỀ PHÁT TRIỂN THEO CẤU TRÚC ĐỀ MINH HỌA BGD NGÀY 22-3-2024 KỲ THI TỐT NGHI...50 ĐỀ PHÁT TRIỂN THEO CẤU TRÚC ĐỀ MINH HỌA BGD NGÀY 22-3-2024 KỲ THI TỐT NGHI...
50 ĐỀ PHÁT TRIỂN THEO CẤU TRÚC ĐỀ MINH HỌA BGD NGÀY 22-3-2024 KỲ THI TỐT NGHI...
 
Đề Tiếng Anh 12 cơ bản unit 8 có đáp án - VipLam.Net
Đề Tiếng Anh 12 cơ bản unit 8 có đáp án - VipLam.NetĐề Tiếng Anh 12 cơ bản unit 8 có đáp án - VipLam.Net
Đề Tiếng Anh 12 cơ bản unit 8 có đáp án - VipLam.Net
 
50 ĐỀ PHÁT TRIỂN THEO CẤU TRÚC ĐỀ MINH HỌA BGD NGÀY 22-3-2024 KỲ THI TỐT NGHI...
50 ĐỀ PHÁT TRIỂN THEO CẤU TRÚC ĐỀ MINH HỌA BGD NGÀY 22-3-2024 KỲ THI TỐT NGHI...50 ĐỀ PHÁT TRIỂN THEO CẤU TRÚC ĐỀ MINH HỌA BGD NGÀY 22-3-2024 KỲ THI TỐT NGHI...
50 ĐỀ PHÁT TRIỂN THEO CẤU TRÚC ĐỀ MINH HỌA BGD NGÀY 22-3-2024 KỲ THI TỐT NGHI...
 
ĐỀ KIỂM TRA GIỮA KÌ 1 TIẾNG ANH 11 GLOBAL SUCCESS NĂM HỌC 2023 - 2024 (CÓ LỜI...
ĐỀ KIỂM TRA GIỮA KÌ 1 TIẾNG ANH 11 GLOBAL SUCCESS NĂM HỌC 2023 - 2024 (CÓ LỜI...ĐỀ KIỂM TRA GIỮA KÌ 1 TIẾNG ANH 11 GLOBAL SUCCESS NĂM HỌC 2023 - 2024 (CÓ LỜI...
ĐỀ KIỂM TRA GIỮA KÌ 1 TIẾNG ANH 11 GLOBAL SUCCESS NĂM HỌC 2023 - 2024 (CÓ LỜI...
 
Đề Tiếng Anh 12 cơ bản unit 12 có đáp án - VipLam.Net
Đề Tiếng Anh 12 cơ bản unit 12 có đáp án - VipLam.NetĐề Tiếng Anh 12 cơ bản unit 12 có đáp án - VipLam.Net
Đề Tiếng Anh 12 cơ bản unit 12 có đáp án - VipLam.Net
 
Tổng hợp đề Tiếng Anh 12 cơ bản unit 1 đến unit 16 có đáp án - VipLam.Net
Tổng hợp đề Tiếng Anh 12 cơ bản unit 1 đến unit 16 có đáp án - VipLam.NetTổng hợp đề Tiếng Anh 12 cơ bản unit 1 đến unit 16 có đáp án - VipLam.Net
Tổng hợp đề Tiếng Anh 12 cơ bản unit 1 đến unit 16 có đáp án - VipLam.Net
 
Đề Tiếng Anh 12 cơ bản unit 6 có đáp án - VipLam.Net
Đề Tiếng Anh 12 cơ bản unit 6 có đáp án - VipLam.NetĐề Tiếng Anh 12 cơ bản unit 6 có đáp án - VipLam.Net
Đề Tiếng Anh 12 cơ bản unit 6 có đáp án - VipLam.Net
 
Lý thuyết câu bị động
Lý thuyết câu bị độngLý thuyết câu bị động
Lý thuyết câu bị động
 
Bai tap trac nghiem ve cach phat am ed va s/es
Bai tap trac nghiem ve cach phat am ed va s/esBai tap trac nghiem ve cach phat am ed va s/es
Bai tap trac nghiem ve cach phat am ed va s/es
 
Đề Tiếng Anh 12 cơ bản unit 5 có đáp án - VipLam.Net
Đề Tiếng Anh 12 cơ bản unit 5 có đáp án - VipLam.NetĐề Tiếng Anh 12 cơ bản unit 5 có đáp án - VipLam.Net
Đề Tiếng Anh 12 cơ bản unit 5 có đáp án - VipLam.Net
 
Các trường hợp viết lại câu full
Các trường hợp viết lại câu fullCác trường hợp viết lại câu full
Các trường hợp viết lại câu full
 

Similar to DE PHAT TRIEN THEO CAU TRUC DE MINH HOA 2022 MON TIENG ANH DE 11 15.pdf

CD21 Exercise 2.1 KEY.docx tieng anh cho
CD21 Exercise 2.1 KEY.docx tieng anh choCD21 Exercise 2.1 KEY.docx tieng anh cho
CD21 Exercise 2.1 KEY.docx tieng anh cho
namc250
 
đáP án và giải thích đề 9
đáP án và giải thích đề 9đáP án và giải thích đề 9
đáP án và giải thích đề 9Huynh ICT
 
đáP án và giải thích đề 24
đáP án và giải thích đề 24đáP án và giải thích đề 24
đáP án và giải thích đề 24Huynh ICT
 
đáP án và giải thích đề 20
đáP án và giải thích đề 20đáP án và giải thích đề 20
đáP án và giải thích đề 20Huynh ICT
 
đáP án và giải thích đề 16
đáP án và giải thích đề 16đáP án và giải thích đề 16
đáP án và giải thích đề 16Huynh ICT
 
đáP án và giải thích đề 23
đáP án và giải thích đề 23đáP án và giải thích đề 23
đáP án và giải thích đề 23Huynh ICT
 
đáP án và giải thích đề 15
đáP án và giải thích đề 15đáP án và giải thích đề 15
đáP án và giải thích đề 15Huynh ICT
 
đáP án và giải thích đề 17
đáP án và giải thích đề 17đáP án và giải thích đề 17
đáP án và giải thích đề 17Huynh ICT
 
đáP án và giải thích đề 31
đáP án và giải thích đề 31đáP án và giải thích đề 31
đáP án và giải thích đề 31Huynh ICT
 
đáP án và giải thích đề 13
đáP án và giải thích đề 13đáP án và giải thích đề 13
đáP án và giải thích đề 13Huynh ICT
 
đáP án và giải thích đề 32
đáP án và giải thích đề 32đáP án và giải thích đề 32
đáP án và giải thích đề 32Huynh ICT
 
đáP án và giải thích đề 25
đáP án và giải thích đề 25đáP án và giải thích đề 25
đáP án và giải thích đề 25Huynh ICT
 
đáP án và giải thích đề 19
đáP án và giải thích đề 19đáP án và giải thích đề 19
đáP án và giải thích đề 19Huynh ICT
 
đáP án và giải thích đề 4
đáP án và giải thích đề 4đáP án và giải thích đề 4
đáP án và giải thích đề 4Huynh ICT
 
Đề thi thử Ôn thi Tiếng Anh vào Cao Đẳng , Đại học năm 2013 - Đề 110
Đề thi thử  Ôn thi Tiếng Anh vào Cao Đẳng , Đại học năm 2013 - Đề 110Đề thi thử  Ôn thi Tiếng Anh vào Cao Đẳng , Đại học năm 2013 - Đề 110
Đề thi thử Ôn thi Tiếng Anh vào Cao Đẳng , Đại học năm 2013 - Đề 110phamnhakb
 
đáP án và giải thích đề 14
đáP án và giải thích đề 14đáP án và giải thích đề 14
đáP án và giải thích đề 14Huynh ICT
 
đáP án và giải thích đề 5
đáP án và giải thích đề 5đáP án và giải thích đề 5
đáP án và giải thích đề 5Huynh ICT
 
đáP án và giải thích đề 26
đáP án và giải thích đề 26đáP án và giải thích đề 26
đáP án và giải thích đề 26Huynh ICT
 
đáP án và giải thích đề 2
đáP án và giải thích đề 2đáP án và giải thích đề 2
đáP án và giải thích đề 2Huynh ICT
 
Giai chi tiet de thi dh khoi d t anh 2011
Giai chi tiet de thi dh khoi d t anh 2011Giai chi tiet de thi dh khoi d t anh 2011
Giai chi tiet de thi dh khoi d t anh 2011Hua Tran Phuong Thao
 

Similar to DE PHAT TRIEN THEO CAU TRUC DE MINH HOA 2022 MON TIENG ANH DE 11 15.pdf (20)

CD21 Exercise 2.1 KEY.docx tieng anh cho
CD21 Exercise 2.1 KEY.docx tieng anh choCD21 Exercise 2.1 KEY.docx tieng anh cho
CD21 Exercise 2.1 KEY.docx tieng anh cho
 
đáP án và giải thích đề 9
đáP án và giải thích đề 9đáP án và giải thích đề 9
đáP án và giải thích đề 9
 
đáP án và giải thích đề 24
đáP án và giải thích đề 24đáP án và giải thích đề 24
đáP án và giải thích đề 24
 
đáP án và giải thích đề 20
đáP án và giải thích đề 20đáP án và giải thích đề 20
đáP án và giải thích đề 20
 
đáP án và giải thích đề 16
đáP án và giải thích đề 16đáP án và giải thích đề 16
đáP án và giải thích đề 16
 
đáP án và giải thích đề 23
đáP án và giải thích đề 23đáP án và giải thích đề 23
đáP án và giải thích đề 23
 
đáP án và giải thích đề 15
đáP án và giải thích đề 15đáP án và giải thích đề 15
đáP án và giải thích đề 15
 
đáP án và giải thích đề 17
đáP án và giải thích đề 17đáP án và giải thích đề 17
đáP án và giải thích đề 17
 
đáP án và giải thích đề 31
đáP án và giải thích đề 31đáP án và giải thích đề 31
đáP án và giải thích đề 31
 
đáP án và giải thích đề 13
đáP án và giải thích đề 13đáP án và giải thích đề 13
đáP án và giải thích đề 13
 
đáP án và giải thích đề 32
đáP án và giải thích đề 32đáP án và giải thích đề 32
đáP án và giải thích đề 32
 
đáP án và giải thích đề 25
đáP án và giải thích đề 25đáP án và giải thích đề 25
đáP án và giải thích đề 25
 
đáP án và giải thích đề 19
đáP án và giải thích đề 19đáP án và giải thích đề 19
đáP án và giải thích đề 19
 
đáP án và giải thích đề 4
đáP án và giải thích đề 4đáP án và giải thích đề 4
đáP án và giải thích đề 4
 
Đề thi thử Ôn thi Tiếng Anh vào Cao Đẳng , Đại học năm 2013 - Đề 110
Đề thi thử  Ôn thi Tiếng Anh vào Cao Đẳng , Đại học năm 2013 - Đề 110Đề thi thử  Ôn thi Tiếng Anh vào Cao Đẳng , Đại học năm 2013 - Đề 110
Đề thi thử Ôn thi Tiếng Anh vào Cao Đẳng , Đại học năm 2013 - Đề 110
 
đáP án và giải thích đề 14
đáP án và giải thích đề 14đáP án và giải thích đề 14
đáP án và giải thích đề 14
 
đáP án và giải thích đề 5
đáP án và giải thích đề 5đáP án và giải thích đề 5
đáP án và giải thích đề 5
 
đáP án và giải thích đề 26
đáP án và giải thích đề 26đáP án và giải thích đề 26
đáP án và giải thích đề 26
 
đáP án và giải thích đề 2
đáP án và giải thích đề 2đáP án và giải thích đề 2
đáP án và giải thích đề 2
 
Giai chi tiet de thi dh khoi d t anh 2011
Giai chi tiet de thi dh khoi d t anh 2011Giai chi tiet de thi dh khoi d t anh 2011
Giai chi tiet de thi dh khoi d t anh 2011
 

More from Nguyen Thanh Tu Collection

BÀI TẬP BỔ TRỢ TIẾNG ANH LỚP 8 - CẢ NĂM - FRIENDS PLUS - NĂM HỌC 2023-2024 (B...
BÀI TẬP BỔ TRỢ TIẾNG ANH LỚP 8 - CẢ NĂM - FRIENDS PLUS - NĂM HỌC 2023-2024 (B...BÀI TẬP BỔ TRỢ TIẾNG ANH LỚP 8 - CẢ NĂM - FRIENDS PLUS - NĂM HỌC 2023-2024 (B...
BÀI TẬP BỔ TRỢ TIẾNG ANH LỚP 8 - CẢ NĂM - FRIENDS PLUS - NĂM HỌC 2023-2024 (B...
Nguyen Thanh Tu Collection
 
TỔNG HỢP 135 CÂU HỎI DI TRUYỀN PHÂN TỬ LUYỆN THI HỌC SINH GIỎI THPT MÔN SINH ...
TỔNG HỢP 135 CÂU HỎI DI TRUYỀN PHÂN TỬ LUYỆN THI HỌC SINH GIỎI THPT MÔN SINH ...TỔNG HỢP 135 CÂU HỎI DI TRUYỀN PHÂN TỬ LUYỆN THI HỌC SINH GIỎI THPT MÔN SINH ...
TỔNG HỢP 135 CÂU HỎI DI TRUYỀN PHÂN TỬ LUYỆN THI HỌC SINH GIỎI THPT MÔN SINH ...
Nguyen Thanh Tu Collection
 
BÀI TẬP DẠY THÊM HÓA HỌC LỚP 12 - CẢ NĂM - THEO FORM THI MỚI BGD 2025 (DÙNG C...
BÀI TẬP DẠY THÊM HÓA HỌC LỚP 12 - CẢ NĂM - THEO FORM THI MỚI BGD 2025 (DÙNG C...BÀI TẬP DẠY THÊM HÓA HỌC LỚP 12 - CẢ NĂM - THEO FORM THI MỚI BGD 2025 (DÙNG C...
BÀI TẬP DẠY THÊM HÓA HỌC LỚP 12 - CẢ NĂM - THEO FORM THI MỚI BGD 2025 (DÙNG C...
Nguyen Thanh Tu Collection
 
BÀI TẬP BỔ TRỢ TIẾNG ANH LỚP 9 CẢ NĂM - GLOBAL SUCCESS - NĂM HỌC 2024-2025 - ...
BÀI TẬP BỔ TRỢ TIẾNG ANH LỚP 9 CẢ NĂM - GLOBAL SUCCESS - NĂM HỌC 2024-2025 - ...BÀI TẬP BỔ TRỢ TIẾNG ANH LỚP 9 CẢ NĂM - GLOBAL SUCCESS - NĂM HỌC 2024-2025 - ...
BÀI TẬP BỔ TRỢ TIẾNG ANH LỚP 9 CẢ NĂM - GLOBAL SUCCESS - NĂM HỌC 2024-2025 - ...
Nguyen Thanh Tu Collection
 
BÀI TẬP DẠY THÊM TIẾNG ANH LỚP 7 CẢ NĂM FRIENDS PLUS SÁCH CHÂN TRỜI SÁNG TẠO ...
BÀI TẬP DẠY THÊM TIẾNG ANH LỚP 7 CẢ NĂM FRIENDS PLUS SÁCH CHÂN TRỜI SÁNG TẠO ...BÀI TẬP DẠY THÊM TIẾNG ANH LỚP 7 CẢ NĂM FRIENDS PLUS SÁCH CHÂN TRỜI SÁNG TẠO ...
BÀI TẬP DẠY THÊM TIẾNG ANH LỚP 7 CẢ NĂM FRIENDS PLUS SÁCH CHÂN TRỜI SÁNG TẠO ...
Nguyen Thanh Tu Collection
 
CHUYÊN ĐỀ DẠY THÊM HÓA HỌC LỚP 10 - SÁCH MỚI - FORM BÀI TẬP 2025 (DÙNG CHUNG ...
CHUYÊN ĐỀ DẠY THÊM HÓA HỌC LỚP 10 - SÁCH MỚI - FORM BÀI TẬP 2025 (DÙNG CHUNG ...CHUYÊN ĐỀ DẠY THÊM HÓA HỌC LỚP 10 - SÁCH MỚI - FORM BÀI TẬP 2025 (DÙNG CHUNG ...
CHUYÊN ĐỀ DẠY THÊM HÓA HỌC LỚP 10 - SÁCH MỚI - FORM BÀI TẬP 2025 (DÙNG CHUNG ...
Nguyen Thanh Tu Collection
 
BÀI TẬP BỔ TRỢ TIẾNG ANH 8 CẢ NĂM - GLOBAL SUCCESS - NĂM HỌC 2023-2024 (CÓ FI...
BÀI TẬP BỔ TRỢ TIẾNG ANH 8 CẢ NĂM - GLOBAL SUCCESS - NĂM HỌC 2023-2024 (CÓ FI...BÀI TẬP BỔ TRỢ TIẾNG ANH 8 CẢ NĂM - GLOBAL SUCCESS - NĂM HỌC 2023-2024 (CÓ FI...
BÀI TẬP BỔ TRỢ TIẾNG ANH 8 CẢ NĂM - GLOBAL SUCCESS - NĂM HỌC 2023-2024 (CÓ FI...
Nguyen Thanh Tu Collection
 
BÀI TẬP BỔ TRỢ TIẾNG ANH I-LEARN SMART WORLD 9 CẢ NĂM CÓ TEST THEO UNIT NĂM H...
BÀI TẬP BỔ TRỢ TIẾNG ANH I-LEARN SMART WORLD 9 CẢ NĂM CÓ TEST THEO UNIT NĂM H...BÀI TẬP BỔ TRỢ TIẾNG ANH I-LEARN SMART WORLD 9 CẢ NĂM CÓ TEST THEO UNIT NĂM H...
BÀI TẬP BỔ TRỢ TIẾNG ANH I-LEARN SMART WORLD 9 CẢ NĂM CÓ TEST THEO UNIT NĂM H...
Nguyen Thanh Tu Collection
 
CHUYÊN ĐỀ BỒI DƯỠNG HỌC SINH GIỎI KHOA HỌC TỰ NHIÊN 9 CHƯƠNG TRÌNH MỚI - PHẦN...
CHUYÊN ĐỀ BỒI DƯỠNG HỌC SINH GIỎI KHOA HỌC TỰ NHIÊN 9 CHƯƠNG TRÌNH MỚI - PHẦN...CHUYÊN ĐỀ BỒI DƯỠNG HỌC SINH GIỎI KHOA HỌC TỰ NHIÊN 9 CHƯƠNG TRÌNH MỚI - PHẦN...
CHUYÊN ĐỀ BỒI DƯỠNG HỌC SINH GIỎI KHOA HỌC TỰ NHIÊN 9 CHƯƠNG TRÌNH MỚI - PHẦN...
Nguyen Thanh Tu Collection
 
BÀI TẬP BỔ TRỢ TIẾNG ANH GLOBAL SUCCESS LỚP 3 - CẢ NĂM (CÓ FILE NGHE VÀ ĐÁP Á...
BÀI TẬP BỔ TRỢ TIẾNG ANH GLOBAL SUCCESS LỚP 3 - CẢ NĂM (CÓ FILE NGHE VÀ ĐÁP Á...BÀI TẬP BỔ TRỢ TIẾNG ANH GLOBAL SUCCESS LỚP 3 - CẢ NĂM (CÓ FILE NGHE VÀ ĐÁP Á...
BÀI TẬP BỔ TRỢ TIẾNG ANH GLOBAL SUCCESS LỚP 3 - CẢ NĂM (CÓ FILE NGHE VÀ ĐÁP Á...
Nguyen Thanh Tu Collection
 
98 BÀI LUYỆN NGHE TUYỂN SINH VÀO LỚP 10 TIẾNG ANH DẠNG TRẮC NGHIỆM 4 CÂU TRẢ ...
98 BÀI LUYỆN NGHE TUYỂN SINH VÀO LỚP 10 TIẾNG ANH DẠNG TRẮC NGHIỆM 4 CÂU TRẢ ...98 BÀI LUYỆN NGHE TUYỂN SINH VÀO LỚP 10 TIẾNG ANH DẠNG TRẮC NGHIỆM 4 CÂU TRẢ ...
98 BÀI LUYỆN NGHE TUYỂN SINH VÀO LỚP 10 TIẾNG ANH DẠNG TRẮC NGHIỆM 4 CÂU TRẢ ...
Nguyen Thanh Tu Collection
 
GIÁO ÁN DẠY THÊM (KẾ HOẠCH BÀI BUỔI 2) - TIẾNG ANH 8 GLOBAL SUCCESS (2 CỘT) N...
GIÁO ÁN DẠY THÊM (KẾ HOẠCH BÀI BUỔI 2) - TIẾNG ANH 8 GLOBAL SUCCESS (2 CỘT) N...GIÁO ÁN DẠY THÊM (KẾ HOẠCH BÀI BUỔI 2) - TIẾNG ANH 8 GLOBAL SUCCESS (2 CỘT) N...
GIÁO ÁN DẠY THÊM (KẾ HOẠCH BÀI BUỔI 2) - TIẾNG ANH 8 GLOBAL SUCCESS (2 CỘT) N...
Nguyen Thanh Tu Collection
 
Nghiên cứu cơ chế và động học phản ứng giữa hợp chất Aniline (C6H5NH2) với gố...
Nghiên cứu cơ chế và động học phản ứng giữa hợp chất Aniline (C6H5NH2) với gố...Nghiên cứu cơ chế và động học phản ứng giữa hợp chất Aniline (C6H5NH2) với gố...
Nghiên cứu cơ chế và động học phản ứng giữa hợp chất Aniline (C6H5NH2) với gố...
Nguyen Thanh Tu Collection
 
50 ĐỀ LUYỆN THI IOE LỚP 9 - NĂM HỌC 2022-2023 (CÓ LINK HÌNH, FILE AUDIO VÀ ĐÁ...
50 ĐỀ LUYỆN THI IOE LỚP 9 - NĂM HỌC 2022-2023 (CÓ LINK HÌNH, FILE AUDIO VÀ ĐÁ...50 ĐỀ LUYỆN THI IOE LỚP 9 - NĂM HỌC 2022-2023 (CÓ LINK HÌNH, FILE AUDIO VÀ ĐÁ...
50 ĐỀ LUYỆN THI IOE LỚP 9 - NĂM HỌC 2022-2023 (CÓ LINK HÌNH, FILE AUDIO VÀ ĐÁ...
Nguyen Thanh Tu Collection
 
BÀI TẬP DẠY THÊM TOÁN LỚP 12 SÁCH MỚI THEO FORM THI MỚI BGD 2025 - CÁNH DIỀU ...
BÀI TẬP DẠY THÊM TOÁN LỚP 12 SÁCH MỚI THEO FORM THI MỚI BGD 2025 - CÁNH DIỀU ...BÀI TẬP DẠY THÊM TOÁN LỚP 12 SÁCH MỚI THEO FORM THI MỚI BGD 2025 - CÁNH DIỀU ...
BÀI TẬP DẠY THÊM TOÁN LỚP 12 SÁCH MỚI THEO FORM THI MỚI BGD 2025 - CÁNH DIỀU ...
Nguyen Thanh Tu Collection
 
TÀI LIỆU DẠY THÊM HÓA HỌC 12 - SÁCH MỚI (BẢN HS+GV) (FORM BÀI TẬP 2025 CHUNG ...
TÀI LIỆU DẠY THÊM HÓA HỌC 12 - SÁCH MỚI (BẢN HS+GV) (FORM BÀI TẬP 2025 CHUNG ...TÀI LIỆU DẠY THÊM HÓA HỌC 12 - SÁCH MỚI (BẢN HS+GV) (FORM BÀI TẬP 2025 CHUNG ...
TÀI LIỆU DẠY THÊM HÓA HỌC 12 - SÁCH MỚI (BẢN HS+GV) (FORM BÀI TẬP 2025 CHUNG ...
Nguyen Thanh Tu Collection
 
ĐỀ THI THỬ TUYỂN SINH VÀO LỚP 10 THPT MÔN TOÁN CÁC TỈNH NĂM HỌC 2023-2024 CÓ ...
ĐỀ THI THỬ TUYỂN SINH VÀO LỚP 10 THPT MÔN TOÁN CÁC TỈNH NĂM HỌC 2023-2024 CÓ ...ĐỀ THI THỬ TUYỂN SINH VÀO LỚP 10 THPT MÔN TOÁN CÁC TỈNH NĂM HỌC 2023-2024 CÓ ...
ĐỀ THI THỬ TUYỂN SINH VÀO LỚP 10 THPT MÔN TOÁN CÁC TỈNH NĂM HỌC 2023-2024 CÓ ...
Nguyen Thanh Tu Collection
 
BÀI TẬP DẠY THÊM TOÁN LỚP 12 SÁCH MỚI THEO FORM THI MỚI BGD 2025 - CHÂN TRỜI ...
BÀI TẬP DẠY THÊM TOÁN LỚP 12 SÁCH MỚI THEO FORM THI MỚI BGD 2025 - CHÂN TRỜI ...BÀI TẬP DẠY THÊM TOÁN LỚP 12 SÁCH MỚI THEO FORM THI MỚI BGD 2025 - CHÂN TRỜI ...
BÀI TẬP DẠY THÊM TOÁN LỚP 12 SÁCH MỚI THEO FORM THI MỚI BGD 2025 - CHÂN TRỜI ...
Nguyen Thanh Tu Collection
 
20 ĐỀ DỰ ĐOÁN - PHÁT TRIỂN ĐỀ MINH HỌA BGD KỲ THI TỐT NGHIỆP THPT NĂM 2024 MÔ...
20 ĐỀ DỰ ĐOÁN - PHÁT TRIỂN ĐỀ MINH HỌA BGD KỲ THI TỐT NGHIỆP THPT NĂM 2024 MÔ...20 ĐỀ DỰ ĐOÁN - PHÁT TRIỂN ĐỀ MINH HỌA BGD KỲ THI TỐT NGHIỆP THPT NĂM 2024 MÔ...
20 ĐỀ DỰ ĐOÁN - PHÁT TRIỂN ĐỀ MINH HỌA BGD KỲ THI TỐT NGHIỆP THPT NĂM 2024 MÔ...
Nguyen Thanh Tu Collection
 
BÀI TẬP DẠY THÊM HÓA HỌC LỚP 12 - CẢ NĂM - THEO FORM THI MỚI BGD 2025 (DÙNG C...
BÀI TẬP DẠY THÊM HÓA HỌC LỚP 12 - CẢ NĂM - THEO FORM THI MỚI BGD 2025 (DÙNG C...BÀI TẬP DẠY THÊM HÓA HỌC LỚP 12 - CẢ NĂM - THEO FORM THI MỚI BGD 2025 (DÙNG C...
BÀI TẬP DẠY THÊM HÓA HỌC LỚP 12 - CẢ NĂM - THEO FORM THI MỚI BGD 2025 (DÙNG C...
Nguyen Thanh Tu Collection
 

More from Nguyen Thanh Tu Collection (20)

BÀI TẬP BỔ TRỢ TIẾNG ANH LỚP 8 - CẢ NĂM - FRIENDS PLUS - NĂM HỌC 2023-2024 (B...
BÀI TẬP BỔ TRỢ TIẾNG ANH LỚP 8 - CẢ NĂM - FRIENDS PLUS - NĂM HỌC 2023-2024 (B...BÀI TẬP BỔ TRỢ TIẾNG ANH LỚP 8 - CẢ NĂM - FRIENDS PLUS - NĂM HỌC 2023-2024 (B...
BÀI TẬP BỔ TRỢ TIẾNG ANH LỚP 8 - CẢ NĂM - FRIENDS PLUS - NĂM HỌC 2023-2024 (B...
 
TỔNG HỢP 135 CÂU HỎI DI TRUYỀN PHÂN TỬ LUYỆN THI HỌC SINH GIỎI THPT MÔN SINH ...
TỔNG HỢP 135 CÂU HỎI DI TRUYỀN PHÂN TỬ LUYỆN THI HỌC SINH GIỎI THPT MÔN SINH ...TỔNG HỢP 135 CÂU HỎI DI TRUYỀN PHÂN TỬ LUYỆN THI HỌC SINH GIỎI THPT MÔN SINH ...
TỔNG HỢP 135 CÂU HỎI DI TRUYỀN PHÂN TỬ LUYỆN THI HỌC SINH GIỎI THPT MÔN SINH ...
 
BÀI TẬP DẠY THÊM HÓA HỌC LỚP 12 - CẢ NĂM - THEO FORM THI MỚI BGD 2025 (DÙNG C...
BÀI TẬP DẠY THÊM HÓA HỌC LỚP 12 - CẢ NĂM - THEO FORM THI MỚI BGD 2025 (DÙNG C...BÀI TẬP DẠY THÊM HÓA HỌC LỚP 12 - CẢ NĂM - THEO FORM THI MỚI BGD 2025 (DÙNG C...
BÀI TẬP DẠY THÊM HÓA HỌC LỚP 12 - CẢ NĂM - THEO FORM THI MỚI BGD 2025 (DÙNG C...
 
BÀI TẬP BỔ TRỢ TIẾNG ANH LỚP 9 CẢ NĂM - GLOBAL SUCCESS - NĂM HỌC 2024-2025 - ...
BÀI TẬP BỔ TRỢ TIẾNG ANH LỚP 9 CẢ NĂM - GLOBAL SUCCESS - NĂM HỌC 2024-2025 - ...BÀI TẬP BỔ TRỢ TIẾNG ANH LỚP 9 CẢ NĂM - GLOBAL SUCCESS - NĂM HỌC 2024-2025 - ...
BÀI TẬP BỔ TRỢ TIẾNG ANH LỚP 9 CẢ NĂM - GLOBAL SUCCESS - NĂM HỌC 2024-2025 - ...
 
BÀI TẬP DẠY THÊM TIẾNG ANH LỚP 7 CẢ NĂM FRIENDS PLUS SÁCH CHÂN TRỜI SÁNG TẠO ...
BÀI TẬP DẠY THÊM TIẾNG ANH LỚP 7 CẢ NĂM FRIENDS PLUS SÁCH CHÂN TRỜI SÁNG TẠO ...BÀI TẬP DẠY THÊM TIẾNG ANH LỚP 7 CẢ NĂM FRIENDS PLUS SÁCH CHÂN TRỜI SÁNG TẠO ...
BÀI TẬP DẠY THÊM TIẾNG ANH LỚP 7 CẢ NĂM FRIENDS PLUS SÁCH CHÂN TRỜI SÁNG TẠO ...
 
CHUYÊN ĐỀ DẠY THÊM HÓA HỌC LỚP 10 - SÁCH MỚI - FORM BÀI TẬP 2025 (DÙNG CHUNG ...
CHUYÊN ĐỀ DẠY THÊM HÓA HỌC LỚP 10 - SÁCH MỚI - FORM BÀI TẬP 2025 (DÙNG CHUNG ...CHUYÊN ĐỀ DẠY THÊM HÓA HỌC LỚP 10 - SÁCH MỚI - FORM BÀI TẬP 2025 (DÙNG CHUNG ...
CHUYÊN ĐỀ DẠY THÊM HÓA HỌC LỚP 10 - SÁCH MỚI - FORM BÀI TẬP 2025 (DÙNG CHUNG ...
 
BÀI TẬP BỔ TRỢ TIẾNG ANH 8 CẢ NĂM - GLOBAL SUCCESS - NĂM HỌC 2023-2024 (CÓ FI...
BÀI TẬP BỔ TRỢ TIẾNG ANH 8 CẢ NĂM - GLOBAL SUCCESS - NĂM HỌC 2023-2024 (CÓ FI...BÀI TẬP BỔ TRỢ TIẾNG ANH 8 CẢ NĂM - GLOBAL SUCCESS - NĂM HỌC 2023-2024 (CÓ FI...
BÀI TẬP BỔ TRỢ TIẾNG ANH 8 CẢ NĂM - GLOBAL SUCCESS - NĂM HỌC 2023-2024 (CÓ FI...
 
BÀI TẬP BỔ TRỢ TIẾNG ANH I-LEARN SMART WORLD 9 CẢ NĂM CÓ TEST THEO UNIT NĂM H...
BÀI TẬP BỔ TRỢ TIẾNG ANH I-LEARN SMART WORLD 9 CẢ NĂM CÓ TEST THEO UNIT NĂM H...BÀI TẬP BỔ TRỢ TIẾNG ANH I-LEARN SMART WORLD 9 CẢ NĂM CÓ TEST THEO UNIT NĂM H...
BÀI TẬP BỔ TRỢ TIẾNG ANH I-LEARN SMART WORLD 9 CẢ NĂM CÓ TEST THEO UNIT NĂM H...
 
CHUYÊN ĐỀ BỒI DƯỠNG HỌC SINH GIỎI KHOA HỌC TỰ NHIÊN 9 CHƯƠNG TRÌNH MỚI - PHẦN...
CHUYÊN ĐỀ BỒI DƯỠNG HỌC SINH GIỎI KHOA HỌC TỰ NHIÊN 9 CHƯƠNG TRÌNH MỚI - PHẦN...CHUYÊN ĐỀ BỒI DƯỠNG HỌC SINH GIỎI KHOA HỌC TỰ NHIÊN 9 CHƯƠNG TRÌNH MỚI - PHẦN...
CHUYÊN ĐỀ BỒI DƯỠNG HỌC SINH GIỎI KHOA HỌC TỰ NHIÊN 9 CHƯƠNG TRÌNH MỚI - PHẦN...
 
BÀI TẬP BỔ TRỢ TIẾNG ANH GLOBAL SUCCESS LỚP 3 - CẢ NĂM (CÓ FILE NGHE VÀ ĐÁP Á...
BÀI TẬP BỔ TRỢ TIẾNG ANH GLOBAL SUCCESS LỚP 3 - CẢ NĂM (CÓ FILE NGHE VÀ ĐÁP Á...BÀI TẬP BỔ TRỢ TIẾNG ANH GLOBAL SUCCESS LỚP 3 - CẢ NĂM (CÓ FILE NGHE VÀ ĐÁP Á...
BÀI TẬP BỔ TRỢ TIẾNG ANH GLOBAL SUCCESS LỚP 3 - CẢ NĂM (CÓ FILE NGHE VÀ ĐÁP Á...
 
98 BÀI LUYỆN NGHE TUYỂN SINH VÀO LỚP 10 TIẾNG ANH DẠNG TRẮC NGHIỆM 4 CÂU TRẢ ...
98 BÀI LUYỆN NGHE TUYỂN SINH VÀO LỚP 10 TIẾNG ANH DẠNG TRẮC NGHIỆM 4 CÂU TRẢ ...98 BÀI LUYỆN NGHE TUYỂN SINH VÀO LỚP 10 TIẾNG ANH DẠNG TRẮC NGHIỆM 4 CÂU TRẢ ...
98 BÀI LUYỆN NGHE TUYỂN SINH VÀO LỚP 10 TIẾNG ANH DẠNG TRẮC NGHIỆM 4 CÂU TRẢ ...
 
GIÁO ÁN DẠY THÊM (KẾ HOẠCH BÀI BUỔI 2) - TIẾNG ANH 8 GLOBAL SUCCESS (2 CỘT) N...
GIÁO ÁN DẠY THÊM (KẾ HOẠCH BÀI BUỔI 2) - TIẾNG ANH 8 GLOBAL SUCCESS (2 CỘT) N...GIÁO ÁN DẠY THÊM (KẾ HOẠCH BÀI BUỔI 2) - TIẾNG ANH 8 GLOBAL SUCCESS (2 CỘT) N...
GIÁO ÁN DẠY THÊM (KẾ HOẠCH BÀI BUỔI 2) - TIẾNG ANH 8 GLOBAL SUCCESS (2 CỘT) N...
 
Nghiên cứu cơ chế và động học phản ứng giữa hợp chất Aniline (C6H5NH2) với gố...
Nghiên cứu cơ chế và động học phản ứng giữa hợp chất Aniline (C6H5NH2) với gố...Nghiên cứu cơ chế và động học phản ứng giữa hợp chất Aniline (C6H5NH2) với gố...
Nghiên cứu cơ chế và động học phản ứng giữa hợp chất Aniline (C6H5NH2) với gố...
 
50 ĐỀ LUYỆN THI IOE LỚP 9 - NĂM HỌC 2022-2023 (CÓ LINK HÌNH, FILE AUDIO VÀ ĐÁ...
50 ĐỀ LUYỆN THI IOE LỚP 9 - NĂM HỌC 2022-2023 (CÓ LINK HÌNH, FILE AUDIO VÀ ĐÁ...50 ĐỀ LUYỆN THI IOE LỚP 9 - NĂM HỌC 2022-2023 (CÓ LINK HÌNH, FILE AUDIO VÀ ĐÁ...
50 ĐỀ LUYỆN THI IOE LỚP 9 - NĂM HỌC 2022-2023 (CÓ LINK HÌNH, FILE AUDIO VÀ ĐÁ...
 
BÀI TẬP DẠY THÊM TOÁN LỚP 12 SÁCH MỚI THEO FORM THI MỚI BGD 2025 - CÁNH DIỀU ...
BÀI TẬP DẠY THÊM TOÁN LỚP 12 SÁCH MỚI THEO FORM THI MỚI BGD 2025 - CÁNH DIỀU ...BÀI TẬP DẠY THÊM TOÁN LỚP 12 SÁCH MỚI THEO FORM THI MỚI BGD 2025 - CÁNH DIỀU ...
BÀI TẬP DẠY THÊM TOÁN LỚP 12 SÁCH MỚI THEO FORM THI MỚI BGD 2025 - CÁNH DIỀU ...
 
TÀI LIỆU DẠY THÊM HÓA HỌC 12 - SÁCH MỚI (BẢN HS+GV) (FORM BÀI TẬP 2025 CHUNG ...
TÀI LIỆU DẠY THÊM HÓA HỌC 12 - SÁCH MỚI (BẢN HS+GV) (FORM BÀI TẬP 2025 CHUNG ...TÀI LIỆU DẠY THÊM HÓA HỌC 12 - SÁCH MỚI (BẢN HS+GV) (FORM BÀI TẬP 2025 CHUNG ...
TÀI LIỆU DẠY THÊM HÓA HỌC 12 - SÁCH MỚI (BẢN HS+GV) (FORM BÀI TẬP 2025 CHUNG ...
 
ĐỀ THI THỬ TUYỂN SINH VÀO LỚP 10 THPT MÔN TOÁN CÁC TỈNH NĂM HỌC 2023-2024 CÓ ...
ĐỀ THI THỬ TUYỂN SINH VÀO LỚP 10 THPT MÔN TOÁN CÁC TỈNH NĂM HỌC 2023-2024 CÓ ...ĐỀ THI THỬ TUYỂN SINH VÀO LỚP 10 THPT MÔN TOÁN CÁC TỈNH NĂM HỌC 2023-2024 CÓ ...
ĐỀ THI THỬ TUYỂN SINH VÀO LỚP 10 THPT MÔN TOÁN CÁC TỈNH NĂM HỌC 2023-2024 CÓ ...
 
BÀI TẬP DẠY THÊM TOÁN LỚP 12 SÁCH MỚI THEO FORM THI MỚI BGD 2025 - CHÂN TRỜI ...
BÀI TẬP DẠY THÊM TOÁN LỚP 12 SÁCH MỚI THEO FORM THI MỚI BGD 2025 - CHÂN TRỜI ...BÀI TẬP DẠY THÊM TOÁN LỚP 12 SÁCH MỚI THEO FORM THI MỚI BGD 2025 - CHÂN TRỜI ...
BÀI TẬP DẠY THÊM TOÁN LỚP 12 SÁCH MỚI THEO FORM THI MỚI BGD 2025 - CHÂN TRỜI ...
 
20 ĐỀ DỰ ĐOÁN - PHÁT TRIỂN ĐỀ MINH HỌA BGD KỲ THI TỐT NGHIỆP THPT NĂM 2024 MÔ...
20 ĐỀ DỰ ĐOÁN - PHÁT TRIỂN ĐỀ MINH HỌA BGD KỲ THI TỐT NGHIỆP THPT NĂM 2024 MÔ...20 ĐỀ DỰ ĐOÁN - PHÁT TRIỂN ĐỀ MINH HỌA BGD KỲ THI TỐT NGHIỆP THPT NĂM 2024 MÔ...
20 ĐỀ DỰ ĐOÁN - PHÁT TRIỂN ĐỀ MINH HỌA BGD KỲ THI TỐT NGHIỆP THPT NĂM 2024 MÔ...
 
BÀI TẬP DẠY THÊM HÓA HỌC LỚP 12 - CẢ NĂM - THEO FORM THI MỚI BGD 2025 (DÙNG C...
BÀI TẬP DẠY THÊM HÓA HỌC LỚP 12 - CẢ NĂM - THEO FORM THI MỚI BGD 2025 (DÙNG C...BÀI TẬP DẠY THÊM HÓA HỌC LỚP 12 - CẢ NĂM - THEO FORM THI MỚI BGD 2025 (DÙNG C...
BÀI TẬP DẠY THÊM HÓA HỌC LỚP 12 - CẢ NĂM - THEO FORM THI MỚI BGD 2025 (DÙNG C...
 

Recently uploaded

Từ ngữ về con người và chiến tranh trong Nhật ký Đặng Thùy Trâm.pdf
Từ ngữ về con người và chiến tranh trong Nhật ký Đặng Thùy Trâm.pdfTừ ngữ về con người và chiến tranh trong Nhật ký Đặng Thùy Trâm.pdf
Từ ngữ về con người và chiến tranh trong Nhật ký Đặng Thùy Trâm.pdf
Man_Ebook
 
Các bình diện Ngôn ngữ học đối chiếu.pdf
Các bình diện Ngôn ngữ học đối chiếu.pdfCác bình diện Ngôn ngữ học đối chiếu.pdf
Các bình diện Ngôn ngữ học đối chiếu.pdf
linhlevietdav
 
YHocData.com-bộ-câu-hỏi-mô-phôi.pdf đầy đủ
YHocData.com-bộ-câu-hỏi-mô-phôi.pdf đầy đủYHocData.com-bộ-câu-hỏi-mô-phôi.pdf đầy đủ
YHocData.com-bộ-câu-hỏi-mô-phôi.pdf đầy đủ
duyanh05052004
 
Tuyển tập 9 chuyên đề bồi dưỡng Toán lớp 5 cơ bản và nâng cao ôn thi vào lớp ...
Tuyển tập 9 chuyên đề bồi dưỡng Toán lớp 5 cơ bản và nâng cao ôn thi vào lớp ...Tuyển tập 9 chuyên đề bồi dưỡng Toán lớp 5 cơ bản và nâng cao ôn thi vào lớp ...
Tuyển tập 9 chuyên đề bồi dưỡng Toán lớp 5 cơ bản và nâng cao ôn thi vào lớp ...
Bồi Dưỡng HSG Toán Lớp 3
 
kl_HOÀN THIỆN CÔNG TÁC ĐÁNH GIÁ THỰC HIỆN CÔNG VIỆC TẠI CÔNG TY CỔ PHẦN ĐẦU T...
kl_HOÀN THIỆN CÔNG TÁC ĐÁNH GIÁ THỰC HIỆN CÔNG VIỆC TẠI CÔNG TY CỔ PHẦN ĐẦU T...kl_HOÀN THIỆN CÔNG TÁC ĐÁNH GIÁ THỰC HIỆN CÔNG VIỆC TẠI CÔNG TY CỔ PHẦN ĐẦU T...
kl_HOÀN THIỆN CÔNG TÁC ĐÁNH GIÁ THỰC HIỆN CÔNG VIỆC TẠI CÔNG TY CỔ PHẦN ĐẦU T...
Luận Văn Uy Tín
 
tiếng việt dành cho sinh viên ngoại ngữ h
tiếng việt dành cho sinh viên ngoại ngữ htiếng việt dành cho sinh viên ngoại ngữ h
tiếng việt dành cho sinh viên ngoại ngữ h
huynhanhthu082007
 
DANH SÁCH XÉT TUYỂN SỚM_NĂM 2023_học ba DPY.pdf
DANH SÁCH XÉT TUYỂN SỚM_NĂM 2023_học ba DPY.pdfDANH SÁCH XÉT TUYỂN SỚM_NĂM 2023_học ba DPY.pdf
DANH SÁCH XÉT TUYỂN SỚM_NĂM 2023_học ba DPY.pdf
thanhluan21
 
trắc nhiệm ký sinh.docxddddddddddddddddd
trắc nhiệm ký sinh.docxdddddddddddddddddtrắc nhiệm ký sinh.docxddddddddddddddddd
trắc nhiệm ký sinh.docxddddddddddddddddd
my21xn0084
 
Bài tập chương 5. Năng lượng phản ứng.docx
Bài tập chương 5. Năng lượng phản ứng.docxBài tập chương 5. Năng lượng phản ứng.docx
Bài tập chương 5. Năng lượng phản ứng.docx
gorse871
 
[NBV]-CHUYÊN ĐỀ 3. GTLN-GTNN CỦA HÀM SỐ (CÓ ĐÁP ÁN CHI TIẾT).pdf
[NBV]-CHUYÊN ĐỀ 3. GTLN-GTNN CỦA HÀM SỐ (CÓ ĐÁP ÁN CHI TIẾT).pdf[NBV]-CHUYÊN ĐỀ 3. GTLN-GTNN CỦA HÀM SỐ (CÓ ĐÁP ÁN CHI TIẾT).pdf
[NBV]-CHUYÊN ĐỀ 3. GTLN-GTNN CỦA HÀM SỐ (CÓ ĐÁP ÁN CHI TIẾT).pdf
NamNguynHi23
 
Halloween vocabulary for kids in primary school
Halloween vocabulary for kids in primary schoolHalloween vocabulary for kids in primary school
Halloween vocabulary for kids in primary school
AnhPhm265031
 
Biểu tượng trăng và bầu trời trong tác phẩm của Nguyễn Quang Thiều
Biểu tượng trăng và bầu trời trong tác phẩm của Nguyễn Quang ThiềuBiểu tượng trăng và bầu trời trong tác phẩm của Nguyễn Quang Thiều
Biểu tượng trăng và bầu trời trong tác phẩm của Nguyễn Quang Thiều
lamluanvan.net Viết thuê luận văn
 
bài dự thi chính luận 2024 đảng chọn lọc.docx
bài dự thi chính luận 2024 đảng chọn lọc.docxbài dự thi chính luận 2024 đảng chọn lọc.docx
bài dự thi chính luận 2024 đảng chọn lọc.docx
HiYnThTh
 
LUẬN VĂN THẠC SĨ LUẬT - Luận Văn Uy Tín.docx
LUẬN VĂN THẠC SĨ LUẬT - Luận Văn Uy Tín.docxLUẬN VĂN THẠC SĨ LUẬT - Luận Văn Uy Tín.docx
LUẬN VĂN THẠC SĨ LUẬT - Luận Văn Uy Tín.docx
Luận Văn Uy Tín
 
Smartbiz_He thong MES nganh may mac_2024june
Smartbiz_He thong MES nganh may mac_2024juneSmartbiz_He thong MES nganh may mac_2024june
Smartbiz_He thong MES nganh may mac_2024june
SmartBiz
 
Khí huyết và tân dịch - Y học cổ truyền VN
Khí huyết và tân dịch - Y học cổ truyền VNKhí huyết và tân dịch - Y học cổ truyền VN
Khí huyết và tân dịch - Y học cổ truyền VN
ThaiTrinh16
 
Tai-lieu-Boi-Duong-HSG-môn-Ngữ-Văn-THPT-Tập-1.docx
Tai-lieu-Boi-Duong-HSG-môn-Ngữ-Văn-THPT-Tập-1.docxTai-lieu-Boi-Duong-HSG-môn-Ngữ-Văn-THPT-Tập-1.docx
Tai-lieu-Boi-Duong-HSG-môn-Ngữ-Văn-THPT-Tập-1.docx
NhNguynTQunh
 
BÁO CÁO CUỐI KỲ PHÂN TÍCH THIẾT KẾ HƯỚNG ĐỐI TƯỢNG - NHÓM 7.docx
BÁO CÁO CUỐI KỲ PHÂN TÍCH THIẾT KẾ HƯỚNG ĐỐI TƯỢNG - NHÓM 7.docxBÁO CÁO CUỐI KỲ PHÂN TÍCH THIẾT KẾ HƯỚNG ĐỐI TƯỢNG - NHÓM 7.docx
BÁO CÁO CUỐI KỲ PHÂN TÍCH THIẾT KẾ HƯỚNG ĐỐI TƯỢNG - NHÓM 7.docx
HngL891608
 
THONG BAO nop ho so xet tuyen TS6 24-25.pdf
THONG BAO nop ho so xet tuyen TS6 24-25.pdfTHONG BAO nop ho so xet tuyen TS6 24-25.pdf
THONG BAO nop ho so xet tuyen TS6 24-25.pdf
QucHHunhnh
 
Tóm tắt Tư tưởng Hồ Chí Minhhhhhhhhhhhhh
Tóm tắt Tư tưởng Hồ Chí MinhhhhhhhhhhhhhTóm tắt Tư tưởng Hồ Chí Minhhhhhhhhhhhhh
Tóm tắt Tư tưởng Hồ Chí Minhhhhhhhhhhhhh
nnguyenthao204
 

Recently uploaded (20)

Từ ngữ về con người và chiến tranh trong Nhật ký Đặng Thùy Trâm.pdf
Từ ngữ về con người và chiến tranh trong Nhật ký Đặng Thùy Trâm.pdfTừ ngữ về con người và chiến tranh trong Nhật ký Đặng Thùy Trâm.pdf
Từ ngữ về con người và chiến tranh trong Nhật ký Đặng Thùy Trâm.pdf
 
Các bình diện Ngôn ngữ học đối chiếu.pdf
Các bình diện Ngôn ngữ học đối chiếu.pdfCác bình diện Ngôn ngữ học đối chiếu.pdf
Các bình diện Ngôn ngữ học đối chiếu.pdf
 
YHocData.com-bộ-câu-hỏi-mô-phôi.pdf đầy đủ
YHocData.com-bộ-câu-hỏi-mô-phôi.pdf đầy đủYHocData.com-bộ-câu-hỏi-mô-phôi.pdf đầy đủ
YHocData.com-bộ-câu-hỏi-mô-phôi.pdf đầy đủ
 
Tuyển tập 9 chuyên đề bồi dưỡng Toán lớp 5 cơ bản và nâng cao ôn thi vào lớp ...
Tuyển tập 9 chuyên đề bồi dưỡng Toán lớp 5 cơ bản và nâng cao ôn thi vào lớp ...Tuyển tập 9 chuyên đề bồi dưỡng Toán lớp 5 cơ bản và nâng cao ôn thi vào lớp ...
Tuyển tập 9 chuyên đề bồi dưỡng Toán lớp 5 cơ bản và nâng cao ôn thi vào lớp ...
 
kl_HOÀN THIỆN CÔNG TÁC ĐÁNH GIÁ THỰC HIỆN CÔNG VIỆC TẠI CÔNG TY CỔ PHẦN ĐẦU T...
kl_HOÀN THIỆN CÔNG TÁC ĐÁNH GIÁ THỰC HIỆN CÔNG VIỆC TẠI CÔNG TY CỔ PHẦN ĐẦU T...kl_HOÀN THIỆN CÔNG TÁC ĐÁNH GIÁ THỰC HIỆN CÔNG VIỆC TẠI CÔNG TY CỔ PHẦN ĐẦU T...
kl_HOÀN THIỆN CÔNG TÁC ĐÁNH GIÁ THỰC HIỆN CÔNG VIỆC TẠI CÔNG TY CỔ PHẦN ĐẦU T...
 
tiếng việt dành cho sinh viên ngoại ngữ h
tiếng việt dành cho sinh viên ngoại ngữ htiếng việt dành cho sinh viên ngoại ngữ h
tiếng việt dành cho sinh viên ngoại ngữ h
 
DANH SÁCH XÉT TUYỂN SỚM_NĂM 2023_học ba DPY.pdf
DANH SÁCH XÉT TUYỂN SỚM_NĂM 2023_học ba DPY.pdfDANH SÁCH XÉT TUYỂN SỚM_NĂM 2023_học ba DPY.pdf
DANH SÁCH XÉT TUYỂN SỚM_NĂM 2023_học ba DPY.pdf
 
trắc nhiệm ký sinh.docxddddddddddddddddd
trắc nhiệm ký sinh.docxdddddddddddddddddtrắc nhiệm ký sinh.docxddddddddddddddddd
trắc nhiệm ký sinh.docxddddddddddddddddd
 
Bài tập chương 5. Năng lượng phản ứng.docx
Bài tập chương 5. Năng lượng phản ứng.docxBài tập chương 5. Năng lượng phản ứng.docx
Bài tập chương 5. Năng lượng phản ứng.docx
 
[NBV]-CHUYÊN ĐỀ 3. GTLN-GTNN CỦA HÀM SỐ (CÓ ĐÁP ÁN CHI TIẾT).pdf
[NBV]-CHUYÊN ĐỀ 3. GTLN-GTNN CỦA HÀM SỐ (CÓ ĐÁP ÁN CHI TIẾT).pdf[NBV]-CHUYÊN ĐỀ 3. GTLN-GTNN CỦA HÀM SỐ (CÓ ĐÁP ÁN CHI TIẾT).pdf
[NBV]-CHUYÊN ĐỀ 3. GTLN-GTNN CỦA HÀM SỐ (CÓ ĐÁP ÁN CHI TIẾT).pdf
 
Halloween vocabulary for kids in primary school
Halloween vocabulary for kids in primary schoolHalloween vocabulary for kids in primary school
Halloween vocabulary for kids in primary school
 
Biểu tượng trăng và bầu trời trong tác phẩm của Nguyễn Quang Thiều
Biểu tượng trăng và bầu trời trong tác phẩm của Nguyễn Quang ThiềuBiểu tượng trăng và bầu trời trong tác phẩm của Nguyễn Quang Thiều
Biểu tượng trăng và bầu trời trong tác phẩm của Nguyễn Quang Thiều
 
bài dự thi chính luận 2024 đảng chọn lọc.docx
bài dự thi chính luận 2024 đảng chọn lọc.docxbài dự thi chính luận 2024 đảng chọn lọc.docx
bài dự thi chính luận 2024 đảng chọn lọc.docx
 
LUẬN VĂN THẠC SĨ LUẬT - Luận Văn Uy Tín.docx
LUẬN VĂN THẠC SĨ LUẬT - Luận Văn Uy Tín.docxLUẬN VĂN THẠC SĨ LUẬT - Luận Văn Uy Tín.docx
LUẬN VĂN THẠC SĨ LUẬT - Luận Văn Uy Tín.docx
 
Smartbiz_He thong MES nganh may mac_2024june
Smartbiz_He thong MES nganh may mac_2024juneSmartbiz_He thong MES nganh may mac_2024june
Smartbiz_He thong MES nganh may mac_2024june
 
Khí huyết và tân dịch - Y học cổ truyền VN
Khí huyết và tân dịch - Y học cổ truyền VNKhí huyết và tân dịch - Y học cổ truyền VN
Khí huyết và tân dịch - Y học cổ truyền VN
 
Tai-lieu-Boi-Duong-HSG-môn-Ngữ-Văn-THPT-Tập-1.docx
Tai-lieu-Boi-Duong-HSG-môn-Ngữ-Văn-THPT-Tập-1.docxTai-lieu-Boi-Duong-HSG-môn-Ngữ-Văn-THPT-Tập-1.docx
Tai-lieu-Boi-Duong-HSG-môn-Ngữ-Văn-THPT-Tập-1.docx
 
BÁO CÁO CUỐI KỲ PHÂN TÍCH THIẾT KẾ HƯỚNG ĐỐI TƯỢNG - NHÓM 7.docx
BÁO CÁO CUỐI KỲ PHÂN TÍCH THIẾT KẾ HƯỚNG ĐỐI TƯỢNG - NHÓM 7.docxBÁO CÁO CUỐI KỲ PHÂN TÍCH THIẾT KẾ HƯỚNG ĐỐI TƯỢNG - NHÓM 7.docx
BÁO CÁO CUỐI KỲ PHÂN TÍCH THIẾT KẾ HƯỚNG ĐỐI TƯỢNG - NHÓM 7.docx
 
THONG BAO nop ho so xet tuyen TS6 24-25.pdf
THONG BAO nop ho so xet tuyen TS6 24-25.pdfTHONG BAO nop ho so xet tuyen TS6 24-25.pdf
THONG BAO nop ho so xet tuyen TS6 24-25.pdf
 
Tóm tắt Tư tưởng Hồ Chí Minhhhhhhhhhhhhh
Tóm tắt Tư tưởng Hồ Chí MinhhhhhhhhhhhhhTóm tắt Tư tưởng Hồ Chí Minhhhhhhhhhhhhh
Tóm tắt Tư tưởng Hồ Chí Minhhhhhhhhhhhhh
 

DE PHAT TRIEN THEO CAU TRUC DE MINH HOA 2022 MON TIENG ANH DE 11 15.pdf

  • 1. Question 1: It’s an extremely __________ cure for a headache. A. effect B. effective C. effectual D. effector A. effect /i’fekt/ (n): tác động, ảnh hưởng B. effective /i’fektiv/ (a): hiệu quả C. effectual /i’fekt∫ʊəl/ (a): có hiệu lực, thành công D. effector/i’fekt/ (n): người thực hành Căn cứ vào trạng từ “extremely’’ đứng trước và danh từ theo sau thì vị trí còn trống cần một tính từ.
  • 2. Question 2: He needs to do the homework before going out with his friends, ____? A. doesn’t he B. does he C. needn’t he D. need he Dịch nghĩa: “Anh ấy cần làm bài tập về nhà trước khi đi chơi với bạn, phải không?” Mệnh đề giới thiệu khẳng định, phần hỏi đuôi phủ định. Ở hiện tại đơn với động từ thường mượn trợ động từ do hoặc does tùy theo chủ ngữ. Câu bắt đầu với He needs thì thành lập hỏi đuôi sẽ dùng doesn’t he.
  • 3. Question 3: We thought the repairs on the car would cost about $500, but our estimate was way off the it was $4000. A. mark B. point C. spot D. trace Kiến thức về thành ngữ Off the mark: không chính xác, sai, ngoài dự đoán, vượt quá dự tính Tạm dịch: Chúng tôi nghĩ rằng việc sửa chữa xe sẽ tốn khoảng 500 đô la, nhưng dự tính của chúng tôi đã sai- nó là $ 4000.
  • 4. Question 4. Vietnam has played_________high spirits and had an impressive 2-0 victory over Yemen. A.At B.in C. on D. with Hướng dẫn: Trước vị trí cần điền là động từ has played (chơi), sau vị trí cần điền là cụm danh từ high spirits (tinh thần quyết liệt) nên ta cần một giới từ để diễn tả việc thực hiện hành động gì với thái độ như thế nào.
  • 5. Question 5. All the applicants for the post are thoroughly ________ for their suitability. A. searched B. vetted C. investigated D. scrutinized Kiến thức về từ vựng A. search /sɜːrtʃ/ (v): tìm kiếm, lục soát B. vet /vet/ (v): xem xét kĩ lưỡng (lý lịch, chuyên môn.. của ai) C. investigate /ɪnˈvestɪɡeɪt/ (v): điều tra D. scrutinize /ˈskruːtənaɪz/ (v): nhìn chăm chú, nghiên cứu cẩn thận Tạm dịch: Tất cả các ứng viên cho vị trí đó được xem xét sự phù hợp một cách kĩ lưỡng
  • 6. Question 6. Solar energy is not widely used________it is friendly to the environment. A.Since B.although C. in spite of D. because of Hướng dẫn: Dựa vào nghĩa của câu, ta có vế thứ nhất “Solar energy is not widely used” (Năng lượng mặt trời không được sử dụng rộng rãi) và vế thứ hai “it is friendly to the environment” (nó thân thiện với môi trường) mang nghĩa tương phản nhau. Dịch nghĩa của các phương án, ta có phương án A và D cùng có nghĩa là “bởi vì” chỉ nguyên nhân nên không phù hợp để điền vào chỗ trống. Phương án B và C cùng có nghĩa là “mặc dù” chỉ sự tương phản. Tuy nhiên, xét về cấu trúc thì “although” đi với mệnh đề còn “in spite of’ đi với danh từ hoặc cụm danh từ.
  • 7. Question 7 : Peter’s wife gave him a (n) _____ bike as a birthday present last week. A. blue Japanese cheap B. cheap Japanese blue C. Japanese cheap blue D. cheap blue Japanese *Theo quy tắc trật tự tính từ trong câu: OSASCOMP : cheap - Opinion; blue - C; Japanese - Origin Tạm dịch: Vợ của Peter tặng anh ấy chiếc xe đạp Nhật màu xanh rẻ tiền như một món quà sinh nhật tuần trước.
  • 8. Question 8: You shouldn’t lose heart; successoften comes to those who are not _________ by failures. A. left out B. put off C. switched off D. turned on Kiến thức về cụm động từ Xét các đáp án: A. leave out sb/sth (phr.v): để sót, thiếu ai/cái gì B. put sb off (phr.v): làm ai nhụt chí, thất vọng; ngăn cản ai đó làm điều gì C. switch off (phr.v): ngừng dành sự chú ý của bản thân với ai/cái gì D. turn on sb (phr.v): tấn công, chỉ trích ai đó bất ngờ Tạm dịch: Bạn không nên bỏ cuộc, thành công thường chỉ đến với những người không nhụt chí bởi những thất bại. Cấu trúc khác cần lưu ý: Lose heart (coll): mất niềm tin vào sự thành công của bản thân, bỏ cuộc
  • 9. Question 9: The students_______ the topic when the bell rang. A. discuss B. were discussing C. have discussed D. are discussing Giải thích: + Sự việc đang diễn ra chia ở thì quá khứ tiếp diễn: S + was/were + V.ing => was watching Tạm dịch: Học sinh đang thảo luận chủ đề này thì chuông reo.
  • 10. Question 10: _______, he will go out with his friends. A. When Tony finished his project B. When Tony finishes his project C. When Tony had finished his project D. When Tony was finishing his project Kiến thức: Mệnh đề chỉ thời gian / Sự hòa hợp thì trong mệnh đề Giải thích: Mệnh đề chính chia thì tương lai => động từ trong mệnh đề chỉ thời gian chia thì hiện tại (hiện tại đơn, hiện tại hoàn thành). Loại luôn được các phương án A, C, D do sai thì. Tạm dịch: khi Tony hoàn thành dự án, anh ấy sẽ đi chơi cùng bạn bè.
  • 11. Question 11: My mother doesn't eye to eye with my father sometimes. A. see B. glance C. look D. agree Kiến thức về cụm từ cố định See eye to eye with some one: đồng quan điểm với ai. Tạm dịch: Mẹ tôi thỉnh thoảng không đồng tình với cha tôi. Question 12: It is not always easy to_______ a decision at the last minute. A. do B. make C. buy D. marry Kiến thức về cụm từ cố định Make decision: quyết định. Tạm dịch: Không phải lúc nào việc đưa ra quyết định đúng đắn vào phút cuối cùng cũng dễ dàng.
  • 12. Question 13. I demand to know how this vase __________, and no one is leaving till I find out. A. got broken B. was breaking C. has broken D. is broke Kiến thức: Câu bị động. Giải thích: Ngoài cấu trúc bị động quen thuộc là be + P2 thì ta còn có cách diễn đạt bị động khác nữa với cấu trúc: Get + P2. Loại B và C là 2 phương án viết ở chủ động, loại D do cách thể hiện “is broke” sai ngữ pháp. Chọn A. Tạm dịch: Tôi yêu cầu được biết làm sao mà cái lọ hoa này lại bị vỡ, và sẽ không ai rời đi cho đến khi nào tôi tìm ra.
  • 13. Question 14. ________ hard all day, I was exhausted. A. To work B. Working C. Having worked D. worked Kiến thức: Mệnh đề phân tử / Rút gọn mệnh đề đồng ngữ Giải thích: Khi 2 mệnh đề có cùng chủ ngữ (I ) thì có thể rút gọn 1 trong 2 mệnh đề về dạng: - V-ing / Having P2: nếu mệnh đề được rút gọn mang nghĩa chủ động - P2 (quá khứ phân từ): nếu mệnh đề được rút gọn mang nghĩa bị động Chủ ngữ “I ” có thể làm chủ (tự thực hiện hành động “work => nghĩa chủ động. Tạm dịch:Làm việc vất vả cả ngày, tôi đã kiệt sức
  • 14. Question 15: The older you are, _______________ you may become. A. the more worrrying B. the more worried C. the more worry D. worried Dịch nghĩa: Càng lớn tuổi, bạn càng lo lắng. Giải thích: *Cấu trúc so sánh kép: “The + more adj(dài)/adj_er(ngắn) + S + V, the + more adj(dài)/adj_er(ngắn) + S + V” (càng……..càng….)
  • 15. Question 16. Cathy and Graham are at a restaurant. Cathy: “Oh, $400. I will pay this bill.” Graham: “_______” A. Do you have any cash? B. I pay by credit, please C. No, it’s on me D. We’d better leave off Giải thích: Kiến thức về giao tiếp Cathy và Graham đang ở một nhà hàng. Cathy: “Ồ, 400 đô la. Tôi sẽ thanh toán hóa đơn này ”. Graham: “_____________” A. Bạn có tiền mặt không? B. Tôi thanh toán bằng tín dụng C. Không, cứ để tôi D. Tốt hơn chúng ta nên rời đi → Chọn đáp án C
  • 16. Question 17. Mark and Tim are talking about British literature. Mark: “I immersed myself in the book on British literature we had borrowed from the library.” Tim: “_______” A. Shall we get started now? B. Is it called “The Pickwick Papers”? C. Really? I assumed you’re quite good at it D. I read a book for ages Giải thích: Kiến thức về giao tiếp : Mark và Tim đang nói về văn học Anh. Mark: “Tôi đắm chìm trong cuốn sách về văn học Anh mà chúng ta đã mượn từ thư viện.” Tim: “_____________” A. Chúng ta sẽ bắt đầu ngay bây giờ chứ? B. Nó có tên là “The Pickwick Papers” đúng không? C. Thật không? Tôi cho rằng bạn khá giỏi về nó D. Tôi đọc một cuốn sách lâu lắm rồi
  • 17. Question 18. A. scholar B. honor C. motto D. fortune Question 19. A. borrowed B. conserved C. approached D. complained Question 18 Giải thích: Kiến thức về phát âm A. scholar /ˈskɒlə(r)/ B. honor /ˈɒnə(r)/ C. motto /ˈmɒtəʊ/ D. fortune /ˈfɔːtʃuːn/ Đáp án D, âm o đọc là /ɔː/, còn lại đọc là /ɒ/ Question 19. Đáp án C đúng vì phần gạch chân của đáp án C đọc là âm /t/ còn phần gạch chân của các đáp án còn lại được đọc là âm /d/ A. borrowed /ˈbɒrəʊd/ (v): mượn, vay B. conserved /kənˈsɜːvd/ (v): bảo tồn, giữ gìn C. approached /əˈprəʊtʃt/ (v): tiếp cận, đến gần D. complained /kəmˈpleɪnd/ (v): phàn nàn, than phiền
  • 18. Question 20. A. continue B. recognise C. uncover D. remember Question 21. A. leaflet B. model C. export D. cartoon Question 20: Giải thích: Kiến thức về trọng âm A. continue /kənˈtɪn.juː/ B. recognise /ˈrek.əɡ.naɪz/ C. uncover /ʌnˈkʌv.ɚ/ D. remember /rɪˈmem.bɚ/ → Đáp án B trọng âm rơi vào âm tiết thứ nhất, các đáp án còn lại trọng âm rơi vào âm tiết thứ hai Q21Giải thích: Kiến thức về trọng âm A. leaflet /ˈliː.flət/ B. model /ˈmɑː.dəl/ C. export /ˈek.spɔːrt/ D. cartoon /kɑːrˈtuːn/ → Đáp án D trọng âm rơi vào âm tiết thứ hai, các đáp án còn lại trọng âm rơi vào âm tiết thứ nhất
  • 19. Question 22. She is a down-to-earth woman with no pretensions. A. ambitious B. creative C. idealistic D. practical Question 23. For environment safety, we need to find ways to reduce emission of fumes and smoke of factory. A. leak B. release C. poison D. pollutant 22. Kiến thức về từ đồng nghĩa : Down-to-earth: thực tế A. ambitious (a): tham vọng B. creative (a): sáng tạo C. idealistic (a): duy tâm D. practical (a): thực tế down-to-earth = practical Dịch: Cô ấy là một người phụ nữ bình thường và không có ảo tưởng. 23. Kiến thức về từ đồng nghĩa : Emission (n): khí thải, sự bốc ra, toả ra A. leak (n): lỗ rò, khe hở B. release (n): sự tha, xả C. poison (n): chất độc D. pollutant (n): chất ô nhiễm → emission = release Dịch: Để đảm bảo an toàn môi trường, chúng ta cần tìm cách giảm
  • 20. Question 24. Poverty in many African countries increases the likelihood that people poach animals to earn their living. A. chance B. prospect C. possibility D. improbability Kiến thức về từ trái nghĩa Likelihood (n): khả năng, có khả năng A. chance (n): cơ hội B. prospect (n): triển vọng C. possibility (n): khả năng D. improbability (n): sự không có khả năng xảy ra → likelihood >< improbability Dịch: Nghèo đói ở nhiều nước châu Phi làm tăng khả năng người dân săn trộm động vật để kiếm sống.
  • 21. Question 25. I don't believe in anything he says, he is unreliable. A. inaccurate B. unstable C. trustworthyD. irresponsible Kiến thức về từ trái nghĩa Unreliable (a): không thể tin tưởng được A. inaccurate (a): không chính xác B. unstable (a): không ổn định C. trustworthy(a): đáng tin D. irresponsible (a): không có trách nhiệm → unreliable >< trustworthy Dịch: Tôi không tin vào bất cứ điều gì anh ấy nói, anh ấy không đáng tin cậy.
  • 22. Question 26: He helped us a lot with the project. We couldn’t continue without him. A.Provided his contribution wouldn’t come, we couldn’t continue with the project. B.But for his contribution, we could have continued with the project. C.Unless we had his contribution, we could continue with the project. D.If he hadn’t contributed positively, we couldn’t have continued with the project. Anh ấy đã giúp đỡ chúng ta rất nhiều trong dự án này. Chúng ta đã không thể tiếp tục nếu không có anh ấy.” Đây là sự việc trong quá khứ nên ta phải dùng câu điều kiện loại 3 để diễn tả sự việc trái trái thực tế trong quá khứ. Provided: miễn là ; But for + N: nếu không có ; -Unless : If ..not: nếu... không Miễn là sự đóng góp của anh ấy sẽ không đến thì chúng tôi không thể tiếp tục dự án. (sai cấu trúc) Nếu không có sự đóng góp của anh ấy thì chúng tôi đã có thể tiếp tục dự án. (sai nghĩa) Nếu chúng tôi không có sự đóng góp của anh ấy thì chúng tôi có thể tiếp tục dự án. (sai nghĩa và cấutrúc)
  • 23. Những cách khác để diễn đạt câu điều kiện: a. Unless = If....not (Trừ phi, nếu...không) E.g: If you don’t study hard, you can’t pass the exam. = Unless you study hard, you can’t pass the exam. b. Suppose / Supposing (giả sử như), in case (trong trường hợp), even if (ngay cả khi, cho dù), as long as, so long as, provided (that), on condition (that) (miễn là, với điều kiện là) có thể thay cho if trong câu điều kiện E.g: Supposing (that) you are wrong, what will you do then? c. Without/ But for: nếu không có E.g: Without water, life wouldn’t exist. = If there were no water, life wouldn’t exist. Nếu anh ấy đã không đóng góp tích cực thì chúng tôi không thể tiếp tục dự án
  • 24. Question27: The storm was so great. Many families had to be evacuated to safer parts of the city. A.Although the storm was not great, many families had to be evacuated to safer parts of the city. B.So great was the storm that many families had to be evacuated to safer parts of the city. C.Many families had to be evacuated to safer parts of the city in spite of the great storm. D.It was so a great storm that many families had to be evacuated to safer parts of the city. “Cơn bão quá mạnh. Nhiều gia đình phải được sơ tán đến những nơi an toàn trong thành phố.” A.Mặc dù cơn bão không mạnh nhưng nhiều gia đình phải được sơ tán đến những nơi an toàn trong thành phố. (sai nghĩa) B.Cơn bão quá mạnh nên nhiều gia đình phải được sơ tán đến những nơi an toàn trong thành phố. C.Nhiều gia đình phải được sơ tán đến những nơi an toàn trong thành phố mặc dù cơn bão mạnh, (sai nghĩa) D.Sai cấu trúc (so a great storm => such a great storm/ so great a storm)
  • 25. Question 28: We have conducted (A) exhausting research (B)into the effects of smartphones on students’ (C) behaviour and their (D) academic performance. Giải thích exhaustive (adj): including everything possible; very thorough or complete: toàn diện hết mọi khía cạnh + exhaustive research: nghiên cứu toàn diện exhausting (adj): làm kiệt sức, mệt nhoài academic performance (n, p): thành tích học tập Do đó: exhausting => exhaustive “Chúng tôi đã tiến hành nghiên cứu toàn diện ảnh hưởng của điện thoại thông minh vào cách cư xử và thành tích học tập của học sinh.”
  • 26. Question 29: (A) On the table (B) is (C) hundreds of books written (D) in English. Question 29 Chọn đáp án B Cấu trúc: - Adv of place + V + S (đảo ngữ trạng từ nơi chốn lên trước để nhấn mạnh) Hundreds of + N-plural + V -plural E.g: Hundreds of people are standing in front of the cinema. Do đó: is => are Question 30: Tim works (A) as a doctor and he (B) earns (C) twice (D)as much as her brother. Cấu trúc: S + V + multiple number (half/ twice/ three times/....) + as + much/many/adj/adv + (N) + as + N/pronoun (so sánh bội so) g: The yellow skirt costs twice as much as the red one. (Cái váy mầu vàng có giá gấp đôi cái váy màu đỏ.) “Tim là một bác sĩ và cậu ấy kiếm được gấp đôi anh trai cậu ấy.” Do đó sai :”her”-his
  • 27. Question 31. “You should have finished your work by noon, Fiona” said Mark. A. Mark warned Fiona against having finished her work by noon. B. Mark encouraged Fiona to finish her work by noon. C. Mark reproached Fiona for not having finished her work by noon. D. Mark scolded Fiona for having finished her work by noon. Giải thích: Câu gốc: Mark nói: “Bạn đã nên hoàn thành công việc của mình vào buổi trưa, Fiona." → cấu trúc "should have done: nên làm gì nhưng đã không làm trong quá khứ" A. Mark cảnh báo Fiona không nên hoàn thành công việc của cô ấy vào buổi trưa. → sai nghĩa B. Mark khuyến khích Fiona hoàn thành công việc của mình vào buổi trưa. → sai nghĩa C. Mark trách móc Fiona vì đã không hoàn thành công việc của cô ấy vào buổi trưa. D. Mark mắng Fiona vì cô ấy đã hoàn thành công việc vào buổi trưa. → sai nghĩa
  • 28. Question 32. It is necessary for the local authority to impose social distancing on this infected area. A. The local authority may impose social distancing on this infected area. B. Social distancing needs imposing on this infected area. C. Social distancing should be imposed on that infected area. D. The local authority can impose social distancing on this infected area. Giải thích: Tạm dịch: Chính quyền địa phương cần áp dụng giãn cách xã hội với khu vực bị nhiễm bệnh này. A. Chính quyền địa phương có thể áp đặt sự giãn cách xã hội đối với khu vực bị nhiễm bệnh này. → Sai ở từ “may” (có thể) # necessary (cần thiết) được đề cập trong bài B. Giãn cách xã hội cần được áp dụng ra đối với khu vực bị nhiễm này. C. Cần áp đặt giãn cách xa xã hội đối với khu vực bị nhiễm bệnh đó. → sai ở từ “that” (That → this) D. Chính quyền địa phương có thể áp dụng giãn cách xã hội đối với khu vực bị nhiễm bệnh này. → khác nghĩa với câu gốc. Need doing sth: cần được làm gì
  • 29. Question 33. I haven’t bought any books for 2 years. A. This is the first time I have ever bought books for 2 years. B. I last bought books for 2 years. C. It’s 2 years since I started to buy books. D. The last time I bought books was 2 years ago. Giải thích: Tạm dịch: Tôi đã không mua bất kỳ cuốn sách nào trong 2 năm. ~ D. Lần cuối cùng tôi mua sách là 2 năm trước. Một số cấu trúc đồng nghĩa: S + last + Ved + O + khoảng thời gian + ago = It’s + khoảng thời gian + since + S + last + Ved = S + haven’t/hasn’t + Ved/PII + O + for + khoảng thời gian = The last time + S + V(ed) + was + khoảng thời gian +ago
  • 30. Question 34. Take exercise: we used to think that the longer we spent on, say, a pleasurable walk in the countryside, the more good it did us. Not anymore. The new (34) _______ is for HIIT – short for High-Intensity Interval Training – (35) _______ just twelve minutes of very intense activity is supposed to be every bit as beneficial as conventional exercise. A. potential B. leisure C. habit D. fad Giải thích: Kiến thức về từ vựng A. potential (n): tiềm năng B. leisure (n): nhàn hạ C. habit (n): thói quen D. fad (n): mốt, sự thịnh hành Căn cứ vào nghĩa ta chọn D Tạm dịch: Lấy ví dụ như tập thể dục: chúng ta từng nghĩ rằng chúng ta càng dành nhiều thời gian cho việc đi bộ chậm rãi ở 1 vùng nông thôn, thì điều đó càng có lợi cho chúng ta. Tuy nhiên bây giờ không còn như vậy nữa. Thứ trào lưu thịnh hành là HIIT - viết tắt của Luyện tập ngắt quãng cường độ cao
  • 31. Question 35. The new (34) _______ is for HIIT – short for High-Intensity Interval Training – (35) _______ just twelve minutes of very intense activity is supposed to be every bit as beneficial as conventional exercise. A. which B. whereby C. whereas D. whom Giải thích: Kiến thức về liên từ A. which: cái mà → không thay thế cho danh từ chỉ vật nào nên sai B. whereby: theo đó, nhờ đó C. whereas: trong khi → không phù hợp về nghĩa D. whom: người mà → không thay thế cho danh từ chỉ người nào nên sai Tạm dịch: Thứ trào lưu thịnh hành là là HIIT - viết tắt của Luyện tập ngắt quãng cường độ cao - theo đó chỉ cần 12 phút hoạt động cường độ cao được cho là có lợi như bài tập thông thường.
  • 32. Question 36. They also maintain that it speeds up metabolism and so makes you burn more calories throughout the day. However, some recent research would appear to (36) ______ these claims. A. engender B. subscribe C. dispute D. cease Giải thích: Kiến thức về từ vựng A. engender (v): gây ra B. subscribe (v): đồng tình, đăng ký C. dispute (v): tranh cãi, tranh luận D. cease (v): chấm dứt Tạm dịch: Họ cũng xác nhận rằng nó tăng tốc độ trao đổi chất và do đó khiến bạn đốt cháy nhiều calo hơn trong suốt cả ngày. Tuy nhiên, một số nghiên cứu gần đây dường như không không tình với những tuyên bố này.
  • 33. Question 37. (37) _______ have our personalities changed, too? Smartphones allow us to access information in no time at all. Research demonstrates that 80% of people will not wait more than 30 seconds for a video to load. A. So B. However C. But D. And Giải thích: Kiến thức về liên từ A. So: vì vậy → không phù hợp về nghĩa B. However: tuy nhiên → sau “however” có dấu phẩy C. But: nhưng D. And: và → không phù hợp về nghĩa Tạm dịch: Nhưng tính cách của chúng ta có thay đổi không? Điện thoại thông minh cho phép chúng ta truy cập thông tin nhanh chóng. Nghiên cứu chứng minh rằng 80% mọi người sẽ không đợi quá 30 giây để tải video.
  • 34. Question 38. Overtime, we come to expect (38) _ to be available infinitely more quickly than in the past. We have far less patience. We’ve forgotten how to slow down. Welcome to modern life. A. each B. everything C. few D. almost Giải thích: Kiến thức về từ vựng A. each: mỗi B. everything: mọi thứ C. few: một vài D. almost: gần như Tạm dịch: Qua thời gian, chúng ta mong đợi mọi thứ sẽ khả dụng nhanh hơn vô hạn so với trước đây. Chúng ta có ít kiên nhẫn hơn. Chúng ta đã quên cách sống chậm lại rồi. Chào mừng bạn đến với cuộc sống hiện đại.
  • 35. Question 39. Which would be the best title for the passage? A. Singapore fixes you up B. The dating life of single Singaporeans C. Singapore’s dating agency D. How to get a partner in Singapore Giải thích: Đâu sẽ là tiêu đề hay nhất cho đoạn văn? A. Singapore làm mai cho bạn B. Cuộc sống hẹn hò của những người Singapore độc thân C. Đại lý hẹn hò của Singapore D. Làm thế nào để có được một người yêu ở Singapore Đoạn văn chủ yếu nói về tình trạng yêu đương kết hôn của những người ở Singapore và việc chính phủ Singapore đang thực hiện một chương trình để giúp mọi người có thể hẹn hò và cưới đúng người
  • 36. Question 40. The phrase “settle down” in paragraph 1 mostly means _______. A. leave a job B. find a job C. get married D. keep calm Giải thích: Cụm từ "settle down" trong đoạn 1 chủ yếu có nghĩa là _________. A. nghỉ việc B. tìm việc làm C. kết hôn D. giữ bình tĩnh → settle down: Bắt đầu cuộc sống ổn định, kết hôn = get married
  • 37. Question41. According to the passage, the main reason why delaying parenthood becomes a norm in developed countries is that _______. A. single individuals focus more on their jobs than on seeking a partner B. they have little time to meet possible partners due to their jobs C. people who have reached the age of 30 are used to being alone D. individuals are not prepared to have a major responsibility Giải thích: Theo đoạn văn, lý do chính khiến việc trì hoãn việc làm cha mẹ trở thành một thứ quá đỗi bình thường ở các nước phát triển là do _________ A. các cá nhân độc thân tập trung nhiều hơn vào công việc của họ hơn là tìm kiếm bạn đời B. họ có ít thời gian để gặp gỡ người bạn đời có thể do công việc của họ C. những người đã bước qua tuổi 30 đã quen với việc cô đơn D. các cá nhân không được chuẩn bị để chịu trách nhiệm chính Căn cứ vào thông tin: Putting off marriage and parenthood is becoming more common in all industrialized nations. This is partly because high pressure jobs leave little time for socializing and meeting potential partners. (Trì hoãn hôn nhân và làm cha mẹ đang trở nên phổ biến hơn ở tất cả các quốc gia công nghiệp hóa. Điều này một phần là do công việc áp lực cao khiến bạn có ít thời gian để giao lưu và gặp gỡ những người có tiềm năng trở thành bạn đời của bạn.)
  • 38. Question42. The word “it” in paragraph 4 refers to _______. A. government B. agency C. Social Development Unit D. graduate Giải thích: Từ "it" trong đoạn 4 đề cập đến _________ A. chính quyền B. cơ quan C. Đơn vị phát triển xã hội D. tốt nghiệp Căn cứ vào ngữ nghĩa của câu: Research scientists Kee-Chuan Goh (29) has never had a girlfriend. “There’s a first time for everything.”, he says. Like Madeline he has signed up with the SDU (Social Development Unit), the government’s very own dating agency. It is open only to graduates, who sign up for five years, although mostmembers are married within three. (Nhà khoa học nghiên cứu Kee-Chuan Goh (29 tuổi) chưa từng có bạn gái. “Đó là lần đầu tiên của mọi thứ.”, Anh ấy nói. Giống như Madeline, anh ấy đã đăng ký với SDU (Đơn vị phát triển xã hội), cơ quan hẹn hò của chính phủ. Nó chỉ mở cửa cho sinh viên tốt nghiệp đăng ký trong năm năm, mặc dù hầu hết các thành viên đã kết hôn trong vòng ba năm.)
  • 39. Question 43. Which of the following is true, according to the passage? Giải thích: Điều nào sau đây là đúng, theo đoạn văn? A. Áp lực kết hôn lớn nhất đến từ cha mẹ của Madeline B. Những người phụ nữ tốt nghiệp đại học trở thành những người vợ được săn đón C. Đàn ông và phụ nữ khó hòa hợp một cách tự nhiên D. Quảng cáo khuyến khích những người trẻ tuổi có những kỳ vọng thực tế hơn Căn cứ vào thông tin: Singapore’s government believes that, to ensure continuousprosperity, future generations must become more intelligent. And it is concerned that female graduates – ideal for breeding this super-race – are staying single and childless. Singapore’smen, on the other hand, are marrying less educated women. So, the government has launcheda campaign to encourage the ‘right’ couples to get together. A sai vì trong bài chỉ nói “And it is not only her family but also her government who are keen for her to marry and reproduce.” – Không chỉ cha mẹ mà cả chin phủ cũng muốn cô ấy kết hôn và sinh con C sai vì trong bài chỉ đề cập “The sexes are not encouraged to mix during childhoodor to date until after university.” – Nam và nữ được khuyến khích là không nên ở cùng nhau trong suốt thời thơ ấu hay hẹn họ cho tới khi tốt nghiệp.” D sai vì đoạn cuối có nói “Government-sponsored ads on prime-time TV aim to get the message across. In one, a young man sits on a park bench, sighing, “Where is my dream girl?” At the other end, a girl stares into space thinking, “Where is the man of my dreams?” They fail to notice each other and wander off, lonely.A voice-over warns “Why not reality? You could wait a lifetime for a dream.” - ……Tạisao không phải là thật chứ? Bạn chỉ có thể chờ đợi 1 giấc mơ cả đời mà thôi” → Quảng cáo tạo cho con người ta cảm giác tìm bạn đời ảo, chứ không hề chân thực.
  • 40. Question 44. Which best serves as the title for the passage? A. The growing disparity between rich and poor B. Seeing valuable time from a crooked angle C. The mad rush to save time D. The slower pace of life: Is it suitable for all? Giải thích: Đâu là tiêu đề hay nhất cho đoạn văn? A. Sự chênh lệch giàu nghèo ngày càng tăng B. Chiêm nghiệm về thời gian quý báu theo cách nhìn hoàn toàn mới (from crooked angle ~ from a completely different perspective) C. Sự vội vã điên cuồng để tiết kiệm thời gian D. Nhịp sống chậm hơn: Liệu có phù hợp cho tất cả Bài viết nói về thời gian và việc con người ngày càng vội vã để tiết kiệm thời gian nên đáp án C là hợp lý
  • 41. Question 45. The word “malaise” in paragraph 1 is closest in meaning to _______. A. uneasinessB. tactic C. morale D. trend Giải thích: Từ “malaise” trong đoạn 1 gần nghĩa nhất với ________ A. bất an B. chiến thuật C. tinh thần D. xu hướng → malaise: bất ổn = uneasiness: bất an
  • 42. Question46. The consequences of not having enough time for family are mentioned in the passage EXCEPT _______. A. Working couples find themselves in a situation of lacking time for each other B. Children find it impossible to pay a visit to their grandparents C. Time to spend with children becomes scarce D. Parents have quarrels over the amount of time allotted for doing household chores Giải thích: Hậu quả của việc không có đủ thời gian cho gia đình được đề cập trong đoạn văn NGOẠI TRỪ __________ A. Các cặp vợ chồng đang đi làm thấy mình trong tình trạng thiếu thời gian dành cho nhau B. Con cái không về thăm ông bà được. C. Thời gian dành cho con cái trở nên khan hiếm D. Cha mẹ cãi nhau về lượng thời gian dành cho việc nhà Căn cứ vào thông tin đoạn 2: The result is parents with a lack of quality time to spend with their children, and surveys showing that working couples see less of each other than ever before and that rows over time spent on domesticlabour or childcare top the list of marital discord.
  • 43. Question 47. The word “this” in paragraph 3 refers to _______. A. answering the phone B. calling someone on the phone C. asking someone to hold the line D. arguing with customers Giải thích: Từ “this” trong đoạn 3 đề cập đến ________ A. trả lời điện thoại B. gọi điện thoại cho ai đó C. yêu cầu ai đó giữ điện thoại D. tranh cãi với khách hàng Căn cứ vào ngữ nghĩa của câu: It is now perfectly acceptable to be asked to hold the instant the phone is answered. This saves the company time and money, but costs you time. We are engaged in a constant, subtle war over time. (Bây giờ hoàn toàn có thể chấp nhận khi được yêu cầu giữ điện thoại ngay sau khi điện thoại được trả lời. Điều này giúp tiết kiệm thời gian và tiền bạc cho công ty, nhưng lại khiến bạn mất thời gian. Chúng ta đang tham gia vào một cuộc chiến liên tục, tinh vi theo thời gian.) ~ Lưu ý: the instant = as soon as
  • 44. Question48. The word “treadmill”in paragraph 5 mostly describes something which is _______. A. energizing B. conducive C. alien D. monotonous Giải thích: Từ “treadmill” trong đoạn 5 chủ yếu mô tả một cái gì đó _________ A. tiếp thêm sinh lực B. có lợi C. người ngoài hành tinh D. đơn điệu, tẻ nhạt → treadmill: guồng quay liên tục = cái gì đó rất tẻ nhạt đơn điệu Căn cứ vào ngữ cảnh trong câu trước nó: At the same time, they are employing others - cleaners, childminders, fast-food restaurant workers - in order to allow themselves to work all the time. Meanwhile, more and more of us are putting ourselves on the treadmill of constant activity, taking on an increasingly heavy workload, and never stopping for a moment to ask ourselves why. (Đồng thời, họ đang thuê những người khác - người dọn dẹp, người trông trẻ, nhân viên nhà hàng thức ăn nhanh - để cho phép bản thân làm việc mọi lúc. Trong khi đó, ngày càng có nhiều người trong chúng ta đặt mình vào guồng quay của hoạt động liên tục, gánh vác khối lượng công việc ngày càng nặng nề, và không bao giờ dừng lại một chút để tự hỏi bản thân tại sao.)
  • 45. Question 49. Which of the following is true, according to the passage? A. People who are idle often experience a sense of willingness B. The rich had more time in the past compared to the poor C. People signing up for time management courses wish to have a free afternoon D. Being late is a symbol of status in the past Giải thích: Điều nào sau đây là đúng, theo đoạn văn? A. Những người nhàn rỗi thường có cảm giác hạnh phúc và sẵn sàng làm mọi thứ B. Người giàu có nhiều thời gian hơn trong quá khứ so với người nghèo C. Những người đăng ký khóa học quản lý thời gian muốn có một buổi chiều rảnh rỗi D. Đến muộn là biểu tượng của địa vị trong quá khứ
  • 46. Căn cứ vào thông tin đoạn 4: Of course, there is a class dimension to the rush culture. One of the biggest transitions of the past few decades has been to take the previous relationship between time and status - the rich had lots of time, the poor very little - and reverse it. (Tất nhiên, có một khía cạnh giai cấp đối với văn hóa của sự vội vàng. Một trong những bước chuyển đổi lớn nhất trong vài thập kỷ qua là lấy mối quan hệ trước đây giữa thời gian và địa vị - người giàu có rất nhiều thời gian, người nghèo rất ít - và đảo ngược nó.) A sai vì “The idea of doing nothing has become terrifying, a sure sign of worthlessness.” – Cái việc ăn không ngồi rồi trở nên vô cùng đáng sợ, 1 dấu hiệu của sự vô dụng” → Lưu ý phép paragraphase: idle (rảnh rỗi ) ~ do nothing C sai vì “We yearn for the lazy afternoons and days of yesteryear - but enthusiastically sign up for email, messaging services, language classes. Even time managementcourses.” – “Chúng tôi mong muốn có được những buổi chiều thảnh thơi, nhưng vẫn cứ hừng hực đăng ký email, dịch vụ tin nhắn, các lớp học ngôn ngữ, hay thậm chí là cả những khóa học quản lý thời gian nữa” D sai vì “Moreover, to be seen to have time to spare is a sign of low status: arranging lunch, it is never done to be available too soon. Similarly, being late is moving from a sign of rudeness to a sign of status.” – “……. Tương tự như vậy, đến muộn từ dần chuyển từ trạng thái bất lịch sự sang dấu hiệu của người có địa vị
  • 47. Question 50. Which of the following can be inferred from the passage? A. People living in an instant society tend to multitask B. Advertising led to a larger number of people who opt to give up their free time C. There is a strong positive correlation between time and status D. Manual workers are required to work against the clock Giải thích: Điều nào sau đây có thể được suy ra từ đoạn văn? A. Những người sống trong một xã hội cấp bách có xu hướng làm nhiều nhiệm vụ B. Quảng cáo đã dẫn đến một số lượng lớn hơn những người chọn từ bỏ thời gian rảnh rỗi của họ C. Có mối tương quan thuận chặt chẽ giữa thời gian và địa vị D. Người lao động chân tay được yêu cầu làm chạy đua với thời gian
  • 48. Căn cứ vào các thông tin: Symptoms include jabbingthe 'door close' button on lift doors to save the two to four seconds required for the door to do it on its own, and an inabilityto do one thing at a time, so that every journey is a phone call opportunity. (Các triệu chứng bao gồm ấn nút 'đóng cửa' trên cửa thang máy để tiết kiệm 2-4 giây cần thiết để cửa tự làm việc đó và việc không thể làm 1 thứ 1 lúc được ………” → Nghĩa là con người ta không thể làm 1 thứ 1 lúc, mà phải làm nhiều thứ 1 lúc để còn tiết kiệm thời gian → multitask. B sai vì trong bài chỉ nói “Adverts for energy-boosting drinks read: 'Having trouble keeping up with yourself?'” – Các quảng cáo đồ uống tăng lực với khẩu hiệu “Không thể chạy kịp bản thân ư?” → Điều đó chưa thể kết luận quảng cáo khiến con người ta từ bỏ thời gian rảnh được. C sai vì trong bài chỉ nói “Similarly,being late is moving from a sign of rudeness to a sign of status.” - Tương tự như vậy, đến muộn từ dần chuyển từ trạng thái bất lịch sự sang dấu hiệu của người có địa vị rồi D sai vì “The rich are working longer and longer hours in order to compete with each other. At the same time, they are employingothers - cleaners, childminders,fast- food restaurant workers - in order to allow themselves to work all the time.” – “Người giàu giờ càng làm nhiều hơn để muốn cạnh tranh với nhau. Cùng lúc đó, họ thuê những người lau dọn, vú nuôi, nhân viên tại các cửa hàng thức ăn nhanh, để cho phép bản thân mình làm việc mọi lúc” → Chính người giàu mới làm việc chạy đua với thời gian, chứ không đề cập là nhân viên lao động chân tay. → Chọn đáp án A
  • 49. Question 1: The children slept soundly because it was so _________ and quiet in the garden. A. peaceable B. peacefulness C. peaceably D. peaceful A. peaceable /’pi:səbl/ (a): thích sống hòa bình B. peacefulness /’pi:sflnis/ (n): sự yên bình C. peaceably /’pi:səbli/ (adv): một cách yên bình D. peaceful /’pi:sfl/ (a): yên bình, hòa bình Căn cứ vào cấu trúc câu (S + to be + Adj…) và cấu trúc song song vì vậy vị trí ô trống cần điền là một tính từ. Dịch nghĩa: Những đứa trẻ đã ngủ rất ngon bởi vì ở ngoài vườn thật yên bình và yên lặng.
  • 50. Question 2: Donald John Trump is the 45th President of the United States, _______? A. is heB. Is not he C. isn’t it D. isn’t he Dịch nghĩa: “Donald John Trump là Tổng thống thứ 45 của Hoa Kỳ phải không?” Câu bắt đầu với S + is thì thành lập hỏi đuôi sẽ dùng isn’t + S.
  • 51. Question 3: Mr Nam knows Hanoi City like the back of his . He used to be a taxi driver there for 10 years. A. head B. mind C. hand D. life head (n): đầu mind (n): trí óc, tâm trí hand (n): tay life (n): cuộc sống + know sb/ sth like the back of one’s hand ~ to be very familiar with somebody/something:hiểu biết rất rõ, biết rõ như lòng bàn tay Dịch: Ông Nam biết thành phổ Hà Nội rõ như lòng bàn tay. Ông ấy từng lái xe taxi ở đó 10 năm.
  • 52. Question 4: Bill Gates gave away $4.6 billion in Microsoft shares ______June to his personal charity, the Bill & Melinda Gates Foundation. A. on B. for C. in D. at Dịch nghĩa: Bill Gates đã tặng 4,6 tỷ đô la cổ phiếu Microsoft vào tháng 6 cho tổ chức từ thiện cá nhân của mình, Quỹ Bill & Melinda Gates. Căn cứ vào cấu trúc: -in+ month.
  • 53. Question 5: Whenever a camera was pointed at her, Marilyn would instantly ________herself into a radiant star. A. transmit B. transform C. convert D. transfer A. transfer / trænz mit / (v): truyền tài (thông tin, tín hiệu) B. biến đổi / træns fo rm / (v): chuyển đối đối, biển đổi (hinh dang cái gi hay ai dó thành một cái / ngurời hoàn toàn khác) C. convert / ken v3 t / (v): chuyển đối. change (from the position / this status to vi tri / khác trang hoặc đơn vị đo lường, vị trí tiên phong) chuyển nhượng lại (tài sản, thu lợi) cho ai D. transfer / træns fa r / (v ): chuyển đổi (từ nơi này sang nơi khác) Tạm dịch: Bát cử khi nào máy ảnh chia vào có, Marilyn sẽ lập tức biển minh thành một ngôi sao rang.
  • 54. Question 6. ________ she was the most prominent candidate, she was not chosen. A. Though B. Because of C. As D. Since Kiến thức: Liên từ Giải thích: A. Though : Mặc dù B. Because of: ( theo sau cụm danh từ)Vì C. As: Vì D. Since: Vì, Từ khi Dịch : Mặc dù cô ấy là ứng cử viên nổi bật nhất, cô ấy đã không được chọn.
  • 55. Question 7. She has just bought a bag. A. small plastic blue B. blue small plastic C. plastic small blue D. small blue plastic. Trật tự của tính từ trong câu này là: small (size) - blue (color) - plastic (material) Dịch: Anh ấy vừa mới mua một cái túi nhỏ màu xanh da trời bằng nhựa
  • 56. Question 8: Although she is my sister, I find it hard to her selfishness. A. put up with B. catch up with C. keep up with D. come down with put up with: chịu đựng E.g: I cannot put up with him any more. catch up with (sb): đuổi kịp (ai) E.g: Go on ahead. I’ll catch up with you. keep up with: theo kịp, bắt kịp E.g: Technology changes so fast it’s hard to keep up with it. come down with: mắc bệnh, ốm (nhẹ) E.g: I came down with flu last week.
  • 57. Question 9. When he came, I_______ in the sitting room. A. readB. am reading C. has read D. was reading Kiến thức: Sự phối hợp về thì của động từ Giải thích: Thì quá khứ đơn và quá khứ tiếp diễn kết hợp trong câu: Diễn tả hành động đang xảy ra trong quá khứ thì một hành động khác xen vào, hành động đang xảy ra chi thì quá khứ tiếp diễn, hành động xen vào chia thì quá khứ đơn Cấu trúc: S + was/ were + V_ing + when + S + Ved/bqt Tạm dịch: Khi anh ấy đến, tôi đang đọc sách ở phòng khách.
  • 58. Question 10. She will call you to confirm _______. A. until she received the parcel B. when she was receiving the parcel C. the moment she receives the parcel D. after she had received the parcel 10.Kiến thức: Mệnh đề chỉ thời gian / Sự hòa hợp thì trong mệnh đề Giải thích: Mệnh đề chính chia thì tương lai => động từ trong mệnh đề chỉ thời gian chia thì hiện tại (hiện tại đơn, hiện tại hoàn thành). Loại luôn được các phương án B, A, D do sai thì. A. cho đến khi cô ấy nhận được bưu kiện B. khi cô ấy đang nhận bưu kiện C. thời điểm cô ấy nhận được bưu kiện D. sau khi cô ấy đã nhận được bưu kiện
  • 59. Question 11: I get quite depressed when I think about the damage we are to the environment. A. having B. taking C. making D. causing (to) cause the damage: gây thiệt hại. Dịch: Tôi rất thất vọng mỗi khi nghĩ về những thiệt hại mà chúng ta đã gây ra cho môi trường
  • 60. Câu 12: : I'm not going to go ice skating! I'd only fall over and...........a fool of myself. A. create B. show C.do D. make Kiến thức về cụm từ cố định make a fool (out) of sb/yourself: khiến ai đó trông như kẻ ngốc Tạm dịch: Tôi sẽ không đi trượt băng đâu! Tôi chỉ bị ngã và khiến mình trông như kẻ ngốc. Kiến thức về cụm từ cố định cause the damage: gây thiệt hại.
  • 61. Question 13. A new supermarket ________ in the city cente now. A. is being built B. being built C. are being built D. be built Câu bị động của hiện tại tiếp diễn S+ is/am/are+ being+Vpp
  • 62. Question 14: _____ a scholarship, she entered one of the most privileged universities of the United Kingdom. A. To award B. Being awarded C. Having awarded D. Having been awarded Kiến thức về rút gọn câu Tạm dịch: Được nhận học bổng, cô ấy vào học tại một trong những trường đại học đặc quyền nhất của Vương quốc Anh. + Hai vế có cùng chủ ngữ là “she”, ta có thể bỏ chủ ngữ vế đầu, đồng thời chuyển động từ về dạng Ving khi nó mang nghĩa chủ động và Vp2/Ved khi nó mang nghĩa bị động. + Vì hành động “được nhận học bổng” xảy ra trước hành động còn lại trong câu nên ta dùng “having been Vp2/Ved”.
  • 63. Question 15: The larger the flat is, the ____________ its price is. A. expensive B. more expensive C. expensively D. most expensive Dịch nghĩa: Căn hộ càng lớn thì giá của nó càng đắt. Xét các đáp án: A. expensive → Đây là cấu trúc so sánh kép nên chỗ trống cần điền từ có dạng so sánh hơn B. more expensive → Cấu trúc so sánh kép khi nói về 2 người hoặc sự vật: The more/-er + S + V, the more/-er + S + V C. expensively → Đây là cấu trúc so sánh kép nên chỗ trống cần điền từ có dạng so sánh hơn D. most expensive → Đây là cấu trúc so sánh kép nên chỗ trống cần điền từ có dạng so sánh hơn
  • 64. Question 16: Marry is talking to Linda over the phone. Mary: “Thank you for helping me prepare for the party.” Linda: “ ”. A. My pleasure B. The meal was out of this world C. Never mention me.D. Of course not Mary đang nói chuyện điện thoại với Linda. Mary: Cảm ơn cậu vì đã giúp tớ chuẩn bị cho bữa tiệc. Đó là niềm hân hạnh của tớ khi giúp được cậu. Bữa ăn thì vô cùng tuyệt vời. Đừng bao giờ nhắc tới tớ. Tất nhiên là không. Ta dùng “You’re welcome”, “My pleasure” để đáp lại lời cảm ơn.
  • 65. Question 17: Tom is in Ho Chi Minh City and asks a passer-by the way to the railway station. Tom: “Can you show me the way to the railway station, please?” - Passer-by: “ ” A. No way. B. Just round the comer over there. C. Look it up in a dictionary! D. There’s no traffic near here. Tom đang ở thành phố Hồ Chí Minh và hỏi người qua đường đường tới nhà ga xe lửa. Không đời nào. Nó chỉ ở quanh góc kia thôi. Hãy tra cứu nó trong cuốn từ điển! Không có giao thông ở gần đây.
  • 66. Question 18. A. legal B. diverse C. polite D. complete Question 19. A. suspicious B. marvelous C. physical D. argument Question 18 Kiến thức: Trọng âm từ có 2 âm tiết Giải thích: legal /ˈliːɡl/ diverse /daɪˈvɜːs/ polite /pəˈlaɪt/complete /kəmˈpliːt/ Câu A trọng âm 1 còn lại trọng âm 2. Question 19. Kiến thức: Trọng âm của từ có 3 âm tiết trở lên Giải thích: suspicious /səˈspɪʃəs/ marvelous /ˈmɑːvələs/ physical /ˈfɪzɪkl/ argument /ˈɑːɡjumənt/ Câu A trọng âm rơi vào âm tiết thứ 2, còn lại là thứ nhất
  • 67. Question 20. A. Lived B. cooked C. laughed D. watched Kiến thức: Phát âm “-ed” Giải thích: Những từ kết thúc bằng các âm: /ʧ/, /s/, /k/, /f/, /p/, /θ/, /∫/ thì „ed‟ sẽ được đọc là /t/. Những từ kết thúc bằng âm /t/ hay /d/ thì „ed‟ sẽ được đọc là /ɪd/. Các trường hợp còn lại, „ed‟ sẽ đọc là /d/. Lived /livd/ cooked /kukt/ laughed /lɑ:ft/watched /wɔtʃ/ Đáp án A có phần gạch chân đọc là /d/, các đáp án còn lại đọc là /t/ Question 21. A. Educate B. engineer C. acceptable D. department Kiến thức: Cách phát âm “e” Giải thích: educate /ˈedʒukeɪt/ engineer /ˌendʒɪˈnɪə(r)/ acceptable /əkˈseptəbl/ department /dɪˈpɑːtmənt/ Âm “e” trong từ “department” phát âm là /ɪ/, trong các từ còn lại phát âm là /e/
  • 68. Question 22: That restaurant tempts many customers because of its affordable price and good food. A. attracts B. frees C. refuses D. offers attract (v): thu hút, hấp dẫn, lôi cuốn free (v): thả, giải thoát, phóng thích . refuse (V): từ chối offer (v): cung cấp tempt (v): cám dỗ, lôi cuốn, quyến rũ Question 23: I’ll back up your ideas if they don’t believe you. A. support B. rebel C. admire D. protect support (v): ủng hộ admire (v): ngưỡng mộ protect (v): bảo vệ back up (ph.v): ủng hộ Dịch: Tôi sẽ ủng hộ ý kiến của bạn nếu họ không tin bạn.
  • 69. Question 24: She had never seen such discourtesy towards the director as it happened in the meeting last week. A. politeness B. rudeness C. measurement D. encouragement politeness (n): sự lịch sự rudeness (n): sự thô lỗ, bất lịch sự, sự khiếm nhã measurement (n): sự đo lường, phép đo encouragement (n): sự khuyến khích, động viên discourtesy (n) ~ impoliteness, rudeness: sự bất lịch sự, sự thô lỗ politeness Dịch: Cô ấy chưa bao giờ nhìn thấy sự bất lịch sự như vậy đối với giám đốc khi điều này đã xảy ra tại cuộc họp vào tuần trước.
  • 70. Question 25: They conducteda campaign to promote people’s awareness of environmental problems. A. encourage B. publicize C. hinder D. strengthen encourage (v): khuyến khích, động viên publicize (v): quảng cáo, đưa ra công khai hinder (v): cản trở, gây trở ngại strengthen(v); tăng cường promote (v) ~ encourage: đẩy mạnh, khuyến khích Dịch: Họ đã thực hiện một chiến lược để tăng cường ý thức của mọi người về các vấn đề môi trường. Mark the letter A, B, C, or D on your answer sheet to indicate the word(s) CLOSEST in meaning to the underlined word(s) in each of the following questions.
  • 71. Question26: He got down to writing the letter as soon as he returned from his walk. A. No sooner had he returned from his walk when he got down to writing the letter. B. Not until he returned from his walk did he get down to writing the letter. C. Only after he had returned from his walk did he get down to writing the letter. D. Hardly had he returned from his walk when he got down to writing the letter. Dịch nghĩa: Anh ta ngồi xuống viết thư ngay khi đi bộ về. Xét các đáp án: A. No sooner had he returned from his walk when he got down to writing the letter. → Cấu trúc đảo ngữ với các cụm từ so sánh về thời gian: No sooner + had + S + P2 + than + S + Ved (nếu dùng no sooner phải có than đằng sau). B. Not until he returned from his walk did he get down to writing the letter. → Mãi cho đến khi anh trở về sau khi đi bộ, anh mới xuống viết thư, sai nghĩa. C. Only after he had returned from his walk did he get down to writing the letter. Chỉ sau khi anh ấy trở về sau khi đi bộ, anh ấy mới viết thư, sai nghĩa D. Hardly had he returned from his walk when he got down to writing the letter. → Cấu trúc đảo ngữ với các cụm từ so sánh về thời gian: Hardly/Scarcely/Barely + had + S + P2 + when + S + Ved: ngay khi/vừa mới….thì
  • 72. Question 27 (VDC): I invitedRachel to my party, but she couldn’t come. She had arranged to do something else. A. Without having arranged to do something else, Rachel would have come to my party as invited. B. If it hadn’t been for her arrangement for something else, Rachel would come to my party as invited. C. Rachel would have come to my party, unless she hadn’t arranged to so something else. D. If Rachel hadn’t arranged to do something else, she would come to my party as invited. Kiến thức: Câu điều kiện loại 3 Giải chi tiết: - Cách dùng: Diễn tả một giả thiết trái ngược với thực tế đã xảy ra ở quá khứ Công thức chung: If + S + had (not) + Ved/ V3, S + would/ could (not) + have + Ved/ V3. - Trong câu điều kiện loại 3, có thể sử dụng “but for” và “without” “had it not been for” thay cho “if” Công thức chung: But for/ Without/ Had it not been for + N/Ving, S + would/ could (not) + have + Ved/ V3. Tạm dịch: Tôi đã mời Rachel đến bữa tiệc của tôi, nhưng cô ấy không thể đến. Cô đã sắp xếp để làm việc khác. = Nếu sắp xếp để làm việc khác, Rachel sẽ đến bữa tiệc của tôi như được mời rồi. B. Sai công thức. “would come” => “would have come” C. Sai công thức. Mệnh đề sau “unless” (nếu … không) không để dạng phủ định D. Sai công thức. “would come” => “would have come”
  • 73. Question 28: I don’t think that he has responsed to my email as I have received nothing. A. don’t think B. received C. as D. responsed Question 29: In order to make progress in her speaking skill, you need to practice everyday. A. in order to B. practice C. make progress D. her speaking skill Question 30: Fifty minutes are the maximum length of time allotted for the exam. A. allotted B. are C. length D. for 28. Kiến thức: Từ vựng * Response (n): sự hồi âm, sự đáp lại * Respond (v): hồi âm, trả lời. Sửa: responsed → responded Tạm dịch: Tôi không nghĩ là anh ấy đã trả lời thư của tôi vì tôi chưa nhận được gì cả. 29. Kiến thức: Cụm từ • make progress: tiến bộ Sửa D. her speaking skill- BỎ HER. Tạm dịch: Để tiến bộ trong kỹ năng nói, bạn cần luyện tập hàng ngày. 30. Kiến thức: Sự phù hợp giữa chủ ngữ và động từ • “Fifty minutes” là một khoảng thời gian, động từ luôn chia số ít Sửa: are => is Tạm dịch: Năm mươi phút là khoảng thời gian tối đa dành cho bài thi.
  • 74. Question 31: It's time for me to start to think about the job I will do in the future. A. I should start to think about the job I will do in the future. B. I may start to think about the job I will do in the future. C. I needn't start to think about the job I will do in the future. D. I must have started to think about the job I will do in the future. Dịch nghĩa: Đã đến lúc tôi bắt đầu nghĩ về công việc tôi sẽ làm trong tương lai. A. Tôi nên bắt đầu nghĩ về công việc tôi sẽ làm trong tương lai. (Đúng) B. Tôi có thể bắt đầu nghĩ về công việc tôi sẽ làm trong tương lai. C. Tôi không cần phải bắt đầu nghĩ về công việc tôi sẽ làm trong tương lai. D. Tôi hẳn là đã phải bắt đầu nghĩ về công việc tôi sẽ làm trong tương lai. => Vì cấu trúc: It’s time for sb to do st (thiên về nghĩa khuyên bảo ) →
  • 75. Question 32: “Buy some meat in the shop”, said my mother to me. A. My mother said to me to buy some meat in the shop. B. My mother told to me to buy some meat in the shop. C. My mother told me to buy some meat in the shop. D. My mother said me to buy some meat in the shop. Dịch nghĩa: “Mua một ít thịt trong cửa hàng nhé!”, mẹ nói với tôi. Xét các đáp án: A. My mother said to me to buy some meat in the shop.→ Cấu trúc: say sth to sb: nói với ai điều gì, không có cấu trúc “say to sb to do sth” B. My mother told to me to buy some meat in the shop. → Cấu trúc: tell sb sth: nói với ai điều gì C. My mother told me to buy some meat in the shop. → Đúng (cấu trúc tell sb to do st: bảo ai làm gì) D. My mother said me to buy some meat in the shop.→ Cấu trúc: say sth to sb: nói với ai điều gì
  • 76. Question 33: I haven't met my grandparents for five years. A. I often met my grandparents five years ago. B. I last met my grandparents five years ago. C. I have met my grandparents for five years. A. I didn't meet my grandparents five years ago. Tạm dịch: “Tôi đã không gặp ông bà tôi trong năm năm.” A. Tôi thường gặp ông bà tôi năm năm trước. (Sai vì nghĩa khác so với câu gốc.) B. Tôi gặp ông bà tôi lần cuối cùng năm năm trước. (Đúng) C. Tôi đã gặp ông bà tôi trong năm năm. (Sai vì khác nghĩa so với câu gốc.) D. Tôi không gặp ông bà tôi cách đây 5 năm. (Sai vì khác nghĩa so với câu gốc.)
  • 77. Question 34 Father’s Day was created to complement Mother’s Day. Like Mother’s Day (34) ________ honors mothers and motherhood, : A. which B. who C. where D. when Kiến thức: Mệnh đề quan hệ * Đại từ quan hệ which dùng để thay thế cho danh từ đứng trước nó, được dùng như chủ ngữ trong mệnh đề quan hệ. Tạm dịch: Giống như Ngày của Mẹ, ngày mà tôn vinh những người mẹ và tình mẫu tử.
  • 78. Question 35 Father’s Day celebrates fatherhood and paternal bonds; it highlights the (35) ________ of fathers in society. A. conquest B. effect C. influence D. impact Kiến thức: Từ vựng A. conquest (n): sự chinh phục B. effect (n): Sự thay đổi hoặc kết quả, hậu quả mà một ai đó/một việc gì đó ảnh hưởng lên người khác/việc khác.( vừa tích cực, vừa tiêu cực) C. influence (n): Sự tác động mạnh mẽ làm thay đổi cách nghĩ, hành động của một người hay sự phát triển của một việc nào đó (không dùng đến vũ lực mà thường là cho họ một tấm gương để noi theo.) D. impact (n): Có ảnh hưởng đáng chú ý lên một người/một việc nào đó, đặc biệt theo chiều hướng xấu. Tạm dịch: nó làm nổi bật ảnh hưởng của những người cha trong xã hội.
  • 79. Question 36 Her father, the Civil War veteran William Jackson Smart, was a single parent who (36) ________ his six children there. A. raised B. realized C. tookD. made Kiến thức: Từ vựng * Raise (v): chăm sóc một người hoặc một động vật hoặc thực vật, cho đến khi chúng đã trưởng thành hoàn toàn. Tạm dịch: Cha của cô, cựu chiến binh Nội chiến William Jackson Smart, là một người cha đơn thân nuôi sáu đứa con của mình ở đó.
  • 80. Question 37 Since then it has become a traditional day (37) ________ year. : A. any B. another C. other D. every Kiến thức: Từ vựng • Every: mỗi, hằng. Tạm dịch: Kể từ đó nó đã trở thành một ngày truyền thống hàng năm.
  • 81. Question 38 In recognition of what fathers do for their families, on this day people may have a party celebrating male parenting or simply make a phone call or send a greeting card. (38) ________, schools help children prepare handmade gifts for their fathers many days before the celebration. A. Besides B. However C. In contrast D. Then Kiến thức: Từ vựng A. Besides: bên cạnh đó (thêm thông tin) B. However: tuy nhiên (chỉ nhượng bộ) C. In contrast: trái lại (chỉ tương phản) D. Then: vậy thì (chỉ kết quả) Tạm dịch: Bên cạnh đó, các trường học giúp các em chuẩn bị những món quà tự tay làm để tặng cha nhiều ngày trước lễ kỷ niệm.
  • 82. Question 39: What does the passage mainly discuss? A.The role of instinct in animalbehavior B.Observations that suggest consciousness in animal behavior C.The use of food in studies of animal behavior D.Differences between the behavior of animals in their natural environments and in laboratoryexperiments. Đoạn văn chủ yếu thảo luận về điều gì? Vai trò của bản năng trong hành vi động vật Những quan sát cho thấy ý thức trong hành vi động vật Việc sử dụng thức ăn trong các nghiên cứu về hành vi động vật Những sự khác nhau giữa hành vi của động vật trong môi trưởng tự nhiên và trong phòng thí nghiệm. Dẫn chứng: - Some animalbehavioristsargue that certain animals can remember past events ... These scientists, however, are cautiousabout the extent to which animalscan be credited with consciousprocessing. (Một số nhà nghiên cứu hành vi của động vật tranh luận rằng một số động vật có thể nhớ các sự việc trong quá khứ... Tuy nhiên, các nhà khoa này thận trọng về mức độ mà động vật có thể được cho là xử lý có ý thức) Explanationsof animalbehaviorthat leave out any sort of consciousness at all and ascribe actionsentirely to instinct leave many questionsunanswered. (Các giải thích về hành vi động vật bỏ qua bất cứ loạiý thức nào và gán cho các hành động hoàn toàn do bản năng để lại nhiều câu hỏi chưa được trả lởi) Other behaviorsthat may indicatesome cognition includetool use (Các hành vi khác có thể chỉ ra một số nhận thức bao gồm việc sử dụng công cụ)
  • 83. Question 40: Which of the following is NOT discussed as an ability animals are thought to have? A. Selecting among choices B. Anticipating events to come C. Remembering past experiences D. Communicating emotions Điều nào sau đây KHÔNG được thảo luận là khả năng mà động vật có? Lựa chọn trong số các lựa chọn Dự đoán được sự sắp đến Nhớ các việc trong quá khứ Truyền đạt cảm xúc Dẫn chứng: Some animal behaviorists argue that certain animals can remember past events. anticipate future ones, make plans and choices, and coordinate activities within a group. (Một số nhà nghiên cứu hành vi của động vật tranh luận rằng một số động vật có thể nhớ các sự việc quá khứ, dự đoán sự việc tương lai, lên kế hoạch và đưa ra lựa chọn và phối hợp các hoạt động trong nhóm)
  • 84. Question 41: What is the purpose of the honeybee dance? A.To determine the quantity of food at a site B.To communicate the location of food C.To identify the type of nectar that is available D.To identify the type of nectar that is available Mục đích của vũ điệu ong mật? Để xác định số lượng thức ăn ở một địa điểm Để truyền đạt thông tin về vị trí thức ăn Để tăng tốc độ di chuyển đến nguồn thức ăn Để nhận ra loại mật hoa sẵn có. Dẫn chứng: “Honeybees communicate the sources of nectar to one another by doing a dance in a figure- eight pattern.” (Loài ong mật truyền đạt thông tin về các nguồn mật hoa với nhau bằng việc thực hiện vũ điệu hình số 8.)
  • 85. Question42: The word “yet” in line 15 is closest in meaning to A. however B. since C. generally D. so far however: tuy nhiên since: từ khi generally: nói chung, thông thưởng so far: cho đến bây giở “But in one study, when experimenters kept changing the site of the food source, each time moving the food 25 percent farther from the previous site, foraging honeybees began to anticipate where the food source would appear next. When the researchers arrived at the new location, they would find the bees circling the spot, waiting for their food. No one has yet explained how bees, whose brains weigh four ten- thousandths of an ounce, could have infeưed the location of the new site.“ (Nhưng trong một nghiên cứu, khi các nhà thí nghiệm tiếp tục thay đổi địa điểm đặt thức ăn, mỗi lần di chuyển thức ăn cách xa hơn 25% so với địa điểm trước, những con ong mật săn mồi bắt đầu dự đoán nguồn thức ăn sẽ xuất hiện ở đâu tiếp theo. Khi các nhà nghiên cứu đến địa điểm mới, họ thấy những con ong vây quanh nơi đó, chở đợi thức ăn. Cho đến bây giở chưa ai giải thích được làm thế nào mà loài ong có bộ não với trọng lượng bốn phần mưởi nghìn của một ounce, có thể suy đoán được vị trí của địa điểm mới.) Do đó: yet ~ so far
  • 86. Question 43: It can be inferred from the passage that brain size is assumed to . A. be an indicator of cognitive ability B. vary among individuals within a species C. be related to food consumption D. correspond to levels of activity Chúng ta có thể suy ra từ bài đọc rằng kích thước não bộ được cho là . một dấu hiệu của khả năng nhận thức đa dạng giữa các thành viên trong cùng một loài liên quan đến việc tiêu thụ thức ăn tương đương với mức độ hoạt động Dẫn chứng: “No one has yet explained how bees, whose brains weigh four ten-thousandthsof an ounce could have inferred the location of the new site.” (Chưa ai giải thích được làm thế nào mà loài ong có bộ não với trọng lượng bốn phần mưởi nghìn của một ounce, có thể suy đoán được vị trí của địa điểm mới.)
  • 87. Question 44. What is the passage mainly about? A.The difficulties of industrializationin North America B.The influence of changes in manufacturing on the growth of urban centers C.The rapid speed of industrialization inNorth America D.Improved ways of organizing the manufacturing of goods Bài đọc chủ yếu nói về điều gì? Khó khăn của việc công nghiệp hóa ở Bắc Mỹ Ảnh hưởng của thay đổi trong việc sản xuất vào sự phát triển của các trung tâm đô thị Tốc độ công nghiệp hóa nhanh chóng ở Bắc Mỹ. Những cách cải tiến tổ chức việc sản xuất hàng hóa Dẫn chứng: - Industrialization cameto the United State after 1790 as North American entrepreneurs increased productivityby reorganizing work and buildingfactories. These innovationsin manufacturing boosted output and living standardsto an unprecedented extent ... (Công nghiệp hóa đã đến nước Mỹ sau năm 1790 khi các doanhnhân Bắc Mỹ tăng năng suất bằng cách tổ chức lại công việc và xây dựng nhà máy. Những cải tiến này trong sản xuất đã giúp tăng sản lượng và mức sống đến mức độ chưa từng thấy...) For tasks that were not suited to the outwork system, entrepreneurs created an even more important new organization, the modem factory, which used power-driven machines and assembly-line techniques to turn out large quantities of well-made goods. (Đối với các nhiệm vụ không phù hợp với hệ thống gia công, các chủ thầu thậm chí đã tạo ra một tổ chức mới quan trọng hơn, nhà máy hiện đại, mà sử dụng các máy chạy bằng năng lượng và các kỹ thuật dây chuyền lắp ráp để tạo ra số lượng lớn hàng hóa được chế tạo tốt.)
  • 88. Question45. The word “scope” in the second paragraph is closest in meaning to . A.value B. popularity C. extent D. diversity Từ “scope” trong đoạn 2 gần nghĩa nhất với . giá trị sự phổ biến quy mô, phạm vi Sự đa dạng “The impressive gain in output stemmed primarily from the way in which workers made goods, since the 1790’s, North American entrepreneurs - even without technological improvements - had broadened the scope of the outwork system that made manufacturing more efficient by distributing materials to a succession of workers who each performed a single step of the production process” (Sự gia tăng ấn tượng về sản lượng xuất phát chủ yếu từ cách công nhân sản xuất hàng hóa, từ những năm 1790 thì các doanh nhân Bắc Mỹ - thậm chí không có cải tiến công nghệ - đã mở rộng phạm vi của hệ thống gia công giúp việc sản xuất hiệu quả hơn bằng cách phân phát chất liệu cho một dây chuyền công nhân mà mỗi người thực hiện một bước duy nhất trong quá trình sản xuất.) Do đó: scope ~ extent
  • 89. Question 46. The authormentions the shoe industry in the second paragraph to provide an example of how entrepreneurs increased output by using an extended outwork system entrepreneurs used technologicalimprovementsto increase output rural workers responded to “shoe bosses” changes in the outwork system improved the quality of shoes Question 46 Chọn đáp án A Tác giả đề cập đến công nghiệp sản xuất giày trong đoạn 2 để cung cấp ví dụ về cách thức ..... các doanh nhân tăng sản lượng bằng việc sử dụng hệ thống gia công mở rộng các doanh nhân sử dụng cài tiến công nghệ để tăng sản lượng công nhân ở nông thôn phản ứng với “các ông chủ xưởng giày” những thay đổi trong hệ thống gia công đã cải thiện chất lượng giày Dẫn chứng: The impressive gain in output stemmed primarily from the way in which workers made goods, since the 1790’s, North American entrepreneurs - even without technological improvements - had broadened the scope of the outwork system that made manufacturing more efficient by distributingmaterials to a succession of workers who each performed a single step of the productionprocess. For example, during the 1820’s and 1830’s the shoe industry greatlyexpanded the scale of the outwork system.... However, it also dramatically increased the output of shoes while cutting their price. (Sự gia tăng ấn tượng về sản lượng xuất phát chủ yếu từ cách công nhân sản xuất hàng hóa, từ những năm 1790 thì các doanh nhân Bắc Mỹ - thậm chí không có cải tiến công nghệ - đă mở rộng phạm vi của hệ thống gia công giúp việc sản xuất hiệu quả hơn bằng cách phân phát chất liệu cho một dây chuyền công nhân mà mỗi người thực hiện một bước duy nhất trong qui trình sản xuất. Chẳng hạn như, trong những năm 1820 và 1830, ngành công nghiệp sản xuất giày đã mở rộng quy mô của hệ
  • 90. Question47. All of the following are true of the outwork system EXCEPT . A. It involved stages of production. B. It was more efficient than the systems used before 1790. C. It made many employers less powerful than they had been before. D. It did not necessarily involve any technological improvements. Tất cả những điều sau đúng về hệ thống gia công NGOẠI TRỪ . Nó bao gồm các giai đoạn sản xuất Nó hiệu quả hơn các hệ thống đã được dùng trước 1790 Nó đã làm cho nhiều ông chủ ít quyền lực hơn trước đây Nó không nhất thiết có cải tiến công nghệ Dẫn chứng: - The impressive gain in output stemmed primarily from the way in which workers made goods, since the 1790’s. North American entrepreneurs - even without technological improvements - had broadened the scope of the outwork system that made manufacturing more efficient by distributing materials to a succession of workers who each performed a single step of the production process. => A, B, D đúng This system of production made the employer a powerful “shoe boss” => C sai
  • 91. Question 48. The word “prolific” in the last paragraph is closest in meaning to . A. famous B. productive C. self-employed D. progressive famous (adj): nổi tiếng productive (adj): sản xuất nhiều, sáng tác nhiều, phát minh nhiều self-employed (adj): tự kinh doanh progressive (adj): tiếp diễn, tiếng bộ “As early as 1782 the prolific Delaware inventor Oliver Evans had built a highly automated, laborsaving flour mill driven by water power.” (Ngay từ năm 1782, nhà phát minh có nhiều sáng chế ở tiểu bang Delaware tên là Oliver Evans đã phát minh máy xay bột tự động giúp tiết kiệm sức lao đông được điều khiển bởi năng lượng nước.) Do đó: prolific ~ productive
  • 92. Question 49. According to the passage, how did later mills differ from the mills built by Oliver Evans? A.They were located away from large cities. B.They used new technology to produce power. C.They did not allow flour to cool before it was placed in Barrels. D.They combined technology with the outwork system. Theo bài đọc, máy xay sau này khác với máy xay được phát minh bởi Oliver Evans như thế nào? Chúng được đặt ở xa các thành phố lớn. Chúng dùng công nghệ mới để tạo ra năng lượng. Chúng không cho phép bột nguội trước khi được bỏ vào khoang. Chúng kết hợp công nghệ với hệ thống gia công. Dẫn chứng: “Subsequently, manufacturers made use of new improved stationary steam engines to power their mills...” (Sau đó, các nhà sản xuất đã sử dụng động cơ hơi nước cố định được cải tiến để cung cấp năng lượng cho các máy xay...)
  • 93. Question 50 The passage mentions which of the following as a result of improvements in factory machinery? A.It become easier for factory’ owners to find workers and customers. B.Manufacturers had to employ more highly skilled workers. C.The amount of power required for factories operate was reduced. D.Factories could operate more than one engine at a time. lượng cho các máy xay...) Bài đọc đề cập đến điều nào sau đây là kết quả của sự cải thiện máy móc nhà máy? Các ông chủ nhà máy trở nên dễ dàng tìm công nhân và khách hàng. Nhà sản xuất phải thuê nhiều công nhân có tay nghề cao. Năng lượng cần cho nhà máy hoạt động thì giảm xuống. Nhà máy không thể hoạt động nhiều hơn 1 động cơ vào một thởi điểm Dẫn chứng: This new technology enabled them to build factories in the nation’s largest cities, taking advantage of urban concentrations of inexpensive labor, good transportation networks, and eager customers.
  • 94.
  • 95.
  • 96. Question 1: Mr. Lam is a cycle driver in Ho Chi Minh City, who usually has a ________ working day. A. business B. busyC. busily D. busying A. business /’biznis/ (n): công việc, việc buôn bán kinh doanh B. busy /’bizi/ (a): bận, nhộn nhịp C. busily/’buzili/ (a): một cách bận rộn, một cách nhộn nhịp D. busying: không tồn tại từ này => Căn cứ vào mạo từ “a” cùng danh từ theo sau nên vị trí còn trống cần một tính từ. Từ đó ta chọn B Dịch nghĩa: Ông Lam là một người lái xích lô ở TP. Hồ Chí Minh, người mà thường có một ngày làm
  • 97. Question 2: You couldn’t give me a helping hand, _____? A. could you B. do you C. couldn’t you D. don’t you Dịch nghĩa: “Bạn không thể giúp tôi một tay phải không?” Câu bắt đầu với you couldn’t thì thành lập hỏi đuôi sẽ dùng could you.
  • 98. Question 3: The employees have been working______out to get the job finished ahead of the deadline. A. narrow B. big C. flat D. large A. narrow /ˈnærəʊ/ (a): chật hẹp B. big /bɪɡ/ (a): to lớn C. flat /flæt/ (a): phẳng D. large /lɑːdʒ/ (a): rộng lớn Ta có: flat out: hoàn toàn, hết sức Tạm dịch: Các nhân viên đã làm việc hết sức để hoàn thành công việc trước thời hạn cuối.
  • 99. Question 4: If you get a laptop as a reward, what will you use it ____? A. by B. for C. to D. with * Dịch nghĩa: Nếu bạn nhận được một chiếc máy tính xách tay như một phần thưởng, bạn sẽ sử dụng nó để làm gì? * Căn cứ vào: use for: dùng để làm gì
  • 100. Question 5: The bank will insist you produce a driving________or passport as a form of ID. A. diploma B. certificate C. degree D. licence A. diploma /dr pleume/ (n): chứng chỉ, văn băng (cái do trường đại học, cao đăng cấp sau khi vuợt qua một kỳ thi đặc biệt hoặc một khóa học nảo đó) B. certificate /se trfiket/ (n): giấy chứng nhận, chứng chỉ (tai liệu chính thức có chúa thông tin chinh xác, chuẩn xác như giấy khai sinh, giay dăng kí kết hôn,. hoặc tài liệu được cung cấp bởi nhà trường sau khi vượt qua kỳ thi) C. degree /dr gri / (n). bằng cấp (bằng có khi mức độ, trình đo,năng lưc của ai đó sau khi tốt nghiệp dại học, cao đảng) D. licence/latsns/ (n): giấy phép (thể hiện quyền có thể am gl, so hữu cái gì hay sử dụng cải gì một cach hợp pháp) Tạm dịch: Ngân hàng sẽ yêu cầu bạn xuất trình giấy phép lài xe hoặc hó chiều dưới dạng ID.
  • 101. Question 6: ____ the TRUMPET is the smallest brass instrument, it can play the highest notes of all the brass instruments and often plays in marches or fanfares. A. Because B. Although C. However D. Since Dịch nghĩa: Mặc dù kèn là nhạc cụ bằng đồng nhỏ nhất, nhưng nó có thể chơi những nốt cao nhất trong tất cả các nhạc cụ bằng đồng và thường chơi trong các cuộc diễn hành hoặc kèn lệnh. Xét các đáp án: A. Because → Mệnh đề chỉ nguyên nhân, dịch là - bởi vì, nếu cho vào câu sẽ trở nên vô nghĩa B. Although → Mệnh đề chỉ sự nhượng bộ, các liên từ được dịch là mặc dù: Although = Even though = Though = In spite that + clause C. However → however là tuy nhiên, nếu cho vào câu sẽ trở nên vô nghĩa D. Since → since = because (bởi vì), nếu cho vào câu sẽ trở nên vô nghĩa
  • 102. Question 7: His sister is not only a (n) ______________ singer but also a distinguished painter. A. famous opera Italian B. opera famous Italian C. famous Italian opera D. Italian famous opera *Theo quy tắc trật tự tính từ trong câu: OSASCOMP : famous - Opinion; Italian - Origin *Note: Opera (n): nghệ thuật, nhạc kịch ô-pê-ra -Tính từ luôn đứng trước danh từ để bổ nghĩa cho danh từ Tạm dịch: Chị anh ấy không chỉ là một ca sĩ opera người Ý nổi tiếng mà còn là một hoạ sĩ lỗi lạc.
  • 103. Question 8: Mark invented a new game, but it never really _________________ with people. A. called for B. caught on C. cut off D. came across A. called for: yêu cầu B. caught on: trở nên phổ biến C. cut off: ngắt, cắt (điện) D. came across: tình cờ gặp Dịch nghĩa: Mark đã phát minh ra một trò chơi mới nhưng nó không bao giờ thực sự trở nên phổ biến với mọi người.
  • 104. Question 9: He fell down when he______towards the pagoda. A. runB. runsC. was runningD. had run “ When” dùng để diễn tả 1 hành động đang xảy ra 1 hành động khác xen vào Thì quá khứ tiếp diễn Tạm dịch: Anh ấy bị ngã khi chạy về phía ngôi chùa.
  • 105. Question 10. She will apply for a job _______. A. when she is graduating from university B. until she graduated from university C. after she had graduated from university D. as soon as she graduates from university Xét các đáp án ta thấy: - Động từ của vế câu cho trước ở trong đề bài ở thì tương lai diễn tả một hành động chưa xảy ra hay sẽ xảy ra ở trong tương lai nên ở vế sau, không thể chia động từ ở thì quá khứ, quá khứ tiếp diễn hay quá khứ hoàn thành. loại A, B, C 4 đáp án D đúng (phối thì hiện tại đơn với tương lai đơn) Dịch: Cô ta sẽ đi xin việc ngay sau khi anh ta tốt nghiệp.
  • 106. Question 11: It ___ without saying that Mrs Ngoc Anh is a very enthusiastic teacher. I love her so much. A. goesB. comes C. appears D. gets A. goes / ɡoʊz/ (v): đi B. comes /kʌmz/ (v): đến C. appears /əˈpɪərz/ (v): xuất hiện D. gets /gets/ (v): có được Cấu trúc: It goes without saying: chắc chắn, hiển nhiên, khỏi phải nói Dịch nghĩa: Khỏi phải nói rằng cô Trang Anh là một giáo viên rất nhiệt tình. Tôi yêu cô ấy rất nhiều.
  • 107. Question 12: As soon as Ferlin came to party, he immediately set his _______on Melin. Maybe he was captivated by her. A. eyesB. heart C. decision D. feeling A. eyes /aɪs/ (n): đôi mắt B. heart /hɑːt/ (n): trái tim C. decision /dɪˈsɪʒən/ (n): quyết định D. feeling /ˈfiːlɪŋ/ (n): cảm xúc => Cấu trúc: Clap/lay/set eyes on sb/sth: để ý, để mắt tới cái gì/ai đó ngay từ lần đầu gặp Dịch nghĩa: Ngay khi Ferlin đến bữa tiệc, anh lập tức để ý tới Melin. Có lẽ anh ấy đã bị cô quyến rũ.
  • 108. Question 13. English ___________in many countries. A. are spoke B. is spoke C. are spoken D. is spoken Phương pháp giải: Kiến thức: Câu bị động: Giải chi tiết: Câu bị động thì Hiện tại đơn . Hành động được nhấn mạnh là tiếng Anh được nói ở nhiều nước S + is/are/am + V-ed/V3 Tạm dịch: Tiếng Anh được nói ở nhiều nước
  • 109. Question 14: ___________ all the lights and other electric devices, we left the classroom. A. Having been turned off B. Turning off C. To have turned off D. Having turned off Xét các đáp án: A. Having been turned off => không dùng dạng bị động ở đây vì đằng sau đã có tân ngữ là “all the lights and other electric devices” => đáp án A sai. B. Turning off => Sai vì ở đây 2 vế đồng chủ ngữ, ta lược bỏ chủ ngữ vế đầu, động từ đưa về dạng Ving khi nó mang nghĩa chủ động. Tuy nhiên, hành động ở vế trước xảy ra trước nên ta dùng “Having Vp2”. C. To have turned off => từ “to” đứng đầu tạo thành mệnh đề chỉ mục đích. Nhưng xét vào ngữ nghĩa, câu diễn tả 2 hành động xảy ra trước và sau, không phải nói về mục đích của hành động. D. Having turned off => Đúng. Tạm dịch: Sau khi tắt hết các đèn và các thiết bị điện tử khác, chúng tôi rời khỏi lớp học.
  • 110. Question 15: ___________ you study for these exams, the better you will do. A. The harder B. The more C. The hardest D. The more hard HD: Dịch nghĩa: Bạn càng học chăm chỉ cho các kỳ thi này, bạn sẽ càng làm tốt hơn. Xét các đáp án: A. The harder / the better → Cấu trúc so sánh kép khi nói về 2 người hoặc sự vật: The more/-er + S + V, the more/-er + S + V B. The more → Sai cấu trúc so sánh kép C. The hardest → Sai cấu trúc so sánh kép D. The more hard → Sai cấu trúc so sánh hơn (hard là từ 1 âm tiết, good là trường hợp bất quy tắc)
  • 111. Question 16. Ben: “You didn’t go to school yesterday, did you?” Jasmine: “ . I saw you, but you were talking to someone” A. No, I didn’t B. Yes, I didn’t C. Let me see D. I went Kỹ năng: Dịch Giải thích: Ben: “Hôm qua, cậu không đi học phải không?” Jasamine: “Tớ có đi. Tớ đã nhìn thấy cậu, nhưng cậu đang nói chuyệnvới ai đó.” Không, tớ đã không đi Sai cấu trúc Để xem nào
  • 112. Question 17. Mother: “How come you didn’t tell me that you would quit your job?” Lisa: “ .” A.I’d love to tell you now B.Because I knew that you would make a fuss about it C.I have no idea D.Because I’m so bored with it Kỹ năng: Giải thích: Mẹ: “Sao con có thể không nói với mẹ rằng con bỏ việc?” Lisa: Bởi vì con biết là mẹ sẽ làm ầm lên Con muốn nói với mẹ bây giờ. Con không biết. D.Bởi vì con chán chuyện đấy rồi
  • 113. Question 18: A. attitude B. manager C. invention D. company A. attitude/ˈætɪtjuːd/(n): thái độ (từ này có trọng âm rơi vào âm tiết đầu tiên) B. manager /ˈmænɪdʒər/ (n): người quản lý (từ này có trọng âm rơi vào âm tiết đầu tiên. Vì theo quy tắc nếu tất cả các âm mà ngắn hết thì trọng âm rơi vào âm tiết đầu.) C. invention /ɪnˈvenʃn/ (n): sáng chế (từ này có trọng âm rơi vào âm tiết thứ hai. Vì theo quy tắc đuôi -ion làm trọng âm rơi vào trước âm đó.) D. company /ˈkʌmpəni/: công ty (từ này có trọng âm rơi vào âm tiết đầu tiên. Vì theo quy tắc nếu tất cả các âm mà ngắn hết thì trọng âm rơi vào âm tiết đầu.) => Đáp án C có trọng âm rơi vào âm tiết thứ hai, các phương án còn lại có trọng âm rơi vào âm tiết thứ nhất.
  • 114. Question 19: A. carry B. remove C. protect D. consist A. carry /ˈkæri/ (v): mang (từ này có trọng âm rơi vào âm tiết đầu tiên. Vì theo quy tắc nếu tất cả các âm mà ngắn hết thì trọng âm rơi vào âm tiết đầu.) B. remove /rɪˈmuːv/ (v): loại bỏ (từ này có trọng âm rơi vào âm tiết thứ hai. Vì theo quy tắc, trọng âm ưu tiên rơi vào nguyên âm dài /uː/.) C. protect /prəˈtekt/ (v): bảo vệ (từ này có trọng âm rơi vào âm tiết thứ hai. Vì theo quy tắc trọng âm không rơi vào âm /ə/.) D. consist /kənˈsɪst/ (v): bao gồm (từ này có trọng âm rơi vào âm tiết thứ hai. Vì theo quy tắc trọng âm không rơi vào âm /ə/.) Đáp án A có trọng âm rơi vào âm tiết thứ nhất, các phương án còn lại có trọng âm rơi vào âm tiết thứ hai.
  • 115. Question 20: A. finished B. developed C. defeated D. looked A. finished /ˈfɪnɪʃt/ B. developed/dɪˈveləpt/ C. defeated /dɪˈfiːtid/ D. looked /lʊkt/ Giải thích: Có 3 cách phát âm ed chính: 1. Đuôi /ed/ được phát âm là /t/: Khi động từ có phát âm kết thúc là /s/, /f/, /p/, /ʃ/, /tʃ/, /k/ và những động từ có từ phát âm cuối là “s”. 2. Đuôi /ed/ được phát âm là /id/: Khi động từ có phát âm kết thúc là /t/ hay /d/. 3. Đuôi /ed/ được phát âm là /d/ với những trường hợp còn lại.
  • 116. Question 21: A. sale B. chalk C. dateD. plane A. sale /seɪl/ B. chalk /tʃɔːk/ C. date /deɪt/ D. plane /pleɪn/
  • 117. Question 22. In the 1980s TV viewers began to hook up videocassette players to their TVs. A. combine B. connect C. fasten D. blend Kỹ năng: Đồng/trái nghĩa Giải thích: hook up = connect: kết nối combine: kết hợp fasten: buộc chặt, thắt blend: pha trộn
  • 118. Question 23. The police have not had time to complete their investigations, but they have concluded tentatively that the explosion was caused by a bomb. A. temporally B. intentionally C. certainly D. hesitantly Kỹ năng: Đồng/trái nghĩa Giải thích: tentatively = hesitantly: ngập ngừng, không chắc chắn temporally: tạm thời intentionally:cố tình certainly: chắc chắn
  • 119. Question 24. Humans can use language deceptively by telling lies or half-truths. A. in an honest way B. in a dishonest way C. for a serious purpose D. at the wrong time A Kỹ năng: Đồng/trái nghĩa Giải thích: deceptively: một cách dối trá, dễ nhầm lần>< in an honest way: một cách thật thà Question 25. Tired of being a tiny cog in a vast machine, he handed in his resignation. A. an important person B. a large piece of equipment C. a small group of people D. a significant instrument Kỹ năng: Đồng/trái nghĩa Giải thích: a tiny cog in a vast machine: một phần nhỏ, kém quan trọng>< an important person: một người quan trọng
  • 120. Question 26: The only way to eliminate world terrorism is by united opposition. A. Only with united opposition could we eliminate terrorism. B. Only by united opposition can we eliminate terrorism. C. Only in this way can world terrorism be eliminated. D. Only then can we eliminate terrorism. Dịch nghĩa: Cách duy nhất để loại bỏ khủng bố thế giới là thành lập một phe liên minh chống lại. Xét các đáp án: A. Only with united opposition could we eliminate terrorism. → Trợ động từ là “can”, không phải là “could”. B. Only by united opposition can we eliminate terrorism. → Cấu trúc đảo ngữ: Only by V-ing/N + Auxiliary + S + V. C. Only in this way can world terrorism be eliminated. → Thiếu “by united opposition”. D. Only then can we eliminate terrorism. → Thiếu “by united
  • 121. Question 27: Collin is not very good at English. He failed the English exam last week. A. If Collin were good at English, he wouldn’t have failed the English exam last week. B. If Collin were better at English, he would pass the English exam last week. C. Should Collin be good at English, he would not fail the English exam last week. D. Had Collin been better at English, he would’ve passed the English exam last week. Đáp án A Ta thấy ở câu đề bài cho một động từ ở thì hiện tại, một động từ ở thì quá khứ. => Nên suy ra ở đây ta sẽ dùng câu điều kiện trộn giữa loại 2 và loại 3. - If S + V - qkđ, S + would + have + V-pp Tạm dịch: Collin không giỏi tiếng Anh lắm. Anh ấy đã trượt kỳ thi tiếng Anh tuần trước. = A. Nếu Collin giỏi tiếng Anh, anh ấy đã không trượt kỳ thi tiếng Anh tuần trước.
  • 122. Question 28: Papyrus (A) was(B)usedful for making not only paper (C) but also sails, baskets, (D) and clothing. (useful=> used) Cấu trúc: be used for + V-ing: được sử dụng làm gì Question 29: (A) Because the boy spent (B) too much time (C) surfing the Internet and playing computer games, (D) she didn’t pass the final exam. Cấu trúc: many + N- đếm được sốnhiều: nhiều; much + N - không đếm được + “time - thời gian” là danh từ không đếm được nên ta dùng “much” The boy là chủ ngữ đại từ thay thế “he”
  • 123. Question 30: According to (A) most medical experts, massage (B) relieves pain and anxiety, eases depression and (C) speeding up recovery from (D) illnesses. Đây là câu trúc song song nên các động từ “relieve, ease và speed up” trong câu này đều được chia theo danh từ “massage” Dịch: Theo nhiều chuyên gia y học, việc mát-xa làm giảm cơn đau và lo lắng, làm giảm chứng trầm cảm và đẩy nhanh việc phục hồi bệnh tật. (speeding up => speeds up)
  • 124. Question 31: It was impossible that she forgot to wear the helmet. A. She should have worn the helmet B. She must have worn the helmet C She might have forgot to wear the helmet D. She needn’t have forgot wearing the helmet Dịch nghĩa: Anh ta không thể nào đã quên đội mũ bảo hiểm được. A. Cô ta lẽ ra nên đội mũ bảo hiểm. (Loại vì câu không mang nghĩa khuyên bảo) B. Cô ta chắc hẳn đã đội mũ bảo hiểm. (Đúng) C. Cô ta có lẽ đã quên đội mũ bảo hiểm. (Sai vì câu không mang nghĩa dự đoán) D Cô ta lẽ ra không cần thiết phải quên đã đội mũ bảo hiểm. (Sai nghĩa so với câu gốc)
  • 125. Question 32: Cindy said: “I haven’t seen John since last month.” A. Cindy said she hasn’t seen John since the previous month. B. Cindy said she hadn’t seen John since the previous month. C. Cindy said she wasn’t seen John since the previous month. D. Cindy said she doesn’t see John since the previous month. HD: Dịch nghĩa: Cindy nói: “Tôi không gặp John kể từ tháng trước.” Xét các đáp án: A. Cindy said she hasn’t seen John since the previous month. → Vì là câu tường thuật nên hiện tại hoàn thành chuyển thành quá khứ hoàn thành. B. Cindy said she hadn’t seen John since the previous month. → Vì là câu tường thuật nên hiện tại hoàn thành chuyển thành quá khứ hoàn thành. C. Cindy said she wasn’t seen John since the previous month. → Không dùng dạng diễn đạt này. D. Cindy said she doesn’t see John since the previous month. → Không dùng dạng diễn đạt này với cấu trúc câu tường thuật.
  • 126. Question 33: He last visited London three years ago. A. He has been in London for three years. B. He hasn't visited London for three years. C.He didn't visit London three years ago. D.He was in London for three years. HD: “Anh ấy đến Luân Đôn lần cuối vào ba năm trước.” A. Anh ấy đã ở Luận Đôn ba năm rồi. (Sai vì nghĩa không giống với câu gốc.) B. Anh ấy đã không đến Luân Đôn 3 năm rồi. (Đúng) C. Anh ấy đã không đến Luân Đôn ba năm trước. (Sai vì nghĩa không giống với câu gốc.) D. Anh ấy đã ở Luân Đôn được ba năm. (Sai vì nghĩa không giống với câu gốc.)
  • 127. Question 34. Rapidly rising caseloads alarm researchers, who fear the virus may make its way across the globe. (34) _________, scientists cannot yet predict how many deaths may result. A. So B. However C. Because D. Although Kiến thức: Liên từ Giải thích: A. So S + V: vì vậy, vì thế B. However, S + V: tuy nhiên C. Because S + V: bởi vì D. Although S + V: mặc dù Tạm dịch: Các nhà nghiên cứu đang lo sợ rằng virus có thể xâm nhập khắp toàn cầu. Tuy nhiên, các nhà khoa học vẫn chưa thể dự đoán có bao nhiêu trường hợp tử vong. Chọn B.
  • 128. Question 35. The Wuhan coronavirus spreading from China is now likely to become a pandemic (35) _________ circles the globe, according to many of the world’s leading infectious disease experts. A. whoB. why C. which D. where Kiến thức: Đại từ quan hệ Giải thích: Dấu hiệu: “a pandemic” (đại dịch) là danh từ chỉ vật. Trong mệnh đề quan hệ: - who: thay thế cho danh từ chỉ người, đóng vai trò làm chủ ngữ - why: tại sao - which: thay thế cho danh từ chỉ vật - where: ở đâu Tạm dịch: Theo nhiều chuyên gia về bệnh truyền nhiễm hàng đầu thế giới, virus coronavirus ở Vũ Hán lây lan từ Trung Quốc có khả năng trở thành đại dịch trên toàn cầu. Chọn C.
  • 129. Question 36. A pandemic - an ongoing epidemic on two or more continents- may well have global consequences, despite the extraordinary travel restrictions and quarantines now imposed by China and (36) _________ countries, including the United States. A. other B. more C. another D. others Kiến thức: Từ vựng Giải thích: Sau chỗ trống là danh từ “countries” (các nước) là danh từ số nhiều. A. other + N (số nhiều): những cái khác B. more + N số nhiều: nhiều hơn C. another + N số ít: cái khác D. others = other + N (số nhiều): những cái khác Tạm dịch: Một đại dịch - một dịch bệnh đang diễn ra trên hai hoặc nhiều lục địa - có thể gây ra những hậu quả toàn cầu, bất chấp những hạn chế đi lại và cách ly bị bắt buộc bởi Trung Quốc và các nước khác, bao gồm cả Hoa Kỳ.
  • 130. Question 37. Scientists do not yet know how (37) _________ the new coronavirus is, however, so there is uncertainty about how much damage a pandemic might cause. A. unhealthy B. lethal C. dead D. wonderful Kiến thức: Từ vựng Giải thích: A. unhealthy (adj): không lành mạnh B. lethal (adj): làm chết người, gây chết người C. dead (adj): chết D. wonderful (adj): tuyệt vời Scientists do not yet know how (37) lethal the new coronavirus is, however, so there is uncertainty about how much damage a pandemic might cause. Tạm dịch: Tuy nhiên, các nhà khoa học vẫn chưa biết mức độ gây chết người của loại coronavirus mới này, do đó, vẫn chưa chắc chắn về mức độ thiệt hại mà một đại dịch có thể gây ra.